Download as pdf or txt
Download as pdf or txt
You are on page 1of 98

MBE PREVIEW

DIAGNOSTIC EXAM
QUESTIONS AND ANALYTICAL ANSWERS

MPQ 100 preview diagnostic exam and divider P.indd 1 12/22/2015 4:13:29 PM
MPQ 100 preview diagnostic exam and divider P.indd 2 12/22/2015 4:13:29 PM
MBE PREVIEW DIAGNOSTIC EXAM 1.

MBE PREVIEW DIAGNOSTIC EXAM


100 QUESTION EXAM

TIME—3 HOURS

You will be given three hours to work on this test. Be sure that the question numbers on your
answer sheet match the question numbers in your test book. You are not to begin work until
the supervisor tells you to do so.

Your score will be based on the number of questions you answer correctly. It is therefore to
your advantage to try to answer as many questions as you can. Give only one answer to each
question; multiple answers will not be counted. If you wish to change an answer, erase your
first mark completely and mark your new choice. Use your time effectively. Do not hurry, but
work steadily and as quickly as you can without sacrificing your accuracy.

YOU ARE TO INDICATE YOUR ANSWERS TO ALL QUESTIONS ON THE


SEPARATE ANSWER SHEET PROVIDED.

DIRECTIONS

Each of the questions or incomplete statements in this test is followed by four suggested
answers or completions. You are to choose the best of the stated alternatives. Answer all
questions according to the generally accepted view, except where otherwise noted.

For the purpose of this test, you are to assume that Article 2 and Revised Article 1 of the
Uniform Commercial Code have been adopted.

The Federal Rules of Evidence, as well as the Federal Rules of Civil Procedure, are deemed
to control.

The terms “Constitution,” “constitutional,” and “unconstitutional” refer to the federal


Constitution unless indicated to the contrary.

You are also to assume that there is no applicable statute unless otherwise specified;
however, survival actions and claims for wrongful death should be assumed to be available
where applicable. You should assume that joint and several liability, with pure comparative
negligence, is the relevant rule unless otherwise indicated.

DO NOT OPEN THE TEST UNTIL


YOU ARE INSTRUCTED TO DO SO.

MPQ 100 preview diagnostic exam and divider P.indd 1 12/22/2015 4:13:29 PM
MPQ 100 preview diagnostic exam and divider P.indd 2 12/22/2015 4:13:29 PM
MBE PREVIEW DIAGNOSTIC EXAM 3.

MBE Preview Diagnostic Exam


....................................................................................................................................................................................................................

1. A B C D 26. A B C D 51. A B C D 76. A B C D

2. A B C D 27. A B C D 52. A B C D 77. A B C D

3. A B C D 28. A B C D 53. A B C D 78. A B C D

4. A B C D 29. A B C D 54. A B C D 79. A B C D

5. A B C D 30. A B C D 55. A B C D 80. A B C D

6. A B C D 31. A B C D 56. A B C D 81. A B C D

7. A B C D 32. A B C D 57. A B C D 82. A B C D

8. A B C D 33. A B C D 58. A B C D 83. A B C D

9. A B C D 34. A B C D 59. A B C D 84. A B C D

10. A B C D 35. A B C D 60. A B C D 85. A B C D

11. A B C D 36. A B C D 61. A B C D 86. A B C D

12. A B C D 37. A B C D 62. A B C D 87. A B C D

13. A B C D 38. A B C D 63. A B C D 88. A B C D

14. A B C D 39. A B C D 64. A B C D 89. A B C D

15. A B C D 40. A B C D 65. A B C D 90. A B C D

16. A B C D 41. A B C D 66. A B C D 91. A B C D

17. A B C D 42. A B C D 67. A B C D 92. A B C D

18. A B C D 43. A B C D 68. A B C D 93. A B C D

19. A B C D 44. A B C D 69. A B C D 94. A B C D

20. A B C D 45. A B C D 70. A B C D 95. A B C D

21. A B C D 46. A B C D 71. A B C D 96. A B C D

22. A B C D 47. A B C D 72. A B C D 97. A B C D

23. A B C D 48. A B C D 73. A B C D 98. A B C D

24. A B C D 49. A B C D 74. A B C D 99. A B C D

25. A B C D 50. A B C D 75. A B C D 100. A B C D

MPQ 100 preview diagnostic exam and divider P.indd 3 12/22/2015 4:13:29 PM
MPQ 100 preview diagnostic exam and divider P.indd 4 12/22/2015 4:13:29 PM
MBE PREVIEW DIAGNOSTIC EXAM 5.

Question 1 Question 2

A “wind farm” was constructed on the open The plaintiff and defendant in a lawsuit were
land next to a gas station where a mechanic in an accident in which the plaintiff was injured.
is employed. The huge windmills generated As a result of the accident, the plaintiff incurred
electrical power and oscillated at low speeds, medical expenses of $100,000. At the time of the
resulting in very low-frequency but high-ampli- accident, the plaintiff and defendant both lived
tude sound waves that caused extreme discom- in State A. Before the action was filed, the plain-
fort in a substantial minority of people. The tiff moved permanently to State B. The plaintiff
mechanic is one of the people adversely affected then filed a negligence action against the defen-
by such sound waves; the vibrations from the dant in federal district court, with subject matter
windmills give him severe headaches and upset jurisdiction being based on diversity of citizen-
his immune system. ship. After the action was filed but before the
defendant was served with process, the defen-
If the mechanic brings an action for nuisance dant was transferred by his employer and moved
against the owners of the wind farm and loses, permanently to State B.
what is the most likely explanation?
For purposes of evaluating the court’s diver-
(A) Sound waves are not the kind of physical sity of citizenship jurisdiction, what are the
phenomenon on which liability for nui- citizenships of the two parties?
sance can be based.
(A) Both are citizens of State A.
(B) The owners of the wind farm cannot
generate power without producing the (B) The plaintiff is a citizen of State B and the
offending sound waves, despite their best defendant is a citizen of State A.
efforts to find an alternative technology.
(C) The plaintiff is a citizen of State A and the
(C) At least six out of 10 people are not affected defendant is a citizen of State B.
by low-frequency sound waves.
(D) Both are citizens of State B.
(D) The mechanic neither owns nor rents the
gas station at which he works.

GO ON TO THE NEXT PAGE

MPQ 100 preview diagnostic exam and divider P.indd 5 12/22/2015 4:13:29 PM
6. MBE PREVIEW DIAGNOSTIC EXAM

Question 3 Question 4

The defendant was charged with forging the The defendant was on trial for shooting and
signature of her employer on several checks killing a man at a restaurant after a violent
made payable to her best friend, a co-worker. argument. The prosecution calls a witness to
After discovering that the checks had been testify that she was standing in the parking lot,
forged, the employer called the defendant into and, as the defendant was leaving the restaurant,
his office and told her that he regretted trusting he dropped a gun into a nearby garbage can.
her with the task of paying his bills and could
not believe that his two most trusted employees Is the witness’s testimony regarding the defen-
devised a scheme to defraud him. The defendant dant’s conduct admissible?
did not respond to the statement and walked out
of the room. At trial, the prosecution wants to (A) Yes, as hearsay falling within the state-of-
offer the defendant’s silence as evidence of her mind exception.
guilt.
(B) Yes, because nonassertive conduct is not
How should the court rule? hearsay, and it is otherwise admissible.

(A) Admissible under an exception to the (C) No, because nothing was said.
hearsay rule for implied statements against
interest. (D) No, because the defendant’s acts constitute
assertive conduct.
(B) Admissible nonhearsay.

(C) Inadmissible, because failure to reply to


an accusation in a criminal case cannot be
used as an implied admission of a criminal
act.

(D) Inadmissible as violating the defendant’s


right against self-incrimination.

GO ON TO THE NEXT PAGE

MPQ 100 preview diagnostic exam and divider P.indd 6 12/22/2015 4:13:29 PM
MBE PREVIEW DIAGNOSTIC EXAM 7.

Question 5 Question 6

During the nighttime, a woman broke into the A consumer purchased an air compressor,
house of the victim with the intention of stealing designed for powering tools and inflating tires,
his diamond ring. When she could not find the to use in a remodeling project. The consumer
diamond ring, she became angry, lit a match to discovered that the reservoir and nozzle for a
a newspaper and threw it on the victim’s bed, paint compressor gun he owned fit the threads
setting the mattress on fire. The flames destroyed on the nozzle of his air compressor. He decided
the bed and a portion of the floor under the bed. to use the manufacturer’s compressor rather than
buying the compressor that went with the spray
In a common law jurisdiction, of which crime gun because he needed to paint a fence. After
or crimes is the woman guilty? painting for a few minutes, the paint reservoir on
the spray gun exploded, causing severe injuries
(A) Burglary only. to the consumer. The consumer subsequently
brought a strict products liability action against
(B) Arson only. the air compressor manufacturer to recover for
his damages.
(C) Burglary and attempted arson.
Evidence at trial revealed the following: (i)
(D) Burglary and arson. The spray gun reservoir had exploded because
the air compressor developed too much air
pressure for use in a spray device; it was for
this reason that the manufacturer did not
manufacture a spray painting attachment for its
compressor. (ii) The manufacturer purposefully
chose an unusual threading for its compressor
nozzle to prevent its use with most spray painting
systems, but the spray gun that the consumer
had was a foreign-manufactured model that
happened to use the same threading. (iii) When
used with the tools and attachments produced
by the air compressor manufacturer, the air
compressor was completely safe and suffered
from no design or manufacturing defects.
If the jury finds for the manufacturer, what is
the most likely explanation?
(A) The manufacturer exercised due care in the
design and manufacture of the paint sprayer.
(B) It was not foreseeable that the compressor
would be used by consumers to attach
power spray devices.
(C) The compressor suffered from no design
or manufacturing defects when used in the
intended manner.
(D) The consumer misused the compressor
when he attached it to his spray-painting
gun.

GO ON TO THE NEXT PAGE

MPQ 100 preview diagnostic exam and divider P.indd 7 12/22/2015 4:13:30 PM
8. MBE PREVIEW DIAGNOSTIC EXAM

Question 7 Question 8

On January 30, a company that designs and A manufacturer of washing machines


builds generators to standard industrial specifi- telephoned an engine company and asked for a
cations received a telephone call from a buyer price quote on an order for 250 washing machine
who ordered two generators at a price of $25,000 motors built according to the specifications
each. The parties agreed that delivery of the that the manufacturer had sent to the engine
first generator would be on March 15, and the company the previous week. The company
second on April 30. Payment was to be made no responded that it would sell the motors to the
more than 30 days after delivery. On March 12, manufacturer at a cost of $20 apiece, with
the company delivered the first generator, which delivery in 30 days. The manufacturer agreed to
the buyer accepted. On April 22, the company the terms and instructed that its order be entered.
completed the second generator but had not yet The engine company immediately started work
notified the buyer. On April 23, the buyer, having on the motors, and had made a substantial begin-
made no payment to the company, canceled ning on their manufacture (having completed
the agreement. The company brings an action 110 motors) when the manufacturer notified the
against the buyer for breach of contract. company that it would not honor the contract.
The engine company stopped work on the
How much should the company recover in motors and sued the manufacturer, which raised
damages? the Statute of Frauds as a defense. The engine
company responded that the specially manufac-
(A) Nothing. tured goods exception takes the contract out of
the Statute of Frauds.
(B) $25,000 only.
If the engine company loses, what is the most
(C) Damages for total breach of contract for the likely reason?
sale of two generators, because the buyer
accepted part performance. (A) It had not substantially completed work on
the contract.
(D) Damages for total breach of contract for the
sale of two generators, because the goods (B) It had completed work on less than half of
were made for the buyer. the motors.

(C) It stopped work on the motors before the


job was completed.

(D) It could have sold the motors in the


ordinary course of its business.

GO ON TO THE NEXT PAGE

MPQ 100 preview diagnostic exam and divider P.indd 8 12/22/2015 4:13:30 PM
MBE PREVIEW DIAGNOSTIC EXAM 9.

Question 9 Question 10

A developer prepared and recorded a subdivi- The federal government has complete jurisdic-
sion plan, calling for 100 home sites on half-acre tion over certain parkland located within a state.
lots. There were five different approved plans To conserve the wildlife inhabiting that land, the
from which a purchaser could choose the design federal government enacted a statute forbidding
of the home to be built on his lot. Each deed, all hunting of animals in the federal park. That
which referred to the recorded plan, stated that statute also forbids the hunting of animals that
“no residence shall be erected on any lot that has have left the federal park and have entered the
not been approved by the homeowners’ associa- state. A hunter has a state hunting license, autho-
tion.” rizing him to hunt deer anywhere in the state.
On land within the state located adjacent to the
A lawyer purchased a lot and built a home federal park, the hunter shot a deer he knew had
based on one of the approved designs. However, recently left the federal land.
many of the lots were purchased by investors
who wanted to hold the lots for investment If the hunter is prosecuted for violating the
purposes. Two years after the lots went on the federal hunting law, what is the strongest ground
market, one such investor sold her lot to an supporting the constitutionality of the federal
architect by a deed that did not contain any law forbidding the hunting of wild animals that
reference to the recorded plan nor the obligation wander off of federal property?
regarding approval by the homeowners’ associa-
tion. In fact, because very few residences had (A) This law is a necessary and proper means
been built in the subdivision since the lots were of protecting United States property.
first available for purchase, no homeowners’
association meetings had been held in two years. (B) The animals are moving in the stream of
interstate commerce.
The architect began building a very modern-
istic house on her one-half acre. When the (C) The police powers of the federal govern-
lawyer noticed the house being built, he brought ment encompass protection of wild animals.
an action to enjoin the construction.
(D) Shooting wild animals is a privilege, not a
For which party will the court rule? right.

(A) The architect, because her deed contained


no restrictive covenants.

(B) The architect, because any restrictive


covenant in her deed can only be enforced
by the opposite party to the covenant or that
person’s successor in title.

(C) The lawyer, because the recorded subdivi-


sion plan, taken with the fact that all lots
were similarly restricted and the architect
had notice of this, gave him the right to
enforce the covenant on her property.

(D) The lawyer, because his deed contained the


restrictive covenant.

GO ON TO THE NEXT PAGE

MPQ 100 preview diagnostic exam and divider P.indd 9 12/22/2015 4:13:30 PM
10. MBE PREVIEW DIAGNOSTIC EXAM

Question 11 Question 12

A husband and wife were both professors at After having much too much to drink at the
the United States Naval Academy. The husband corner bar, the defendant decided it was time to
made a speech criticizing United States foreign go home. However, because of her intoxication,
policy with respect to a Middle Eastern country she did not notice that she grabbed an expensive
and was dismissed from his teaching position and distinctive fur coat instead of her inexpen-
soon thereafter. Six months later, he accepted sive cloth coat from the bar’s coat room. The
new employment in another state. owner of the fur coat, who noticed the defendant
staggering out of the bar and down the street,
The man’s wife has commenced suit in federal ran out of the bar and shouted for the defendant
court claiming that the Naval Academy violated to stop. A police officer on foot patrol heard the
her husband’s right to due process and his right shout and immediately apprehended the defen-
of free speech when it fired him. dant.

What is the most likely ground on which the If charged with larceny of the fur coat, should
court will dismiss the suit? the defendant be found guilty?

(A) The political question doctrine, because (A) Yes, because her mistake was unreason-
foreign policy is in the domain of the ex- able.
ecutive branch.
(B) Yes, because voluntary intoxication is not a
(B) The case is moot, because the husband has defense to a charge of larceny.
a new job.
(C) No, because the defendant did not reach a
(C) The plaintiff has no standing. place of temporary safety and thus did not
complete the crime.
(D) There is no federal question presented.
(D) No, because she was too intoxicated to
realize the property did not belong to her.

GO ON TO THE NEXT PAGE

MPQ 100 preview diagnostic exam and divider P.indd 10 12/22/2015 4:13:30 PM
MBE PREVIEW DIAGNOSTIC EXAM 11.

Question 13 Question 14

For many years, a farmer has grown and Two partners bought a commercial building
harvested a sizeable wheat crop on his land. On from an owner. They paid cash for the building
May 1, the farmer entered into an agreement and took title as joint tenants with right of
with a bakery to supply the bakery with 10,000 survivorship. Several years later, the first partner
bushels of wheat at $15 a bushel. The agree- executed a mortgage on the building to secure
ment called for the farmer to deliver the bushels a personal loan to a bank. The second partner
to the bakery’s plant on or before September 1. had no knowledge of the mortgage to the bank.
In mid-July, it became apparent that the year’s The state in which the commercial building is
wheat crop grown in the United States was going located recognizes the lien theory of mortgages.
to be substantially smaller than originally antici- The first partner died before paying off his loan.
pated. As a result, the price of wheat on August He left all of his property by will to his daughter,
1 rose to $35 a bushel. On August 10, the farmer his only heir.
informed the bakery that he would be unable to
deliver any bushels to the bakery on September 1 Who has title to the commercial building?
because he had sold his entire harvested crop to
other consumers. (A) The second partner has title free and clear
of the mortgage.
If the bakery sues the farmer for breach of
contract, will the bakery prevail? (B) An undivided one-half is held by the second
partner free and clear of the mortgage, and
(A) Yes, as long as it files suit on or after Sep- the other one-half is held by the daughter,
tember 1. subject to the mortgage.

(B) Yes, regardless of whether it sues before or (C) An undivided one-half is held by the
after September 1. second partner and the other one-half by
the daughter, with both halves subject to the
(C) No, because the sharp increase in the price mortgage.
results in a commercial frustration of the
original deal, which excuses the farmer’s (D) The second partner has title to the entire
performance. property, with an undivided one-half being
subject to the mortgage.
(D) No, because the unforeseen rise in price is
a substantial change of circumstances that
excuses both parties.

GO ON TO THE NEXT PAGE

MPQ 100 preview diagnostic exam and divider P.indd 11 12/22/2015 4:13:30 PM
12. MBE PREVIEW DIAGNOSTIC EXAM

Question 15 Question 16

A drug dealer was convicted in federal court A defendant calls a secretary present at a
of possession of 10 kilos of cocaine with intent meeting between the defendant and the plain-
to distribute. She was sentenced to a prison term. tiff in which attempts were made to negotiate
Subsequently, a federal grand jury indicted the a settlement. The secretary offers to testify
dealer under a separate statute for conspiracy that the plaintiff said at the meeting, “Well,
to distribute the same 10 kilos of cocaine. She maybe it wasn’t fraud—I’ll settle for refund of
moved to dismiss the indictment. the purchase price plus $50,000.” The plaintiff
objects.
How should the court rule on her motion?
Is the offered testimony admissible?
(A) Grant it, because the Double Jeopardy
Clause protects her against a second pros- (A) Yes, as a statement by a party-opponent.
ecution for the same criminal conduct.
(B) Yes, because statements made in the course
(B) Grant it, because the Due Process Clause of settlement negotiations are admissible
protects her against double punishment for where relevant, even though offers to
the same criminal conduct. compromise are excluded.

(C) Deny it, because the Double Jeopardy (C) No, because of public policy.
Clause does not apply when the second
prosecution is for violation of a separate (D) No, because the statement was made in the
statute. office of the defendant, an attorney, and
hence privileged.
(D) Deny it, because each prosecution requires
proof of an element that the other does not.

GO ON TO THE NEXT PAGE

MPQ 100 preview diagnostic exam and divider P.indd 12 12/22/2015 4:13:30 PM
MBE PREVIEW DIAGNOSTIC EXAM 13.

Question 17 Question 18

A Hispanic woman was a citizen of a foreign A homebuyer filed a breach of contract action
country but had lived in the United States for 10 in a State A state court against the contractor (a
years. She had properly acquired resident alien corporation) that agreed to build the home and
status and regularly complied with all regula- sell it to the buyer. The homebuyer’s complaint
tions pertaining to resident aliens, such as annual joined as an additional defendant the contractor’s
registration at the local post office. Several of her agent who negotiated the contract and signed it
friends had been called for jury service and she on behalf of the contractor. The action sought
thought it would be interesting to serve on a jury. $150,000 in damages from both defendants. The
However, after extensive research she discovered contractor is a citizen of State B. The homebuyer
that no Hispanic female alien had ever served and the contractor’s agent are citizens of State A.
on a jury in the county where she lived; in fact, Six months after filing the action, the homebuyer
no person of that class had ever been in a venire dismissed the claims against the agent, leaving
panel. only the claims against the contractor. The
corporation immediately and appropriately filed
May she compel the county officials to include a notice of removal.
her and other persons of her class in venire
panels through a lawsuit in federal district court? May the contractor remove the action to
federal district court?
(A) Yes, because there is a consistent pattern of
discrimination against persons of her class. (A) Yes, because the action can be properly
removed at any time after the action was
(B) Yes, because alienage is a suspect category. filed.

(C) No, because certain privileges, such as jury (B) Yes, because, while the action was not
service, may be constitutionally denied to properly removable when it was filed, it
aliens. now can be properly removed to federal
district court.
(D) No, because she lacks standing, in that she
is not involved in a case or controversy (C) No, because, while the requirements for
where the lack of jurors in her class might diversity of citizenship jurisdiction now
prejudice her right to a fair trial. are satisfied, the time in which removal is
allowed has passed.

(D) No, because removal is allowed only if


federal subject matter jurisdiction exists at
the time the complaint is filed.

GO ON TO THE NEXT PAGE

MPQ 100 preview diagnostic exam and divider P.indd 13 12/22/2015 4:13:30 PM
14. MBE PREVIEW DIAGNOSTIC EXAM

Question 19 Question 20

A bolt of lightning struck a tree, causing it to A man and woman who shared an apartment
fall on a farmer’s fence which enclosed a pasture were arrested and charged with murder. The
containing the farmer’s large bull. The bull woman’s attorney moved to have the defendants
escaped through the broken fence and entered tried separately. The motion was granted. The
the neighbor’s property. It gored a hiker who was woman was tried first and was acquitted of the
crossing the neighbor’s property without permis- crime. The man’s trial began one month later.
sion. The man’s attorney called a witness, who was
now living with the man, to testify that, shortly
In the hiker’s action against the farmer based after she was acquitted of murder, the woman
on strict liability, is the hiker likely to prevail? had told the witness that she (the woman) had
killed the victim because the victim owed her
(A) Yes, because the bull caused harm while money, and that the man was not involved at all
trespassing on another’s property. in the victim’s death.
(B) Yes, because bulls have known dangerous Is the witness’s testimony admissible?
propensities.
(A) Yes, as a statement by a co-conspirator.
(C) No, because a bull is a domestic animal.
(B) Yes, as a declaration against penal interest.
(D) No, because the hiker was a trespasser.
(C) No, because it is hearsay not within any
exception.

(D) No, because the woman is estopped from


contradicting the jury verdict of acquittal.

GO ON TO THE NEXT PAGE

MPQ 100 preview diagnostic exam and divider P.indd 14 12/22/2015 4:13:30 PM
MBE PREVIEW DIAGNOSTIC EXAM 15.

Question 21 Question 22

A farmer fleeing from a bull that had gotten A small group of terrorists hijacked a foreign
out of its pen bolted for the nearest safety—the airliner. Ten prominent bankers from the United
fence between the pen and a melon grower’s States were among the airline passengers.
melon patch. Leaping over, the farmer landed The terrorists demanded that three individ-
on and shattered several melons. Hearing the uals imprisoned in a state prison be released.
melee, the melon grower ran from a hiding place Although these individuals were properly
where he had been waiting to try to catch some convicted of violation of state laws, the terrorists
children who had been stealing melons. He contended that they were political prisoners and
pointed his shotgun, which was loaded with rock that their convictions were merely a subterfuge
salt, and exclaimed, “Stop or I’ll shoot.” Just to prevent them from inciting the local populace.
then two juvenile melon thieves decided to make The President of the United States agreed to
their escape and ran from where they had been release the three prisoners in return for the
crouching. The melon grower aimed his shotgun freedom of the 10 bankers. He issued official
at them and prepared to fire but slipped, and pardons and ordered the state governor to release
the load of rock salt struck the farmer, whom the prisoners. The governor refused to do so,
the melon grower had recognized and had not contending that the President had overreached
intended to hit. his constitutional powers and had violated the
concept of federalism.
Has the melon grower committed an assault
against the farmer? What is the best argument for the governor’s
position?
(A) Yes, because he actually pointed the shot-
gun at the farmer. (A) The President had no power to make an
agreement with the terrorists because his
(B) Yes, if the farmer reasonably believed that treaty power extended only to dealings
the melon grower might shoot him. with the legitimate governments of other
nations.
(C) Yes, because assault is included within
the tort of battery, and the melon grower (B) The Constitution immunizes a governor
committed a battery when he shot the acting pursuant to a state constitution from
farmer in the shoulder. inconsistent orders by a federal official.

(D) No, because the shotgun was only loaded (C) The President’s duty to see that the laws
with rock salt. are faithfully executed prevents him from
ordering state officials to release prisoners.

(D) The President’s constitutional power to


grant pardons extends only to prisoners
who have violated federal laws.

GO ON TO THE NEXT PAGE

MPQ 100 preview diagnostic exam and divider P.indd 15 12/22/2015 4:13:30 PM
16. MBE PREVIEW DIAGNOSTIC EXAM

Question 23 Question 24

A state operates a nuclear power plant on a A homeowner agreed to sell his home to an
river. The plant uses river water for cooling and accountant. He gave the accountant a general
discharges water back into the river 10 degrees warranty deed and the accountant gave him
warmer than when it was taken in. While the $86,000, his asking price. The deed was
discharged water quickly cools, it has adversely recorded. A few years later, the accountant sold
affected the business of a downstream ice cutting the property to a doctor, who paid her $125,000
operator, located in a neighboring state. The ice for the property. To save on attorney’s fees,
cutter successfully urged his state to bring suit the accountant went to her local office supply
in the United States Supreme Court against the store to purchase a general warranty deed
state operating the power plant, alleging damage form. The store was out of those forms, but the
to its environment and seeking an injunction clerk suggested that she use the form labeled
against the thermal discharge. “Quitclaim Deed,” asserting that it would
transfer the title just as well as the other form.
What should the United States Supreme Court The accountant purchased the form and filled in
do? the blanks with the appropriate information she
copied from her old deed. The doctor did not
(A) Dismiss the action, because the suit is re- hire a lawyer to represent him in the purchase of
ally one by the ice cutter against the state the house.
operating the power plant and is barred by
the Eleventh Amendment. The doctor accepted the deed from the
accountant and gave her $125,000. Soon after
(B) Hear the matter on the merits, because the the doctor moved into the house, it was discov-
ice cutter’s state is suing in its own right ered that the homeowner’s title was not good.
and jurisdiction is proper. The true owner now demands that the doctor
vacate. Title is judicially determined to be with
(C) Dismiss the action, because it does not have the true owner, and the doctor is forced out.
original jurisdiction.
Does the doctor have any action against the
(D) Remand the case to the district court for homeowner or the accountant based on any
trial, because it does not have the time to covenant for title?
function as a trial court.
(A) Yes, the doctor can sue both and can re-
cover $125,000.

(B) Yes, the doctor can sue the homeowner, but


not the accountant, and can recover his full
$125,000.

(C) Yes, the doctor can sue the homeowner, but


not the accountant, and can recover only
$86,000.

(D) No, the doctor can sue neither the


homeowner nor the accountant.

GO ON TO THE NEXT PAGE

MPQ 100 preview diagnostic exam and divider P.indd 16 12/22/2015 4:13:30 PM
MBE PREVIEW DIAGNOSTIC EXAM 17.

Question 25 Question 26

A defendant is on trial for robbing a liquor A mountain lodge catered to cross-country


store. The store clerk testified that the defen- skiing enthusiasts. Every winter, the owner of
dant came into the store at about 11 p.m., the lodge, along with his assistants, marked
pointed a black gun with a silver grip at him, and groomed trails on his extensive lands. He
and demanded that he give him all the money charged a fee for a day’s use of the trails. He
in the cash register. The clerk testified that the also supplied equipment for those who needed
store was well lit and that the defendant was not it and provided lodging and meals for skiers
wearing a mask. The defendant’s attorney called staying more than one day. A woman on her first
the clerk’s employer to testify that when the clerk cross-country skiing outing was striding along a
gave her a report of the robbery, he told her that groomed trail when, with a loud crack, the rotten
the defendant pointed a silver gun with a black limb of a huge oak under which the trail passed
grip at him. separated from the tree because of the load of ice
and snow it bore. It came crashing down on top
How should the trial judge rule on the admis- of the woman, killing her instantly. The woman’s
sibility of this testimony? heirs brought a wrongful death action against the
lodge owner in a jurisdiction applying traditional
(A) Admissible, because it tends to show that rules for landowners and possessors of land.
the clerk is an unreliable witness.
If the lodge owner prevails, what is the most
(B) Admissible, because it has bearing on the likely explanation?
clerk’s truthfulness and veracity.
(A) The oak limb was a natural condition of the
(C) Inadmissible, because it is extrinsic land.
evidence of a prior inconsistent statement
on a collateral matter. (B) The lodge owner did not know that the oak
limb was rotten and was likely to fall.
(D) Inadmissible, because it is hearsay not
subject to any exceptions. (C) The lodge owner could not reasonably have
discovered the rotten limb by inspecting his
property.

(D) The ice and snow was an intervening force


that caused the limb to fall.

GO ON TO THE NEXT PAGE

MPQ 100 preview diagnostic exam and divider P.indd 17 12/22/2015 4:13:30 PM
18. MBE PREVIEW DIAGNOSTIC EXAM

Question 27 Question 28

A textile company and the local textile union A hotelier opening a new inn in the Pacific
were involved in a bitter labor dispute that had Northwest sent letters to all known hotel and
required the governor to use the National Guard. motel suppliers on June 1, alerting them to his
The Guard took up positions surrounding the need for such items as ice buckets, televisions,
company’s main plant and issued orders that no linen, and mattresses. The hotelier received a
more than 20 picketers could picket at any one letter dated June 8 from a hotel supply company,
time and that all employees who were on strike stating that the company had 250 ice buckets
were forbidden to cross the perimeter estab- left in stock and will sell them to the hotelier
lished by the Guard. In spite of this order, union for $1 each. The company added that it must
marchers approached the plant and refused to receive the hotelier’s answer by November 1 and
obey the commander’s order to disperse. As they will hold the ice buckets for the hotelier until
crossed the perimeter, the Guard was ordered to then. On July 1, the company sold 200 of the
fire on the marchers. As a result, seven strikers ice buckets to a competing hotel chain, which
were injured. had recently opened a hotel on the East Coast.
On July 2, the company sent the hotelier a fax
The seven strikers brought a claim for stating it had only 50 ice buckets left for sale.
damages against the state, which has abolished The hotelier received the fax that day, but put it
governmental immunity. Evidence indicates that aside and never read it. On July 10, the hotelier
the use of weapons was not necessary to protect notified the company that he was accepting the
anyone, but that the Guard used the weapons company’s offer to sell 250 ice buckets. The
because it had previously experienced trouble in company, upon receiving the hotelier’s accep-
controlling civil disorders. tance, shipped the remaining ice buckets. The
hotelier sues the company for failing to deliver
A claim based on which of the following is all 250 ice buckets.
the best basis for recovery by the strikers?
Will the hotelier prevail?
(A) Battery.
(A) No, because the hotelier is not a hotel sup-
(B) Abnormally dangerous activity. ply merchant.
(C) Negligence. (B) No, because the company’s offer was to
remain open for more than three months.
(D) Strict liability in tort.
(C) Yes, because the company promised in a
signed writing to hold the offer open.

(D) Yes, because the hotelier never read the


company’s July 2 fax.

GO ON TO THE NEXT PAGE

MPQ 100 preview diagnostic exam and divider P.indd 18 12/22/2015 4:13:30 PM
MBE PREVIEW DIAGNOSTIC EXAM 19.

Question 29 Question 30

A bathroom fixture company, in a signed The neighbors to the north and south of a
writing, contracted with the developer of an poorly maintained house, a mechanic and a
apartment building for the sale to the building of doctor, decided to “send a message” by placing
50 identical sets of specified bathroom fixtures, a smoke bomb on the owner’s front porch. The
25 sets to be delivered on March 1, and the doctor and mechanic consulted with another
remaining 25 sets on April 1. The agreement did neighbor, an army veteran who was known
not specify the place of delivery, nor the time to be an explosives expert, to help them. The
or place of payment. On March 1, the fixture army veteran agreed, but unbeknownst to the
company tendered 24 sets to the developer and two neighbors, the army veteran manufactured
explained that one of the 25 sets was damaged a bomb containing a lethal quantity of explo-
in transit from the manufacturer to the company. sives because he wanted to make sure the owner
The fixture company also promised to deliver a had a good scare. He gave the bomb to the
replacement within five days. mechanic and doctor, along with a timed fuse.
In the middle of the night, the mechanic set
Which of the following statements is correct? the bomb on the owner’s porch. A few minutes
later, a huge explosion rocked the neighborhood.
(A) The developer is entitled to accept any Although nobody was injured, the blast blew
number of the 24 sets, reject the rest, and away the front of the owner’s house.
cancel the contract both as to any rejected
sets and the lot due on April 1. Who, of the following, is likely to be found
guilty of attempted murder?
(B) The developer is entitled to accept any
number of the 24 sets and to reject the rest, (A) The doctor, mechanic, and the army vet-
but is not entitled to cancel the contract as eran.
to any rejected sets or the lot due on April
1. (B) The army veteran and the mechanic, but not
the doctor.
(C) The developer must accept the 24 sets but is
entitled to cancel the rest of the contract. (C) The army veteran, but not the doctor or the
mechanic.
(D) The developer must accept the 24 sets
and is not entitled to cancel the rest of the (D) Neither the mechanic nor the doctor nor the
contract. army veteran.

GO ON TO THE NEXT PAGE

MPQ 100 preview diagnostic exam and divider P.indd 19 12/22/2015 4:13:30 PM
20. MBE PREVIEW DIAGNOSTIC EXAM

Question 31 Question 32

A defendant was charged with murdering his While driving in State A, the defendant, a
cousin, whose body was found near the defen- State B resident, was in an automobile accident
dant’s office. The prosecution alleged that the with the plaintiff, a resident of State A. The
cousin, who lived in another state, confronted plaintiff filed a negligence action against the
the defendant to tell him about an affair that defendant in a State A state court consistent with
the cousin was having with the defendant’s State A’s long arm statute, and properly served
wife. The defendant denied having even seen the State B defendant pursuant to the State A
the cousin for several weeks before he was long arm statute. The State B defendant immedi-
murdered. The prosecution wishes to introduce ately filed a motion to dismiss the action on the
a properly authenticated letter from the cousin grounds that the State A court does not have
to the defendant’s wife, dated the day before the personal jurisdiction.
cousin’s death, that stated: “I am going to go to
your husband’s office and tell him face-to-face How should the court rule on the motion to
that he must let us be together.” Defense counsel dismiss?
objects.
(A) The court should deny the motion, because
How should the judge rule on the objection? the defendant has purposeful contacts with
State A that are directly related to the claim
(A) Overruled, because the statement in the being asserted.
letter falls under the present state of mind
exception to the hearsay rule. (B) The court should deny the motion because
the plaintiff is a State A resident.
(B) Overruled, because the statement in the
letter is being offered to show the defen- (C) The court should grant the motion unless
dant’s motive for murder rather than for the the defendant is engaged in substantial and
truth of the matter asserted. continuous activities such that it is essen-
tially at home in State A.
(C) Granted, because the statement in the letter
is hearsay and does not fall within any (D) The court should grant the motion because
exception. State A courts lack constitutional authority
to assert jurisdiction over defendants
(D) Granted, because the cousin’s state of mind outside of State A unless such defendants
is not in issue. consent to the State A courts’ jurisdiction.

GO ON TO THE NEXT PAGE

MPQ 100 preview diagnostic exam and divider P.indd 20 12/22/2015 4:13:30 PM
MBE PREVIEW DIAGNOSTIC EXAM 21.

Question 33 Question 34

A landowner conveyed his parcel of land “to The plaintiff in a negligence action in federal
my sister and her heirs so long as it is used for district court was injured when the defendant’s
residential purposes, but if it is ever used for car crashed into his. The defendant’s friend was
other than residential purposes, then to the local a passenger in the defendant’s vehicle at the time
community center.” of the accident. The defendant’s attorney sent
an email to the passenger asking the passenger
Five years later, the landowner died, devising to describe in detail what the passenger remem-
all of his real estate to his friend and leaving his bered about the accident and the events leading
daughter as his only heir. up to it. In response, the passenger sent the
defendant’s attorney an email describing the
The following year, the landowner’s sister and events. The plaintiff served on the defendant
the landowner’s daughter entered into a contract the following request for documents: “Please
with a third party to sell the parcel to him in fee produce for inspection or copying any and all
simple for $100,000. After examining title, the statements obtained by you or your attorney that
third party refused to perform under the contract relate in any way to the events and/or issues that
because he believed the sister and the daughter are the subject of this legal action.”
could not deliver good title.
Must the defendant produce the passenger’s
The jurisdiction follows the common law Rule statement?
Against Perpetuities and has a statute providing
that all future estates and interests are alienable, (A) Yes, because the statement is relevant to the
descendible, and devisable in the same manner parties’ claims and defenses.
as possessory estates and interests.
(B) Yes, because the work product doctrine
If the sister and the daughter sue the third does not apply to emails and other digitally
party for specific performance, how should the stored information.
court rule on their request?
(C) No, because the statement is protected from
(A) Grant it, because the sister owns the parcel discovery pursuant to the work product
in fee simple. doctrine.

(B) Grant it, because the sister and the daughter (D) No, because the statement is privileged.
together own the parcel in fee simple.

(C) Deny it, because the local community


center has a valid interest in the parcel.

(D) Deny it, because the friend has a valid


interest in the parcel.

GO ON TO THE NEXT PAGE

MPQ 100 preview diagnostic exam and divider P.indd 21 12/22/2015 4:13:30 PM
22. MBE PREVIEW DIAGNOSTIC EXAM

Question 35 Question 36

The police suspected that an 18-year-old An intoxicated airline passenger collecting


living with his mother was selling marijuana to his bags after passing through security grabbed
his high school classmates. The police went to a garment bag from its owner because he
the house without a search warrant and secured mistakenly thought it was his own. The owner
the mother’s permission to search the teen’s of the garment bag tried to take it back from
bedroom. In the bedroom, they found a locked the passenger. During the tug of war over the
suitcase under the teen’s bed. The mother said garment bag, the passenger knocked the owner
that it was her son’s suitcase but that they could to the floor, took the garment bag, and ran to
search it. The police then broke the lock, opened his gate. The passenger was later arrested and
it and found a substantial amount of marijuana. charged with robbery.
The police charged the teen with possession of
marijuana with intent to distribute it. Should the passenger be found guilty?

If the teen should bring a pretrial motion to (A) Yes, because his intoxication was volun-
suppress the marijuana, how should the court tary.
rule on the motion?
(B) Yes, because mistake is no defense to
(A) Grant it, because the mother was not in- robbery.
formed of her right to refuse consent to the
search. (C) No, because he made no threats and was
intoxicated.
(B) Grant it, because the mother’s consent to
search the suitcase was invalid. (D) No, because his mistake negated the
required specific intent.
(C) Deny it, because the mother had authority
to consent to a search of anything within
her home.

(D) Deny it, under the doctrine of in loco


parentis.

GO ON TO THE NEXT PAGE

MPQ 100 preview diagnostic exam and divider P.indd 22 12/22/2015 4:13:30 PM
MBE PREVIEW DIAGNOSTIC EXAM 23.

Question 37 Question 38

A gardener entered into a contract with a A writer registered under federal copyright
landowner under which, for a monthly fee of law his copyright in certain song lyrics he wrote.
$200, the gardener would provide the landowner The writer later entered into a contract with an
with monthly gardening services for the calendar advertiser in which the writer granted the adver-
year. On June 1, the gardener told the landowner tiser a license to use the lyrics in radio advertise-
that she could no longer afford to work for $200 ments. When the writer heard the advertisement
a month and demanded that she be paid $400 a using the lyrics, the writer was incensed at how
month. The landowner refused, stating that he the lyrics had been used. Believing that the
already had a valid contract with her and that advertiser had lied to him about how the lyrics
he did not have to pay any more. The gardener would be used, the writer filed an action in
again insisted that she would not work for less federal district court claiming that the advertiser
than $400. As a result, the gardener and the had made false representations that fraudulently
landowner agreed in writing on July 1 that the induced the writer into entering the contract to
gardener would thereafter be paid $400 a month license the lyrics. The writer is a citizen of State
for the gardening services and that the term of A. The advertiser is a partnership comprised of
the agreement would be extended through the partners who are citizens of State A, State B,
end of next year. and State C. The partnership’s headquarters and
most of its operations are in State B.
Is the gardener legally entitled to receive a
monthly fee of $400 from the landowner after Does the federal court have subject matter
July 1 of this year? jurisdiction over the action?

(A) Yes, but only for the period from January 1 (A) No, because the action does not arise under
to December 31 of next year. federal law and the parties are citizens of
the same state.
(B) Yes, because the gardener and the
landowner mutually agreed to modify their (B) Yes, because the plaintiff and defendant are
original contract. citizens of different states.

(C) No, because the gardener used economic (C) Yes, because the action arises under federal
duress in order to obtain the landowner’s law.
agreement to the modification.
(D) Yes, because the transaction involves inter-
(D) No, because the gardener was under a state commerce.
preexisting duty to perform the gardening
services for $200 a month.

GO ON TO THE NEXT PAGE

MPQ 100 preview diagnostic exam and divider P.indd 23 12/22/2015 4:13:30 PM
24. MBE PREVIEW DIAGNOSTIC EXAM

Question 39 Question 40

A tire company is the manufacturer of, and A pregnant woman visited her family doctor
owns the patent for, a new and unique type of to seek relief from arthritis in her hips. The
truck tire. The company is the largest employer doctor prescribed a steroid-type drug. The
in the state in which it is located. That state pregnant woman took the drug as prescribed
enacted a law requiring all trucks using its and obtained relief from her pain. The pharma-
highways to be equipped with the company’s ceutical company which manufactured the drug
unique truck tires. An out-of-state trucking enclosed information concerning the drug in
company filed suit against the state, and eventu- the packages it shipped to drugstores and with
ally the United States Supreme Court held samples provided to physicians. This literature
that the state’s statute violated the Commerce warned that the drug should not be prescribed
Clause. Subsequently, Congress enacted a law for pregnant women because it caused enlarge-
that required all trucks engaged in interstate ment of the ears of male children. The enlarge-
commerce to use the company’s unique tires. ment occurred when the children reach puberty.
Studies confirming this result were published in
Is this federal law constitutional? various medical journals. The pregnant woman
gave birth to a son soon thereafter. Although the
(A) Yes, because Congress’s control over inter- son had a normal early childhood, his ears grew
state commerce is very broad. disproportionately large when he reached age
16, and he has suffered severe emotional distress
(B) Yes, only if Congress can demonstrate that because of his appearance. Now 17, the son has
the use of tires of another design somehow brought suit against the doctor for malpractice.
unduly burdens interstate commerce.
In that lawsuit, which party is likely to
(C) No, if most states permit tires of another prevail?
design.
(A) The son will prevail if he can prove that
(D) No, because the federal law reverses a most doctors would not have prescribed the
decision of the Supreme Court, and the drug for his mother.
Supreme Court is the sole arbiter of consti-
tutional issues. (B) The son will prevail unless the doctor
establishes that he had not read the journals
reporting on the side effects of the drug.

(C) The doctor will prevail because he quali-


fies as a “learned intermediary” with regard
to the warning about the drug from the
pharmaceutical company.

(D) The doctor will prevail because no duty is


owed to an unborn child.

GO ON TO THE NEXT PAGE

MPQ 100 preview diagnostic exam and divider P.indd 24 12/22/2015 4:13:30 PM
MBE PREVIEW DIAGNOSTIC EXAM 25.

Question 41 Question 42

Several years ago a number of employees A large oil producer wanted to expand into
of the state transportation department formed the oil refinery business. It contacted a small
a workers’ organization, a voluntary associa- oil refinery company and offered to purchase
tion incorporated under the laws of the state. the company’s refining facilities as well as its
The organization has grown in size over the output of petroleum products from its remaining
years and has become very powerful politically. production facilities.
Nearly 85% of all state highway maintenance
workers are members of the organization, which Lawyers for the oil producer prepared a
is supported entirely by the dues of its members. written agreement specifying the sale price for
Membership is not required as a condition of the smaller company’s refining business. The
employment or advancement in the state trans- agreement also included provisions permitting
portation department. After a vote of its entire the oil producer, at its option, to purchase 100%
membership on the question, a member was of all petroleum products removed from the
expelled from the organization solely because earth by the smaller company for the following
she is an active, knowing member of a group that 15 years, at current market prices. The agree-
seeks to propagate the concept of white racial ment was signed by both parties. One month
supremacy. The expelled member brings suit in after the sale of the refining facilities, the oil
an appropriate court seeking reinstatement of her producer sent a fax to the smaller company
membership. The suit alleges that her expulsion stating that it would be exercising its option to
violates rights guaranteed to her by the First and purchase the smaller company’s oil output for
Fourteenth Amendments. the next 15 years.

Which of the following is the strongest consti- Is the written agreement between the oil
tutional defense that the organization may assert producer and the smaller company enforceable?
to this suit?
(A) No, because the provision permitting the
(A) The First and Fourteenth Amendments do oil producer to purchase all of the smaller
not apply to the actions of the organization. company’s output was not supported by
any consideration.
(B) Active, knowing membership in groups
with overtly racist objectives is not (B) No, because the provision permitting the
protected by the First and Fourteenth oil producer to purchase all of the smaller
Amendments. company’s output for 15 years was indefi-
nite and uncertain.
(C) The organization could reasonably believe
that it has a compelling interest in elimi- (C) Yes, as to the sale of the smaller company’s
nating from its membership active, knowing refining facilities, but unenforceable as to
members of groups with overtly racist the output provision.
objectives.
(D) Yes, in its entirety.
(D) The expulsion of the member took place
only after a vote of the organization’s entire
membership.

GO ON TO THE NEXT PAGE

MPQ 100 preview diagnostic exam and divider P.indd 25 12/22/2015 4:13:30 PM
26. MBE PREVIEW DIAGNOSTIC EXAM

Question 43 Question 44

A successful stockbroker and his best friend, A tool producer was in the business of
a land developer, were pleased that their only manufacturing and selling various hardware.
children were about to marry each other. The The owner of a retail hardware store ordered
stockbroker and the developer wanted to get the 1,000 hammers from the tool producer, using
marriage off to a good start. Thus, on July 1, the standard form that the tool producer’s sales
the two agreed in writing that, on July 15, the personnel used. Shortly after the hardware
stockbroker would deliver to the land developer store placed their order, the tool producer’s
a check for one-half of the market value of a headquarters caught fire. All the purchase and
two-bedroom condominium owned by the devel- sale records were destroyed in the fire. After-
oper, and the developer would simultaneously wards, the hardware store owner asserted that
deed the condominium to the bride and groom. the salesperson had agreed that the price for the
hammers would be $5,000. The tool producer
On July 10, the newlyweds learned of the maintained that the price was $5,500. The tool
agreement but neither assented to it nor changed producer then sued the hardware store. At the
their position in reliance on it. On July 12, the trial, the tool producer’s director of the order
stockbroker and the developer mutually agreed department took the stand. After establishing
to cancel their agreement. that the original order had been destroyed by
fire, the director sought to testify that the price
If the newlyweds were to sue the developer of the order was $5,500.
and the stockbroker for breach of contract, would
they prevail? How should the court rule on the admissibility
of such testimony?
(A) Yes, because their rights as intended ben-
eficiaries vested when the contract was (A) Inadmissible, because it is self-serving and
made on July 1 and a later revision was untrustworthy.
ineffective in affecting their rights.
(B) Inadmissible, because of the best evidence
(B) Yes, because they received notice of the rule.
contract on July 10, and the rescission
thereafter was ineffective. (C) Inadmissible, because it is not the best
degree of secondary evidence.
(C) No, because the stockbroker and developer
had the right to mutually rescind their (D) Admissible.
agreement.

(D) No, because the newlyweds were incidental


beneficiaries and had no right to enforce the
contract.

GO ON TO THE NEXT PAGE

MPQ 100 preview diagnostic exam and divider P.indd 26 12/22/2015 4:13:30 PM
MBE PREVIEW DIAGNOSTIC EXAM 27.

Question 45 Question 46

A plaintiff sued a defendant in federal district While walking down the street, a pedestrian
court for breach of contract. Based on a prelimi- accidentally bumped into a street performer.
nary investigation, the defendant’s attorney The performer told the pedestrian to pay atten-
believes that no legally enforceable contract tion to where she was going. The pedestrian got
ever existed. Unfortunately for the defendant, mad and slapped the performer across the face.
however, his attorney is unsure whether she can The performer, afraid that the pedestrian would
prove that point at trial. If a valid contract did continue the assault and escalate it to the point
exist, the defendant’s attorney believes that the of murdering him, pulled out a gun and shot the
defendant did not breach it. She also thinks that pedestrian, killing her instantly.
she has a better chance of prevailing on that
point. In what type of state is the performer most
likely to be convicted of manslaughter?
Which of the following is true regarding the
defendant’s ability to assert as defenses both that (A) A state that recognizes the heat of passion
no contract existed and that the defendant did resulting from adequate provocation as a
not breach the contract if one did exist? mitigating circumstance.

(A) The defendant may plead only one of these (B) A state that applies the doctrine of dimin-
defenses because they are inconsistent. ished responsibility.

(B) The defendant may plead both defenses, but (C) A state that recognizes the misdemeanor-
the plaintiff will then be able to have one manslaughter rule.
defense stricken.
(D) A state that recognizes the imperfect self-
(C) The defendant may plead both defenses, defense doctrine.
as long as he labels them as affirmative
defenses.

(D) The defendant may plead both defenses


regardless of how they are labeled.

GO ON TO THE NEXT PAGE

MPQ 100 preview diagnostic exam and divider P.indd 27 12/22/2015 4:13:30 PM
28. MBE PREVIEW DIAGNOSTIC EXAM

Question 47 Question 48

An owner of 40 acres of mountain land sold Twenty-five years ago, a property owner
the western 20 acres to a buyer. Because no placed a large sewer line (to service a single-
access to any public road existed on the western family house he built on the property) across
side of the property, the deed conveying title a neighbor’s property without the neighbor’s
to the buyer included an easement for ingress permission. Four years ago, the owner tore down
and egress that ran along the southern border of the house in preparation for the construction of a
the owner’s land. This deed was duly recorded. larger house on the land, and made an agreement
About the same time that this sale took place, with municipal authorities to take an easement
the county extended the public road so that it across the neighbor’s property and install a
abutted on the buyer’s 20 acres. The buyer then new sewer line to service the house. After a
built a cabin on the property. long delay, the municipal authorities failed to
perform their agreement, and the owner finally
Later, the owner and the buyer both sold their constructed the house last year using the existing
20 acres to the owner’s cousin. The cousin then sewer line. The state has a 20-year statute for
sold the 20 acres acquired from the buyer to his acquiring property interests by adverse use.
lawyer and sold the other 20 acres to a doctor.
All deeds involved in the various conveyances If the neighbor attempts to enjoin the use of
of the 20-acre parcels were validly recorded. the sewer line to service the new house, will she
Neither the cousin-lawyer deed nor the cousin- prevail?
doctor deed made any mention of easements or
rights-of-way. In fact, no use was ever made of (A) Yes, because the agreement with the mu-
the easement. nicipal authorities estopped the owner from
using the existing line to service the new
A few years later, the lawyer sold his 20 acres house.
to a development company that wished to build a
hunting lodge on the property. The development (B) Yes, because the owner’s tearing down of
company now wants to construct a road across the house and nonuse of the sewer line for
the doctor’s property in the manner contemplated several years constituted an abandonment
in the deed from the original owner to the buyer. of the easement.

What is the strongest argument as to why the (C) No, because the original sewer line was
development company should not be permitted a license which ripened into an implied
to construct the road? easement.

(A) Any easement that might have existed was (D) No, because the use was within the scope of
extinguished by abandonment because of the prescriptive easement acquired.
nonuse.

(B) Any easement that once existed was termi-


nated by merger.

(C) The county’s subsequent expansion of the


public road removed the necessity of using
the easement across the eastern parcel.

(D) Neither the cousin-doctor deed nor the


cousin-lawyer deed made any mention of
the easement.

GO ON TO THE NEXT PAGE

MPQ 100 preview diagnostic exam and divider P.indd 28 12/22/2015 4:13:30 PM
MBE PREVIEW DIAGNOSTIC EXAM 29.

Question 49 Question 50

On January 1, state legislation was enacted The police set up an undercover “sting” opera-
that prohibited private employers, under penalty tion in which they posed as fences of stolen
of civil fines, from hiring any person who had property and bought and sold such property
not been a resident of that state for 10 years. to anyone who came into their downtown
The legislation also required private employers warehouse. A defendant is being prosecuted for
to immediately discharge any employee who receiving stolen property in connection with
had not resided in the state for a minimum of 10 his arrest by the undercover operatives, and the
years. The new statutes additionally provided prosecution attempts to introduce a videotape
that the applicable civil fines would be assessed showing the defendant offering to sell a televi-
by the state department of employment inspec- sion set to one of the police officers.
tors, at their sole discretion, any time they
believed that an employer had hired or failed to If this evidence is held to be inadmissible,
discharge a person not meeting the residency what is the most likely reason?
requirements, and that the civil fines would
be imposed on any employer who had hired (A) It is hearsay not within any exception.
or failed to discharge such persons on or after
January 1. An employee within the state, who is (B) It violates the defendant’s privilege against
a nonresident, brought suit challenging the state self-incrimination.
statute.
(C) A proper foundation was not established for
Which of the following arguments would its introduction into evidence.
most likely result in a favorable ruling for the
challenger? (D) Criminality may not be proven by specific
instances of misconduct.
(A) The statute is an unconstitutional Bill of
Attainder.

(B) The statute violates the Privileges and


Immunities Clause of Article IV.

(C) The statute violates the Equal Protection


Clause of the Fourteenth Amendment.

(D) The statute violates the Ex Post Facto


Clause.

GO ON TO THE NEXT PAGE

MPQ 100 preview diagnostic exam and divider P.indd 29 12/22/2015 4:13:30 PM
30. MBE PREVIEW DIAGNOSTIC EXAM

Question 51 Question 52

An employee of a construction company A husband purchased a cabin as his separate


working in a high crime neighborhood negli- property. The husband provided in his will that
gently cut through power cables. The accident the cabin be devised “to my sister-in-law for so
knocked out power to the area and disabled long as my wife is alive, remainder to my son.”
a home’s security system. The homeowner, Shortly thereafter, the husband died. When the
who was out of town, received an alert from sister-in-law learned of her interest in the cabin,
the security company advising her to arrange she decided to lease the property. She entered
for someone to go to the house and restart into a valid written lease for a term of three
the system with a backup power source. The years with a tenant, who agreed to pay $3,000
homeowner got distracted soon after getting the per year rent, with $1,500 due in advance every
message and neglected to contact anyone. That six months. The tenant paid the first $1,500
evening, a burglar broke into the home while and took possession of the property. Two years
power was still off in the neighborhood. Because later, one week after the tenant had paid the
the security system was not working and the $1,500 due for the ensuing six-month period,
alarm did not go off, the burglar was able to the wife died. The son demanded that the tenant
steal jewelry and other valuables from the home surrender possession of the cabin, but the tenant
and escape. The homeowner sued the construc- refused.
tion company for the loss of her valuables in a
jurisdiction that has adopted pure comparative In an appropriate action for possession of the
negligence rules. disputed property, is the son likely to prevail?

Is the homeowner likely to recover? (A) Yes, because his rights are superior to the
tenant’s.
(A) Yes, because the homeowner’s negligence
contributed the least to her loss. (B) Yes, if he reimburses the tenant for the
portion of the six months’ rent already paid
(B) Yes, because the conduct of the construc- to the sister-in-law that is attributable to the
tion company’s employee created the oppor- period after the tenant surrenders posses-
tunity for the burglar to steal the valuables. sion.

(C) No, because the homeowner’s negligence (C) No, because he did not wait to bring the
was a superseding cause of her loss. action until the period for which the tenant
had already paid rent had elapsed.
(D) No, because the burglar committed a
criminal act that was a superseding cause (D) No, because the tenant has the right to
of the loss. remain in possession for the last year of his
lease as long as he pays the final $1,500.

GO ON TO THE NEXT PAGE

MPQ 100 preview diagnostic exam and divider P.indd 30 12/22/2015 4:13:30 PM
MBE PREVIEW DIAGNOSTIC EXAM 31.

Question 53 Question 54

A motorcyclist had just bought a new motor- After the failure of a state bill granting
cycle. One week later, while still learning how gay and lesbian couples the right to marry,
to ride the motorcycle, he saw a classmate from 30 students from a local university marched
school whom he did not like walking along the on the state capitol to protest, carrying signs
sidewalk. He tried to scare him by swerving with slogans such as “let gays marry” and “no
onto the sidewalk at a driveway, planning to religious tyranny.” As they marched, about 15
swerve back onto the street at the next driveway people began following them, shouting anti-gay
just in front of the classmate. As the motorcy- remarks. At the capitol building, they were met
clist attempted to swerve back onto the street, by 50 officers in full riot gear. The leader of
the motorcycle’s front tire blew because of a the students addressed his followers in the park
latent defect, causing him to lose control of the across the street, vigorously denouncing the
steering. He attempted to apply the brakes, but legislature’s actions, which caused the counter-
due to his inexperience, hit the accelerator by protestors to become more vocal. The officer
mistake. The motorcycle struck and seriously in charge told the leader that he must end his
injured the classmate. The classmate sues the speech because a riot was about to start. The
motorcyclist for battery. leader refused and was arrested and convicted of
disorderly conduct.
Who is likely to prevail?
If the leader appeals his conviction on
(A) The classmate, because the motorcyclist constitutional grounds, will the conviction be
intended to frighten the classmate. reversed?

(B) The classmate, unless the motorcyclist’s (A) Yes, because the leader’s arrest constituted
negligence in hitting the accelerator was the an interference with his First Amendment
proximate cause of the accident. right to free speech.

(C) The motorcyclist, because he did not intend (B) Yes, because the leader’s arrest constituted
to inflict bodily harm on the classmate. undue interference with the students’ right
to peaceably assemble under the First
(D) The motorcyclist, because the injury was Amendment.
proximately caused by the defective front
tire. (C) No, because the leader’s speech caused an
immediate and substantial threat to public
order.

(D) No, because the leader had a duty to obey


the police officer since other avenues of
communication likely were available.

GO ON TO THE NEXT PAGE

MPQ 100 preview diagnostic exam and divider P.indd 31 12/22/2015 4:13:30 PM
32. MBE PREVIEW DIAGNOSTIC EXAM

Question 55 Question 56

A mechanic noticed that his neighbor had An owner of land who was also a home
a “for sale” sign on his old car. The mechanic contractor agreed in writing with a buyer to
thought it would be fun to have an old car to fix build a house to the buyer’s specifications on his
up and customize, and he asked his neighbor (the contractor’s) land and then sell the house
how much he wanted for the car. The neighbor and lot to the buyer. The contract provided that
told him that he would sell the car for $400, and the house was to be completed by March 1, with
the mechanic responded that he would buy the full payment due at that time. On March 1, the
car for $400. They agreed that the mechanic house was nearly complete, but due to delays in
would come to the neighbor’s house by 6 p.m. the delivery of materials, the contractor would
the next day with the money. At 9:15 the next need 20 more days to finish construction. On
morning, the neighbor called the mechanic and March 5, after discovering that the house had
told him that when he agreed to the sale the not yet been completed, the buyer notified the
preceding day, he forgot that he had just put two contractor in writing of her election to cancel
new tires on the car and, therefore, would need the contract because of the contractor’s failure
to be paid an extra $50. The mechanic agreed to to deliver the house by March 1. The contractor
bring $450 in cash to his neighbor’s house at six responded that, due to an unanticipated strike at
o’clock that night. his supplier’s company, performance had been
unforeseeably delayed and that the house would
Is the mechanic legally bound to pay his be ready by March 20. The buyer responded that
neighbor the additional $50? she would no longer accept delivery of the house
and land. The contractor then brought an action
(A) Yes, because the original contract was not to recover damages for breach of contract.
in writing.
Who will prevail?
(B) Yes, because the contract, as modified, does
not exceed the minimum dollar amount (A) The buyer, because the express date listed
required to invoke the Statute of Frauds. in the contract indicates that time was of
the essence.
(C) No, because no additional consideration
was given for the oral modification. (B) The buyer, because delivery by March 1
was a condition precedent to the buyer’s
(D) No, because contracts for the sale of goods performance.
can only be modified by merchants.
(C) The contractor, because the strike was truly
an unforeseeable intervening event.

(D) The contractor, because the buyer received


the substantial benefit of the bargain.

GO ON TO THE NEXT PAGE

MPQ 100 preview diagnostic exam and divider P.indd 32 12/22/2015 4:13:30 PM
MBE PREVIEW DIAGNOSTIC EXAM 33.

Question 57 Question 58

A hunter drove to one of his old spots for A shopper was buying groceries and
target shooting. When he arrived at the spot, he attempted to pay with a personal check. The
noticed that an adjacent area that had formerly grocery clerk thought that she recognized the
been an open field now contained a new housing shopper’s name from a store memo listing
tract with a playground where children were known habitual bad check writers in the area.
playing. The hunter tacked a paper target to a The clerk immediately called for the store’s
tree that was located between the playground security guard over the store’s loudspeaker.
and the point from which he planned to shoot. The guard quickly approached the shopper
He fired a number of shots at the target, and all and requested that she step into his office. The
but one hit the target. The one shot that missed shopper protested her innocence but accom-
that went wide of the target, ricocheted off a panied the guard. He told her not to leave or
tree, and flew into the playground, striking a he would call the police. She was then kept
child in the head, killing the child. waiting in the guard’s office for an hour until the
guard finally returned with a copy of the memo
If the hunter is tried for the death of the child, showing that the shopper’s name was not on
what is the most serious crime of which he can the list. The shopper was terribly embarrassed
be convicted? by the entire incident because a number of her
neighbors had seen the guard take her away.
(A) Murder, because the hunter’s intent to
shoot the child can be inferred from his If the shopper sues the grocery store for
conduct. damages for her humiliation, will she prevail?

(B) Murder, because the hunter acted with (A) Yes, because she was falsely imprisoned.
wanton recklessness when he chose to shoot
the gun in an area near the playground. (B) Yes, because the grocery store was negli-
gent in identifying her.
(C) Manslaughter, because the hunter did not
intend to kill the child. (C) No, because humiliation is not actionable.

(D) Neither murder nor manslaughter, because (D) No, unless the grocery store’s conduct is
the hunter’s shooting was negligent, and judged to be extreme and outrageous.
mere negligence is insufficient to support
conviction for a homicide crime.

GO ON TO THE NEXT PAGE

MPQ 100 preview diagnostic exam and divider P.indd 33 12/22/2015 4:13:31 PM
34. MBE PREVIEW DIAGNOSTIC EXAM

Question 59 Question 60

While driving a new car he recently Late one night, a young couple were killed
purchased from an authorized dealer, a consum- instantly when their car was struck by a speeding
er’s car caught fire for an unknown reason. The truck as the couple’s car crossed an intersection
fire not only damaged the car, but also injured with the light green in their favor. Several weeks
the consumer. The consumer filed a products later, a burglar awaiting trial on burglary charges
liability action against the manufacturer of the asked a jail officer to let him speak with a
car in federal district court, seeking to recover highway patrol officer. When the highway patrol
compensatory damages for his injuries. The officer came to the cell, the burglar told him that
complaint alleged that parts of the electrical he was the driver of the truck that had struck the
system in the car were defective and that the car and had been speeding away from a burglary
defects caused the fire. The manufacturer filed when the accident occurred.
an answer that denied the existence of any
defects and denied that any defects caused the The burglar was charged with felony murder,
fire, but stated that it lacked sufficient knowl- on the theory that he had not yet reached a place
edge and information to know what caused the of temporary safety when the accident occurred.
fire. During discovery, the consumer served an At trial, the prosecution seeks to introduce the
interrogatory on the manufacturer that asked the burglar’s statements to the highway patrol officer
manufacturer to “identify and summarize all regarding the events of the night of the accident.
evidence of which the manufacturer [was] aware The burglar’s attorney objects.
that indicated that the fire was not caused by a
defect in the car.” The manufacturer’s response Which of the following is the strongest
stated that it did not have, and was not aware argument for permitting the statements into
of, any evidence indicating that the fire was not evidence?
caused by a defect. Based on that interrogatory
response, the consumer filed a motion for partial (A) The burglar had not been charged in con-
summary judgment on the issue of causation to nection with the auto accident at the time
establish that any fire was caused by defects. the statements were made to the highway
patrol officer.
How should the court rule on the motion?
(B) The burglar made the statements spontane-
(A) Grant the consumer’s motion for summary ously, without inducement or interrogation
judgment. by the police.

(B) Grant the consumer’s motion for summary (C) The highway patrol officer had no connec-
judgment unless the manufacturer files tion with the burglary investigation for
affidavits or other evidence indicating that which the burglar had been incarcerated.
the fire was not caused by a defect.
(D) The burglar’s statements were not the
(C) The court should deny the consumer’s product of coercion by the police officers.
motion, because it addresses an ultimate
issue in the case.

(D) The court should deny the consumer’s


motion, because, unless the consumer has
presented evidence that a defect caused
the fire, the manufacturer does not need to
present evidence regarding causation.

GO ON TO THE NEXT PAGE

MPQ 100 preview diagnostic exam and divider P.indd 34 12/22/2015 4:13:31 PM
MBE PREVIEW DIAGNOSTIC EXAM 35.

Question 61 Question 62

A homeowner leased his home to a tenant for A biker sued a driver for injuries allegedly
three years. The following year, the homeowner sustained in an automobile accident. The driver’s
conveyed the house to a buyer, who never defense was that the biker was the cause of
recorded her deed nor did anything with regard the accident, because the biker ran a red light
to the house. The tenant continued paying rent to and crashed into the driver’s car. The driver’s
the homeowner. Three months after the convey- principal witness testified that he was standing
ance to the buyer, the homeowner conveyed the on the corner when the accident occurred, and
property to his proctologist, who knew nothing he observed the biker run the red light and crash
of the prior conveyance to the buyer. The into the driver’s car. On cross-examination of
homeowner took the proctologist’s money and the witness, the biker’s lawyer asked, “Isn’t it
skipped town. The proctologist told the tenant true that on the day of the accident you were 300
that he now owned the house and that all rents miles away from the scene attempting to extract
should be paid to him. The tenant complied. protection money from the owner of a seaside
restaurant?” The driver’s attorney objected to
Six months later, the proctologist went to his the question, but the objection was overruled by
local bank for a loan. He offered to put up the the court. The witness then refused to answer
property as security. The bank discovered that the question, invoking the privilege against self-
the proctologist had never recorded his deed and incrimination.
that, just two weeks prior to his loan application,
the buyer had recorded a deed to the house that The biker’s attorney moved to have the
bore an earlier date than the deed the proctolo- witness’s direct testimony stricken from the
gist had shown the bank. Because of this cloud record.
on the title, the bank refused the loan request.
When the tenant discovered this, she quit paying How should the court rule on the motion?
rent to the proctologist. The state has a recording
statute that provides, “a conveyance of an (A) Grant the motion, because the privilege
interest in land, other than a lease for less than against self-incrimination is not applicable
one year, shall not be valid against any subse- to civil cases.
quent purchaser for value, without notice thereof,
unless the conveyance is recorded.” (B) Grant the motion, because the biker has
not had an adequate opportunity for
If the proctologist sues the tenant to compel the meaningful cross-examination.
payment of rent, is the proctologist likely to win?
(C) Deny the motion, because the witness has a
(A) Yes, because the tenant is estopped from
right to refuse to answer.
denying a landlord-tenant relationship with
the proctologist, since she had paid rent for (D) Deny the motion, because the jury can
many months. assess the weight to be given to the
(B) Yes, because the proctologist was a bona witness’s testimony.
fide purchaser when he bought the property
from the homeowner.
(C) No, because the proctologist failed to record
his deed to the property.
(D) No, because the proctologist did not have
good title to the property and cannot
demand rent from tenants in possession.

GO ON TO THE NEXT PAGE

MPQ 100 preview diagnostic exam and divider P.indd 35 12/22/2015 4:13:31 PM
36. MBE PREVIEW DIAGNOSTIC EXAM

Question 63 Question 64

At 3 a.m. on the state turnpike, a driver was Two police officers stopped an automobile
stopped for driving 30 m.p.h. over the posted for improperly proceeding through a red light.
speed limit. Because of the speed at which she When one of the officers approached the car,
was traveling and because the driver possessed he observed some hand-rolled cigarettes on the
an out-of-state driver’s license, the officer dashboard. He ordered the driver out of the car,
decided to place the driver under arrest and take examined the cigarettes, and determined that
her to the station house. Under state law, such they were joints of marijuana. The officer then
an arrest is valid. After placing the driver in his arrested the driver and guarded him in the back
patrol car, the officer searched the passenger seat of the police cruiser while the second officer
area of the automobile. Under the front seat searched the entire car, including the trunk. In
he found a small package containing what he the trunk, he found two rare paintings that had
immediately determined to be marijuana. recently been stolen from the city’s art museum.

If the driver is charged with possession of The driver was charged with possession of
marijuana, how should the court rule on her stolen goods and brought a motion to suppress
motion to suppress the marijuana? the introduction of the paintings into evidence.

(A) Grant it, unless the officer had probable If the driver’s motion is denied, it will be
cause to believe the car contained items because the second officer conducted which type
that were legitimate to seize. of search properly?

(B) Grant it, because the search was of an area (A) Search incident to an arrest.
outside the control of the arrestee.
(B) Automobile search.
(C) Deny it, because the search was incident to
a valid arrest. (C) Custody search.

(D) Deny it, if the court determines the officer (D) Inventory search.
had a reasonable fear of the driver.

GO ON TO THE NEXT PAGE

MPQ 100 preview diagnostic exam and divider P.indd 36 12/22/2015 4:13:31 PM
MBE PREVIEW DIAGNOSTIC EXAM 37.

Question 65 Question 66

Under a program created by Congress, the A debt that a brother owed to his sister had an
federal government allocated $5 million to outstanding balance of $4,000 when the statute
one of the nation’s leading energy producing of limitations ran out. Urgently needing money
states “to be used solely for the development but aware that she had no legal recourse, the
of synthetic materials to replace petroleum.” sister asked her brother if he would be able to
The state accepted the funds and distributed $3 repay any part of the balance of the loan. The
million from those funds to a state university, brother agreed in writing to assign to his sister
an agency of the state government. The univer- a debt of $2,000 that was owed to him by his
sity applied the $3 million to its ongoing energy stepson and was coming due in a week, and the
research program, which included projects inves- sister accepted. The brother informed his stepson
tigating the creation of synthetic replacements of the assignment and instructed him to pay the
for petroleum products, solar power, geothermal sister. When the $2,000 debt became due, the
power, biomass, and fusion power generation. stepson refused to pay her.
The Justice Department brings suit in federal
district court seeking to enjoin the expenditure If the sister brings an action against the
of the funds in the university research programs. stepson to collect the debt, will she likely
prevail?
How should the court rule?
(A) No, because partial assignments are inef-
(A) The university expenditure is unconstitu- fective.
tional because Congress has improperly
delegated its power to spend for the general (B) No, because she cannot sue the stepson
welfare to a state government. on the same debt that she is barred by the
statute of limitations from recovering from
(B) The expenditure is unconstitutional as a her brother.
violation of the Supremacy Clause.
(C) Yes, because the stepson may not raise the
(C) The expenditure is constitutional because brother’s statute of limitations defense on
it conforms substantially with the purpose the original contract.
and goals of the federal program.
(D) Yes, because the assignment is valid as a
(D) The expenditure is constitutional because novation.
the federal government’s attempt to control
the actions of a state agency violates the
concepts of federalism.

GO ON TO THE NEXT PAGE

MPQ 100 preview diagnostic exam and divider P.indd 37 12/22/2015 4:13:31 PM
38. MBE PREVIEW DIAGNOSTIC EXAM

Question 67 Question 68

A landowner in fee simple signed a promis- A developer owned an office building


sory note for $10,000 to a bank, and secured the subject to a first mortgage with a creditor in the
note by a mortgage of her land to the bank. The amount of $1 million. Subsequently, the devel-
mortgage was duly recorded. The landowner oper borrowed $100,000 from a bank secured
then sold the property to an attorney, who by a second mortgage on the building to help
assumed and agreed to pay the mortgage to the pay the first mortgage and other expenses of
bank on the land. The attorney did not make the building. The developer’s financial condi-
payments on the mortgage note to the bank. The tion worsened, and he was unable to make the
bank, following appropriate statutory proce- required payments on the first mortgage to the
dures, foreclosed the mortgage and gave notice creditor. The developer approached the creditor
to both the landowner and the attorney that it and offered to give her a deed to the building
intended to sue for any deficiency. At the foreclo- in satisfaction of all of his obligations to the
sure sale, the property sold for $6,000. The bank creditor. The developer delivered to the creditor
now sues both the landowner and the attorney a quitclaim deed to the building, which recited
for $5,000, which is the remaining amount of as consideration the release of the developer
the unpaid principal and interest on the note plus from all liability on the mortgage to the creditor.
costs of foreclosure. The deed was duly recorded.

Against which party will the bank be Shortly thereafter, the office market greatly
successful in obtaining a judgment? improved, and the building was worth $1.5
million. The developer then brought an action
(A) Only the landowner. against the creditor, claiming that the deed was
an equitable mortgage, and the bank served
(B) Only the attorney. notice on the creditor that it was preparing to
foreclose its mortgage on the building.
(C) Either the landowner or the attorney.
Against which parties will the creditor
(D) Both the landowner and the attorney. prevail?

(A) The developer only.

(B) The bank only.

(C) Both the developer and the bank.

(D) Neither the developer nor the bank.

GO ON TO THE NEXT PAGE

MPQ 100 preview diagnostic exam and divider P.indd 38 12/22/2015 4:13:31 PM
MBE PREVIEW DIAGNOSTIC EXAM 39.

Question 69 Question 70

A chemical plant and a steel mill located on A bank executive was on trial for embezzling
a river both negligently discharged toxic waste $10,000 from the bank where he worked. A key
into the river during the same time period. A witness for the prosecution was called to testify,
farmer downstream who used water from the but on the stand he had difficulty remembering
river for irrigation suffered substantial crop the specifics of a conversation he had with the
losses as a result of using the poisoned water. He executive regarding the executive’s accounting
brought suit against the chemical plant. At trial, procedures. To refresh the recollection of the
the evidence established that the discharge from witness, the prosecutor showed the witness a
either plant alone was sufficient to have caused memorandum that the witness had written for
the farmer’s crop losses. his file, detailing the conversation. The witness
reviewed the memorandum, and then testified
How much of the farmer’s damages should he that he recalled the conversation. He proceeded
recover from the chemical plant? to testify about the specifics of the conversa-
tion. The defense counsel then asked that the
(A) All of his damages, because the chemical memorandum be introduced into evidence.
plant’s negligence was a substantial factor
in causing the farmer’s damage. How should the court rule on the
memorandum?
(B) Half of his damages, because he did not
present evidence to allow the court to (A) Exclude the evidence if it determines that
reasonably apportion damages between the the witness’s memory was refreshed.
two tortfeasors.
(B) Exclude the evidence if it determines that
(C) None of his damages, because he needed to a reasonable jury would conclude that the
join the other tortfeasor in the litigation. witness’s memory was refreshed.

(D) None of his damages, because the harm (C) Admit the evidence if it determines that the
would have occurred even in the absence of witness’s memory was not refreshed.
the chemical plant’s negligence.
(D) Admit the evidence if it was used to refresh
the witness’s recollection.

GO ON TO THE NEXT PAGE

MPQ 100 preview diagnostic exam and divider P.indd 39 12/22/2015 4:13:31 PM
40. MBE PREVIEW DIAGNOSTIC EXAM

Question 71 Question 72

A boater and a water skier were involved in A toy makers’ union, angry about poor
a boating accident. Shortly after the accident, working conditions and low wages, staged a
the water skier prepared a written summary of nationwide strike just weeks before the holiday
the events surrounding the accident. At trial two season. Larger toy sellers immediately hired
years later, the water skier is on the stand and independent toy makers to fulfill the traditionally
is unable to accurately recall the details of the increased demand for toys during the holidays.
accident, even after reviewing the aforemen- Enraged, striking toy makers committed acts
tioned account of the accident. of violence against independent toy makers and
attempted to destroy shipments of independently
Assuming a proper foundation is laid, may the made toys as they were being loaded off trucks
summary of the accident be read into evidence? at toy sellers’ receiving docks.

(A) No, because the best evidence is the writ- In response to the increasing violence,
ing itself. Congress met in emergency session and enacted
a measure directing the President to send
(B) Yes, because the water skier’s memory of military troops to the affected areas to preserve
the actual event is insufficient. order and to ensure the continued flow of
commerce.
(C) Yes, because it refreshes the water skier’s
recollection. Is this enactment constitutional?
(D) Yes, even though hearsay, because the (A) Yes, under Congress’s power to raise and
out-of-court declarant is on the stand and is support the armed forces.
capable of being cross-examined.
(B) Yes, under Congress’s power to regulate
commerce.

(C) No, because it infringes on the President’s


authority to faithfully execute the laws of
the United States.

(D) No, because it infringes on the President’s


authority as Commander in Chief of the
armed forces.

GO ON TO THE NEXT PAGE

MPQ 100 preview diagnostic exam and divider P.indd 40 12/22/2015 4:13:31 PM
MBE PREVIEW DIAGNOSTIC EXAM 41.

Question 73 Question 74

A stockbroker visited a customer at the A landlord entered into a written four-year


customer’s office and sold her some securities. lease with a tenant for an apartment in the
Coincidentally, three days after the sale, the landlord’s apartment house. The tenant’s lease,
stockbroker and customer were involved in a car and all leases in the apartment house, prohibited
accident on the freeway into the city. After the the playing of musical instruments between 10
securities dramatically declined in value, the p.m. and 8 a.m. The lease required the tenant
customer determined that the broker violated to pay the rent on a monthly basis. Two years
federal securities statutes when the broker sold into the lease, the tenant assigned the lease to a
the securities. The customer and the broker are nurse with the landlord’s permission. The nurse
citizens of the same state. The customer filed an then assigned the lease to his brother with the
action against the broker in federal district court landlord’s permission. The brother went into
asserting a claim for $70,000 for the broker’s possession. A neighboring tenant in the same
violation of federal securities statutes and a apartment house insisted upon playing a trumpet
claim for $4,000 for the broker’s negligence in in a loud manner between 2 a.m. and 4 a.m.
damaging the customer’s car. The brother complained to the landlord without
success. Unable to sleep each night, the brother
May a federal court hear these claims abandoned his apartment after occupying it for
together? two months.

(A) No, because unrelated claims may not be If the landlord sues the nurse for the rent due
joined in the same action. during the period after the nurse’s brother left,
what would be the nurse’s best defense?
(B) No, because the federal court lacks subject
matter jurisdiction over the negligence (A) A breach of the landlord’s covenant of
claim. quiet enjoyment.

(C) Yes, because the amount in controversy is (B) Estoppel, because the landlord consented to
irrelevant in federal question cases. a further assignment.

(D) Yes, because the court has federal question (C) Lack of privity of estate.
jurisdiction over the statutory securities
claim and supplemental jurisdiction over (D) Constructive eviction.
the negligence claim.

GO ON TO THE NEXT PAGE

MPQ 100 preview diagnostic exam and divider P.indd 41 12/22/2015 4:13:31 PM
42. MBE PREVIEW DIAGNOSTIC EXAM

Question 75 Question 76

The plaintiff, who resides in the Southern During a defendant’s trial for murder, the state
District of State A, was involved in a three-car introduced circumstantial evidence tending to
accident in the Northern District of State A. establish her guilt, but could not provide direct
The plaintiff intends to file a negligence action evidence that she had committed the murder. No
against the other two drivers in federal district one had seen the killer going into or coming out
court. One defendant resides in the District of of the victim’s office around 2:30 p.m., the time
State B and the other resides in the District of of the victim’s death by strangulation. During
State C. her defense case, the defendant took the stand
and testified that she could not have committed
In which federal district(s) is venue proper? the charged murder, because on the day of the
victim’s death she had been on a business trip
(A) The Northern District of State A only. 3,000 miles away. The prosecution then called a
rebuttal witness. The witness will testify that, at
(B) The District of State B and the District of 1 p.m. on the day of the charged murder, he had
State C. put his hand on the defendant’s knee while both
sat at a bar across town. The defendant had then
(C) The Northern District of State A, the coquettishly removed his necktie and attempted
District of State B, and the District of State to strangle him with it until bystanders pulled
C. her away and she left the bar. The defendant
objects to admission of the witness’s testimony.
(D) The Northern District of State A, the
Southern District of State A, the District of How should the trial judge rule?
State B, and the District of State C.
(A) For the defendant, because she did not
place her character at issue in the trial.

(B) For the defendant, because the testimony


is character evidence not directed at her
character for truth and veracity.

(C) For the state, because the testimony estab-


lished that the defendant had the opportu-
nity to commit the charged crime.

(D) For the state, because the testimony estab-


lished that the defendant has a character for
violence.

GO ON TO THE NEXT PAGE

MPQ 100 preview diagnostic exam and divider P.indd 42 12/22/2015 4:13:31 PM
MBE PREVIEW DIAGNOSTIC EXAM 43.

Question 77 Question 78

On April 4, a homeowner hired an electri- A developer purchased a downtown lot on


cian to evaluate his home’s wiring. The electri- which she intended to have an office building
cian found the wiring to be faulty, called it a constructed. She entered into a written agree-
fire hazard, and urged the homeowner to fix it ment with a construction company to build a
soon. That day, the homeowner entered into a 50-story office tower for $250 million. The
written contract with the electrician, whereby agreement specified that monthly progress
the electrician agreed to repair the wiring for payments of $20 million were to commence one
$1,600. The contract provided that the repair month after the foundation was in place, with
would take place prior to April 30, on a day all remaining amounts due upon approval of the
when the homeowner would be home. On completed structure by the developer’s architect.
April 26, the work had yet to be done so the Construction commenced on March 10, and by
homeowner called the electrician to pin down April 1 the foundation was completed. On the
a date. The electrician suggested April 29, and night of April 30, a terrorist group detonated 500
the homeowner agreed. On the morning of April pounds of explosives in the partially completed
29, the electrician arrived ready to do the repair structure, totally destroying the building plus
work. However, he discovered that no one was all of the construction equipment involved in
home, and he could not get inside. Earlier that the project. The construction company was
morning, the homeowner had been rushed to forced into bankruptcy when its casualty
the hospital for an emergency appendectomy, insurer classified the loss as having occurred
and the homeowner did not return home from in an “undeclared war,” and refused to replace
the hospital until May 1. On that day, the faulty the lost equipment. The developer engaged
wiring caused a fire that severely damaged the another construction company to construct the
house. 50-story tower as originally planned at a price
of $260 million. The trustee in bankruptcy has
Will the electrician succeed in a breach of now brought an action against the developer to
contract action? recover all payments due the company for the
work completed prior to the terrorist act.
(A) Yes, because the homeowner made and
breached an implied-in-fact promise to How much is the trustee likely to recover?
make the house available to the electrician
before April 30. (A) $20 million.

(B) Yes, because any recovery by the electri- (B) The difference between $20 million and
cian will be subject to a setoff by the the cost to the construction company of the
homeowner on account of his damage from work completed at the time of the terrorist
the fire. attack.

(C) No, because the electrician did nothing (C) The reasonable value of the labor and
about the agreement between April 4 and materials expended by the construction
April 26. company at the time of the terrorist attack.

(D) No, because the homeowner’s obliga- (D) Nothing.


tions under the contract were subject to
an express condition precedent that failed
because of supervening impossibility.

GO ON TO THE NEXT PAGE

MPQ 100 preview diagnostic exam and divider P.indd 43 12/22/2015 4:13:31 PM
44. MBE PREVIEW DIAGNOSTIC EXAM

Question 79 Question 80

In order to protect its citizens from “loan A pedestrian filed an action against a driver
sharks” and unscrupulous business practices, a in federal district court, alleging negligence.
state had an anti-usury statute that prohibited The attorney for the driver has interviewed an
loans for an interest rate in excess of 12%. When eyewitness whose testimony will clearly indicate
interest rates rose throughout the nation and the that the driver was at fault.
federal reserve began lending money to banks
at 11.5%, banks in that state could not profit- Must the driver disclose the existence and
ably make mortgage loans. Consequently, only identity of the eyewitness to the pedestrian?
extremely wealthy persons who could afford
to pay cash for a home were able to purchase (A) No, because the identity of the eyewitness
homes or farms in the state. is protected from discovery under the work
product doctrine.
If an appropriate party challenges the consti-
tutionality of the anti-usury statute, how should (B) No, because the driver is not likely to use
the court rule? the eyewitness as part of her case.

(A) The statute is constitutional, because the (C) Yes, because the driver must disclose all
statute does not conflict with a specific witnesses who have discoverable informa-
federal statute. tion, even without a specific request.

(B) The statute is constitutional, because the (D) Yes, but only in response to an appropriate
state acted properly under its police power interrogatory.
in enacting the statute.

(C) The statute is unconstitutional, because


the statute denies nonwealthy people equal
protection of the law.

(D) The statute is unconstitutional, because it


restrains interstate commerce.

GO ON TO THE NEXT PAGE

MPQ 100 preview diagnostic exam and divider P.indd 44 12/22/2015 4:13:31 PM
MBE PREVIEW DIAGNOSTIC EXAM 45.

Question 81 Question 82

A gangster was threatening to inflict a “disfig- A pilot had given her 15-year-old son informal
uring injury” on a shopkeeper if the shopkeeper piloting lessons, including practice on takeoffs
didn’t accept “gang protection.” The shopkeeper and landings on their farm’s landing strip.
refused to pay and told the gangster to get out Confident in her son’s ability, she agreed to the
of his store. The gangster pulled out a switch- son’s request to fly by himself to his uncle’s
blade and he flipped it at the shopkeeper. The ranch, which also had a landing strip, to attend
knife thunked into the wall a few centimeters his cousin’s high school graduation party. The
from the shopkeeper’s left ear. “Oops. I slipped,” son took off safely, but about five minutes later
said the gangster. “This is your last chance. the aircraft’s engine stopped running; the plane
Without my protection, you’re a dead man.” The stalled and then crashed, killing the son. It was
gangster turned and began to walk from the later determined that the engine ran out of fuel
store. The shopkeeper took a revolver from under because a burst fuel line had been overlooked
the cash register, aimed it at the gangster, and during a routine maintenance check of the pilot’s
shot. However, the shopkeeper missed, killing a plane the week before the crash. An airplane
delivery person instead. mechanic at the small private airport where
the pilot went for fuel and maintenance had
At the shopkeeper’s subsequent trial for performed the maintenance check.
the murder of the delivery person, which of
the following is the strongest argument that The pilot brought a wrongful death action
the shopkeeper can only be convicted of for the loss of her son against the mechanic’s
manslaughter? employer, the owner of the private airport. At
trial, the owner of the airport raises a contribu-
(A) The shopkeeper did not intend to kill or tory negligence defense.
seriously injure the delivery person.
Whose negligence, if found by the jury to be a
(B) The shopkeeper was acting in self-defense contributing factor in causing the accident, will
when he shot at the gangster. reduce the pilot’s recovery?

(C) The shopkeeper was acting under duress (A) The pilot in permitting her son to fly the
when he shot the delivery person. airplane.

(D) The shopkeeper was reacting to a sufficient (B) The son in his handling of the plane after
provocation to mitigate the homicide. the engine quit.

(C) Both the pilot in permitting her son to fly


and the son in handling the plane.

(D) Either the pilot in permitting her son to fly


or the son in handling the plane.

GO ON TO THE NEXT PAGE

MPQ 100 preview diagnostic exam and divider P.indd 45 12/22/2015 4:13:31 PM
46. MBE PREVIEW DIAGNOSTIC EXAM

Question 83 Question 84

A man put his leaf mulcher back into his A nursing home, as a matter of policy,
garage after finishing his yard and then got into admitted patients known for violent behavior.
his car and drove away, accidentally leaving his Each night before the patients went to bed, all
garage door standing wide open. An envious doors were locked so that intruders could not
neighbor raking his leaves across the street enter the premises without forcing an entry. One
noticed the open garage and decided to help morning an elderly and docile patient was found
himself to the use of the man’s leaf mulcher. The beaten in her bed. The doors had been locked
neighbor thought he could quickly finish his own and there were no signs of forced entry into the
leaves before the man returned without the man building during the previous night.
ever realizing that it had been used. However,
the man pulled up in the driveway before the The patient sued the nursing home for her
neighbor had taken three steps with the mulcher injuries and established the above facts. At the
in hand. The neighbor quickly put down the close of the patient’s case, the nursing home
mulcher and exited the garage, but the man was moved for a directed verdict.
furious and called the police.
How should the court rule?
Why will the neighbor be found not guilty of
larceny? (A) Deny the motion, because the jury could
find that an occurrence such as this would
(A) The neighbor had not used the mulcher. not ordinarily happen unless the nursing
home was negligent.
(B) The garage door was left standing wide
open when the neighbor took the mulcher. (B) Deny the motion, because the nursing home
is required to exercise a very high degree of
(C) The neighbor planned to return the mulcher care towards the patients in its care.
before the man found out.
(C) Grant the motion, because the patient did
(D) The neighbor was still in the garage when not establish that the beating was caused by
the man stopped him. an employee of the nursing home.

(D) Grant the motion, because the beating was


caused by the criminal act of a third party.

GO ON TO THE NEXT PAGE

MPQ 100 preview diagnostic exam and divider P.indd 46 12/22/2015 4:13:31 PM
MBE PREVIEW DIAGNOSTIC EXAM 47.

Question 85 Question 86

A seller entered into a written land sale A pedestrian and driver were involved in
contract with a buyer on May 20, whereby the an automobile crash in the Eastern District of
seller agreed to sell a home to the buyer for State A. The pedestrian resides in the Southern
$60,000. The closing date was set at August 1. District of State A, and the driver resides in the
The buyer put up $6,000 as earnest money, as District of State B. The pedestrian filed a negli-
provided by the contract. The contract stated gence action in the United States District Court
that if the buyer failed to perform by tendering for the Southern District of State A properly
the balance due on the house on August 1, the based on diversity jurisdiction. The driver
$6,000 could be treated as liquidated damages responded by filing a motion to dismiss the
“at the option of the seller.” action for improper venue or, in the alternative,
to transfer the action to the Northern District of
On July 21, the week before the closing, the State A.
house burned to the ground because of a freak
lightning strike during a thunderstorm. When What options for disposition are available to
August 1 arrived, the buyer refused to tender the court?
$54,000 to the seller. The buyer asked the seller
for the refund of his earnest money because the (A) Deny dismissal or transfer because the
house had been destroyed. The seller refused Southern District of State A is a proper
and filed suit, asking for specific performance. venue.
The buyer countersued, demanding refund of the
$6,000 earnest money. (B) Either dismiss the action or grant transfer to
the Northern District of State A.
How should the court rule on the suits?
(C) Deny transfer of the action to the Northern
(A) The court will order specific performance District of State A because the Northern
by the buyer. District of State A is not a proper venue.

(B) The court will order the seller to return the (D) Grant transfer of the action to the Northern
$6,000 to the buyer because of frustration District of State A only if it determines that
of purpose of the contract. the Northern District is a fair and conve-
nient forum.
(C) The court will award the seller $6,000 as
liquidated damages.

(D) The court will award neither party the relief


sought because of mutual mistake.

GO ON TO THE NEXT PAGE

MPQ 100 preview diagnostic exam and divider P.indd 47 12/22/2015 4:13:31 PM
48. MBE PREVIEW DIAGNOSTIC EXAM

Question 87 Question 88

A State A citizen and a State B citizen A witness who was unable to speak was an
were involved in a car accident. The State B eyewitness to a robbery in the park. He went to
citizen filed a negligence action against the the police station shortly after the crime was
State A citizen in federal district court, seeking committed, and when asked who committed
$500,000 for injuries incurred in the accident. the robbery, pointed to the defendant in a
The State A citizen believes that she was not police lineup in which there were several men
at fault and that the accident was caused by the who were dressed like the defendant and were
negligence of the State B citizen. the defendant’s size. The defendant was then
indicted for robbery. The witness died before
May the State A citizen assert in the pending trial, and the sergeant in charge of the lineup
action a negligence claim against the State B desires to testify at the defendant’s trial that the
citizen, seeking $400,000 for the injuries the witness pointed to the defendant at the lineup.
State A citizen suffered in the accident?
Is his testimony admissible?
(A) No, joinder is improper under these cir-
cumstances. (A) Yes, as evidence of prior identification.

(B) Yes, and the court has discretion to grant or (B) Yes, to prove the defendant committed the
deny the motion in the interest of justice. crime.

(C) Yes, as a counterclaim in the pending (C) No, because the sergeant’s testimony is
action, or she may assert it as an indepen- hearsay.
dent action.
(D) No, because the defendant’s counsel was
(D) Yes, but only as a counterclaim in the not present at the lineup.
pending action and will be barred from
asserting it as an independent action.

GO ON TO THE NEXT PAGE

MPQ 100 preview diagnostic exam and divider P.indd 48 12/22/2015 4:13:31 PM
MBE PREVIEW DIAGNOSTIC EXAM 49.

Question 89 Has the lawyer violated Rule 11 of the Federal


Rules of Civil Procedure?
A corporation filed a breach of contract action
against an individual in federal district court. (A) No, because when the lawyer signed the
Attached to the complaint was a written contract complaint, he had evidentiary support in
providing that the individual agreed to pay the the expected testimony of the individual
corporation $100,000 over a period of years for who denied signing the contract.
specified services. The contract also contained
the individual’s signature. The individual told his (B) No, because the lawyer failed to state in the
lawyer that the signature was a forgery and that motion hearing that the signature on the
she had never signed or entered into the contract. contract was not that of the individual.
On the basis of the individual’s statement, the
lawyer drafted, signed, and filed an answer. The (C) Yes, because when the lawyer referred
answer denied the claim on a number of grounds to the answer in the motion hearing, he
and denied that the signature on the contract was renewed his certification that facts in the
that of the individual. The individual’s lawyer answer had evidentiary support at the time
later served on the corporation a request for of the hearing.
production of documents. When the corpora-
tion objected to some of the requests, the lawyer (D) Yes, because the lawyer should not have
filed a motion to compel production. Shortly signed the answer without having the
before the hearing on the motion to compel, the individual sign a sworn statement that the
individual advised the lawyer that the signature signature on the contract was not hers.
on the contract was in fact hers, but she and
the lawyer agreed that she nonetheless should
not be liable on the contract for other reasons.
The lawyer thus continued to assert the motion
to compel production. At the hearing on the
motion, the lawyer referred to the complaint
and the answer to justify the relevancy of the
requests for production, but he did not mention
the signature in any way.

GO ON TO THE NEXT PAGE

MPQ 100 preview diagnostic exam and divider P.indd 49 12/22/2015 4:13:31 PM
50. MBE PREVIEW DIAGNOSTIC EXAM

Question 90 Question 91

The President of the United States issued A mother had her lawyer prepare and execute
an executive order banning conversation by all two notarized deeds: one conveying a 640-acre
executive employees with members of the press farm to her daughter and the other conveying a
unless prior permission had been obtained from 590-acre farm to her son. The mother produced
a supervisor. Executive Department employees the deed to the daughter, who told her mother
were subject to dismissal for violation of the to keep the deed just in case the daughter
order. An employee of the United States Depart- misplaced it. The mother held on to the deed.
ment of Agriculture spoke to a reporter and The next day, the mother went to her lawyer’s
told the reporter that the USDA sanctioned the office to have the deeds recorded, but she forgot
approval of a dangerous pesticide because of the daughter’s deed at home. The mother gave
payoffs made by lobbyists to a high-ranking the lawyer the son’s deed and promised to bring
USDA official. The newspaper printed the story the daughter’s deed in the next day. Later that
and quoted the employee by name. After a afternoon, the lawyer recorded the son’s deed.
hearing in line with civil service regulations, the That night, the mother suffered a stroke and
employee was found to have violated the execu- died in her sleep. The daughter’s deed was never
tive order barring unauthorized conversations recorded but was found in the mother’s home
with news media reporters, and the employee after her death. In her will, the mother left all
was fired. The employee brought suit in federal of her property to her daughter, her son, and the
court for reinstatement, back pay, and other local animal shelter, in three equal shares. Aside
benefits and also prayed that the court should from her modest home and some small bank
strike down the executive order as unconstitu- accounts, the mother had no appreciable estate.
tional.
If the animal shelter challenges the deeds to
If the employee’s case reaches the United the farms on grounds of lack of proper delivery,
States Supreme Court, how should the Court what is the likely result?
rule?
(A) The shelter will win on both farms.
(A) The executive order is constitutional, be-
cause the President has plenary power to (B) The shelter will lose on both farms.
control Executive Department employees.
(C) The shelter will win on the 640-acre farm
(B) The executive order is constitutional, but lose on the 590-acre farm.
because government employment is a privi-
lege and not a right. (D) The shelter will win on the 590-acre farm
but lose on the 640-acre farm.
(C) The executive order is unconstitutional,
because Congress, rather than the President,
has authority to set the terms of federal
employment.

(D) The executive order is unconstitutional,


because the President cannot broadly limit
all executive employees’ freedom of speech
and association.

GO ON TO THE NEXT PAGE

MPQ 100 preview diagnostic exam and divider P.indd 50 12/22/2015 4:13:31 PM
MBE PREVIEW DIAGNOSTIC EXAM 51.

Question 92 If the buyer seeks an injunction prohibiting


the telephone company from undertaking the
Thirty years ago, a telephone company planned excavation, for which party should the
purchased an easement from an owner to install, trial court rule?
inspect, repair, replace, and maintain under-
ground transmission cables within a speci- (A) The telephone company, because the ben-
fied portion of the owner’s property. The deed efits of installing the new cables outweigh
granting the easement was validly recorded, and the harm done to the buyer.
the telephone company shortly thereafter laid
underground long distance lines traversing the (B) The telephone company, because owner-
owner’s land beneath the area described in the ship of the easement permits it to make the
easement deed. excavation.

Fifteen years later, the owner sold a 50-acre (C) The buyer, because the telephone company
portion of his land to a buyer. This portion concealed the existence of the underground
included part of the easement granted to easement from the buyer when it acquired
the telephone company. The buyer erected a the overhead easement from him.
residence on the purchased land and planted 49
acres of walnut trees. He granted an easement to (D) The buyer, because the deed by which he
the telephone company to string aerial telephone purchased his property made no mention of
wires. The underground easement was never the easement, and he had no actual knowl-
disclosed. edge of it.

Last year, the telephone company informed


the buyer that it would have to excavate under-
ground cables and replace them with new ones.
This was the first the buyer knew of the cables
lying under his land. The cables, lying within
the easement, are directly beneath a row of black
walnut trees running the length of the buyer’s
property. Black walnuts have become extremely
valuable, and the buyer estimates that their value
is approximately $400,000.

GO ON TO THE NEXT PAGE

MPQ 100 preview diagnostic exam and divider P.indd 51 12/22/2015 4:13:31 PM
52. MBE PREVIEW DIAGNOSTIC EXAM

Question 93 Question 94

A tourist was struck and killed at a street A homeowner raked up numerous dead, dry
crossing by an automobile. The accident was leaves that had fallen on his yard and set fire
witnessed by a bystander and by the tourist’s to the pile, even though the wind was blowing
wife. The wife brought an action against the at 15-20 miles per hour that day, and a county
automobile driver for wrongful death, alleging ordinance made it unlawful to burn leaves on
excessive speed, failure to observe traffic signals, any day when the wind speed exceeded 10
and defective brakes. A witness for the driver miles per hour. After the homeowner went
was called to testify that the driver enjoyed a into his house, the wind whipped the burning
reputation for being a safe and prudent driver leaves into the air and deposited some of them,
based on his personal knowledge of the driver’s still burning, on the wood-shingled roof of a
driving habits. neighboring house, setting the house on fire.
A driver was passing by and saw the burning
How should the trial judge rule on the roof of the neighbor’s house. He pulled over, got
witness’s testimony? out of his car and ran across the street toward
the neighbor’s yard so that he could warn the
(A) Admissible, because, where there are no occupants of the house. The driver was struck by
unbiased eyewitnesses to an accident, repu- a paramedic’s vehicle just arriving in response to
tation as a safe driver may be used to prove a telephone report of the fire. The driver brought
the driver acted in conformity with that an action against the homeowner to recover for
reputation at the time in question. the injuries he suffered from being struck by the
emergency vehicle.
(B) Admissible, because the witness first testi-
fied that he has personal knowledge of the How should the court rule in this action?
driver’s driving habits.
(A) For the driver, because the homeowner’s
(C) Inadmissible, because evidence of a reputa- negligence was a proximate cause of the
tion as a safe and prudent driver cannot driver’s injuries.
be used to prove that the driver acted in
conformity with that reputation at the time (B) For the driver, because the homeowner’s
in question. conduct constituted negligence per se.

(D) Inadmissible, because, in a civil case, (C) For the homeowner, because the emergency
character evidence may be used only after vehicle was the actual cause of the driver’s
the plaintiff has attacked the character of injuries.
the defendant.
(D) For the homeowner, because one who
responds to a fire assumes the risk of
foreseeable dangers.

GO ON TO THE NEXT PAGE

MPQ 100 preview diagnostic exam and divider P.indd 52 12/22/2015 4:13:31 PM
MBE PREVIEW DIAGNOSTIC EXAM 53.

Question 95 Question 96

A plaintiff filed a breach of contract action A privately owned bus company operated
against a defendant in federal district court, over fixed city routes under a franchise granted
invoking the court’s diversity of citizenship juris- to it by the city council. To enhance revenues,
diction. The defendant filed an answer denying the bus company rented advertising space on its
the material allegations in the complaint. vehicles. Although the city received a certain
Approximately two months after the answer was fixed percentage of the fare revenues from the
served, the court entered a scheduling order that company, it received nothing from the advertising
required the parties to complete all discovery fees. The management of the bus company had
within 10 months after the entry of the sched- learned through bitter experience that any adver-
uling order. Two months later (four months tising that could be characterized as “controver-
after service of the answer and two months into sial” led to complaints from riders and sometimes
the discovery period), the defendant sought to to losses in revenue. To that end, management
amend his answer to add an affirmative defense employs a firm “no controversial advertising”
that the plaintiff’s claim was barred by the policy. Thus, they refused to take ads for X-rated
statute of limitations. movies, abortion clinics, political candidates, and
anything else that might be deemed “controver-
Which of the following statements is correct sial.” A candidate for city council was waging
regarding the defendant’s right to amend his a populist campaign for the job and among
answer to add an affirmative defense? his targets were the local utilities and the bus
company. The candidate demanded that he be
(A) The defendant has a right to amend his allowed to buy advertising on the buses, but the
answer any time before trial. company’s advertising manager refused, citing
the company ban on all political ads. The candi-
(B) The defendant must obtain leave of the date filed suit in federal district court, asking that
court to amend his answer, but the court the court require the company to accept his ads.
should freely grant leave.
Which of the following is the company’s best
(C) The defendant may amend his answer to argument against the candidate’s constitutional
add the affirmative defense only if he can claim?
show he could not, with due diligence, have (A) The company is a privately owned com-
discovered the defense prior to serving his pany, and thus there is insufficient “state
answer. action” to invoke the United States Consti-
tution.
(D) The defendant may not amend his answer
more than 21 days after serving it. (B) Commercial speech receives less protection
under the First Amendment than most other
forms of speech.
(C) Bus riders have First and Fourteenth
Amendment rights to see the type of adver-
tising they desire, and past experience has
shown that there is substantial opposition
by riders to controversial ads.
(D) The candidate has many other sources of
advertising available and he does not need
ads on the company’s vehicles to get his
message to the public.

GO ON TO THE NEXT PAGE

MPQ 100 preview diagnostic exam and divider P.indd 53 12/22/2015 4:13:31 PM
54. MBE PREVIEW DIAGNOSTIC EXAM

Question 97 Question 98

A software designer and software programmer A large wholesaler of European clothing sent
ran into each other at a crowded software out a circular to all of its customers advertising
convention. The designer suggested to the fine quality coats for sale at a price of $300 each
programmer that they become partners. The when purchased in lots of 100 coats. The circular
programmer quickly retorted, “Look buddy, I also stated that payment for each lot was to be
like you, you’re a nice guy and all that, but I’d made in three $10,000 installments, the first
never become your business partner, because payment due within 10 days of delivery and the
when it comes to software design, you are totally subsequent installments within 40 and 70 days of
incompetent.” This remark was overheard by the delivery, respectively. The day after receiving the
private secretary to the CEO of a major software circular, a retailer sent the wholesaler a standard
company that often subcontracted portions of order form with price, quantity, and delivery date
jobs to smaller software design businesses, terms filled in by the retailer, clearly indicating
although it had never used this designer before. that the retailer was ordering one lot of 100
coats. The wholesaler shipped the coats to the
If the designer sues the programmer for retailer, and they arrived on September 10. The
defamation, who is likely to prevail? retailer accepted delivery of the coats and, on
September 11, mailed to the wholesaler a check
(A) The programmer, if her statement about the for $29,400. The retailer enclosed a letter with
designer’s competence is true. the check stating, in relevant part, “By cashing
the enclosed check in the amount of $29,400,
(B) The programmer, because there was no we assume that you accept our standard policy
publication. of taking a 2% discount for early payment.” The
wholesaler received the check on September 13,
(C) The designer, unless the programmer did read the letter, and cashed the check the next
not know her statement could have been day.
overheard.
What is the legal effect of the wholesaler’s
(D) The designer, if he suffered pecuniary loss cashing of the retailer’s check?
as a result of the programmer’s statement.
(A) The entire debt is discharged, because there
has been an accord and satisfaction.

(B) The entire debt is discharged on the basis of


account stated.

(C) The retailer remains liable for the unpaid


portion of the debt, because the discount
provision was never accepted as part of the
contract.

(D) The retailer remains liable for the unpaid


portion of the debt because the check did
not state “payment in full” or words to that
effect.

GO ON TO THE NEXT PAGE

MPQ 100 preview diagnostic exam and divider P.indd 54 12/22/2015 4:13:31 PM
MBE PREVIEW DIAGNOSTIC EXAM 55.

Question 99 Question 100

A suspect was captured in a high-speed police A large apple farm sells its product to various
chase five minutes after a bank robbery in which producers in the area who market apple products
the robbers had handed the teller a handwritten for commercial use. Prior to the harvesting
note demanding money. The suspect was taken season—the period when apple producers
to the police station where, over his protests, normally negotiate with processors and whole-
he was required to write out the words of the salers for sale of the apples to be harvested—the
note. He was then charged with the robbery. farm was contacted by a health foods corpora-
At a pretrial suppression hearing, the suspect tion that agreed in writing to purchase all of the
challenged the prosecution’s plan to offer in farm’s harvested apples. After the contract was
evidence the writing that he had been required signed, but before the harvest had begun, a rival
to make by the police so that the jury could apple farm offered to sell its apples to the corpo-
compare it with the robbers’ note. ration for 20% less than the price agreed to by
the first farm. The corporation signed a contract
How should the court rule on the admissibility with the rival farm for the purchase of raw apple
of the writing? products early enough before the beginning
of the harvest so that the first farm had plenty
(A) Admissible. of time to obtain another buyer for its apples.
The corporation informed the first farm that its
(B) Inadmissible, because the suspect was not requirements for apples for the next year would
advised that his handwriting sample could be about 5% of the farm’s total production.
be admitted into evidence against him.
If the first farm seeks specific performance of
(C) Inadmissible, because the suspect was its contract with the corporation, which party is
not advised of his right to refuse to give a likely to prevail?
handwriting sample.
(A) The farm, because performance of the
(D) Inadmissible, because the suspect had corporation-rival farm contract will auto-
not been informed he had a right to have matically breach the first farm’s contract.
counsel present.
(B) The farm, because its contract was prior in
time to the rival farm’s contract.

(C) The corporation, because the first farm can


resell its apples elsewhere and then seek
damages, if any, from the corporation.

(D) The corporation, because a seller of goods


cannot obtain specific performance.

STOP

MPQ 100 preview diagnostic exam and divider P.indd 55 12/22/2015 4:13:31 PM
MPQ 100 preview diagnostic exam and divider P.indd 56 12/22/2015 4:13:31 PM
EXPLANATORY ANSWERS

MPQ 100 preview diagnostic exam and divider P.indd 57 12/22/2015 4:13:32 PM
MPQ 100 preview diagnostic exam and divider P.indd 58 12/22/2015 4:13:32 PM
MBE PREVIEW DIAGNOSTIC EXAM 59.

ANSWER KEY AND SUBJECT KEY


Answer Subject Answer Subject
1. D Torts 51. B Torts
2. B Civil Procedure 52. A Real Property
3. B Evidence 53. A Torts
4. B Evidence 54. A Constitutional Law
5. D Criminal Law & Procedure 55. B Contracts
6. B Torts 56. D Contracts
7. B Contracts 57. B Criminal Law & Procedure
8. D Contracts 58. A Torts
9. C Real Property 59. D Civil Procedure
10. A Constitutional Law 60. B Criminal Law & Procedure
11. C Constitutional Law 61. B Real Property
12. D Criminal Law & Procedure 62. B Evidence
13. B Contracts 63. A Criminal Law & Procedure
14. A Real Property 64. B Criminal Law & Procedure
15. D Criminal Law & Procedure 65. B Constitutional Law
16. C Evidence 66. C Contracts
17. C Constitutional Law 67. D Real Property
18. B Civil Procedure 68. A Real Property
19. C Torts 69. A Torts
20. C Evidence 70. D Evidence
21. B Torts 71. B Evidence
22. D Constitutional Law 72. D Constitutional Law
23. B Constitutional Law 73. B Civil Procedure
24. C Real Property 74. C Real Property
25. C Evidence 75. A Civil Procedure
26. C Torts 76. C Evidence
27. A Torts 77. D Contracts
28. C Contracts 78. D Contracts
29. D Contracts 79. B Constitutional Law
30. D Criminal Law & Procedure 80. D Civil Procedure
31. A Evidence 81. D Criminal Law & Procedure
32. A Civil Procedure 82. D Torts
33. D Real Property 83. C Criminal Law & Procedure
34. C Civil Procedure 84. A Torts
35. B Criminal Law & Procedure 85. A Real Property
36. D Criminal Law & Procedure 86. C Civil Procedure
37. B Contracts 87. D Civil Procedure
38. A Civil Procedure 88. C Evidence
39. A Constitutional Law 89. C Civil Procedure
40. A Torts 90. D Constitutional Law
41. A Constitutional Law 91. B Real Property
42. D Contracts 92. B Real Property
43. C Contracts 93. C Evidence
44. D Evidence 94. A Torts
45. D Civil Procedure 95. B Civil Procedure
46. D Criminal Law & Procedure 96. A Constitutional Law
47. B Real Property 97. A Torts
48. D Real Property 98. A Contracts
49. B Constitutional Law 99. A Criminal Law & Procedure
50. C Evidence 100. C Contracts

MPQ 100 preview diagnostic exam and divider P.indd 59 12/22/2015 4:13:32 PM
MPQ 100 preview diagnostic exam and divider P.indd 60 12/22/2015 4:13:32 PM
MBE PREVIEW DIAGNOSTIC EXAM 61.

Answer to Question 1

(D) If the mechanic loses, it will be because he does not own or rent the affected property. A private
nuisance is a substantial and unreasonable interference with the use and enjoyment of real
property. Because the mechanic does not own or rent the property where he works, he may not
pursue a claim based on interference with that real property’s use and enjoyment. (A public
nuisance is an interference with the rights of the community at large—a situation not presented
by the fact pattern.) (A) is wrong because sound waves may be a basis for a nuisance action.
(C) is wrong because, if 40% of the people are bothered by the sound waves, they probably are
a nuisance. It is not necessary for a majority of the population to be affected for there to be a
nuisance, but more than one must be affected. The choice between (B) and (D) is a difficult one.
Nuisance requires an unreasonable interference with the property, and if the facts in (B) were
true, the defendants could arguably be acting reasonably. However, (D) is more clearly a basis for
the mechanic to lose than (B).

Answer to Question 2

(B) The plaintiff is a citizen of State B and the defendant is a citizen of State A. In addition to an
amount in controversy that exceeds $75,000, diversity of citizenship jurisdiction requires complete
diversity, meaning that each plaintiff must be a citizen of a different state from every defendant.
Whether complete diversity exists is determined when the suit is filed, not when the cause of
action arose or when the defendant is served with process. The citizenship of a natural person
is the state in which he is domiciled. A new state citizenship may be established by (i) physical
presence in a new place; and (ii) the intention to remain there permanently. In this question, the
plaintiff was originally from State A, but then moved permanently to State B before suit was filed.
After suit was filed, the defendant also moved to State B from State A. Because the plaintiff’s
move to State B was before he filed suit, he is considered to be a citizen of State B for purposes of
diversity jurisdiction, whereas the defendant is considered to be a citizen of State A because his
move did not occur until after suit was filed. Thus, complete diversity exists. Choices (A), (C), and
(D) are incorrect for the reasons stated above.

Answer to Question 3

(B) The silence is admissible nonhearsay as a statement by a party-opponent (commonly called an


admission). Under the Federal Rules of Evidence, silence in response to an accusatory state-
ment can be considered an implied admission if the party heard and understood the statement,
was capable of denying the statement, and a reasonable person would have denied the accusation
under the same circumstances. Here, there is nothing to indicate that the defendant did not hear or
understand the statement or that she was incapable of denying it, and a reasonable person who was
innocent would have in fact denied such an accusation. Thus, the defendant’s silence is admissible
as an admission, which is nonhearsay under the Federal Rules. (A) is incorrect because an implied
admission by silence is not considered hearsay at all, and therefore is not an exception to the
hearsay rule. (C) is incorrect because the rule against admissibility applies only if the accusation
or statement is made by the police; here, the accusation was made by the employer. (D) is incor-
rect because the Fifth Amendment right against self-incrimination, which provides that a defen-
dant cannot be compelled to testify against herself, applies only after the initiation of a criminal
proceeding. Here, the statement was made prior to the institution of criminal proceedings against
the defendant.

MPQ 100 preview diagnostic exam and divider P.indd 61 12/22/2015 4:13:32 PM
62. MBE PREVIEW DIAGNOSTIC EXAM

Answer to Question 4

(B) The witness’s testimony regarding the defendant’s conduct would be admissible. The defen-
dant’s acts constitute nonassertive conduct. Alternatively, this could be considered an admission
by conduct. Therefore, the testimony is direct, relevant, nonhearsay evidence. Because it is not
hearsay, (A) is incorrect. (C) is incorrect because evidence is not limited to testimony regarding
spoken words; a witness may testify as to what she saw. (D) is incorrect because the defendant did
not intend his conduct to be a substitute for words.

Answer to Question 5

(D) The woman is guilty of arson and burglary. She is guilty of arson because she deliberately set a
fire that, in addition to burning the mattress, also burned part of the dwelling house of another,
namely the floor. She is also guilty of burglary because she broke and entered into the dwelling
house of another during the nighttime to commit a felony. The fact that she was not successful in
committing the crime she intended or that she in fact committed another felony is irrelevant to
her guilt for burglary; it is the intent to commit a felony at the time of the breaking and entering
which is critical. (A) is incorrect. This choice correctly states that the woman is guilty of burglary,
but she is also guilty of arson; she deliberately set a fire that, in addition to burning the mattress,
also burned part of the dwelling house of another, namely the floor. Therefore, (B) and (C) are
incorrect.

Answer to Question 6

(B) If the jury finds for the manufacturer, it will be because the misuse by the consumer was not
foreseeable. A manufacturer is liable for a defective product, even if the plaintiff misuses it, as
long as that misuse is foreseeable. Foreseeability in this case is an issue for the trier of fact. If
the jury finds that the misuse was not foreseeable, the manufacturer will prevail. (A) is incorrect
because the consumer’s action is based on strict liability. (C) and (D) appear to be true based on
the facts, but they ignore the key issue that the jury must decide, which is whether the consumer’s
misuse was foreseeable.

Answer to Question 7

(B) The company should recover $25,000 only. Contracts for goods for $500 or more must be
evidenced by a writing to be enforceable. There are three exceptions to this rule: specially
manufactured goods unsuitable for resale in the seller’s regular course of business, contracts
admitted in court, and contracts partially accepted (enforceable to the extent of the acceptance).
Here, the contract was for $50,000 and was oral. Thus, it will be enforceable only if one of the
exceptions applies. The buyer’s acceptance of the first generator constitutes part acceptance that
will make the buyer liable to the extent of the acceptance: $25,000. Therefore, (B) is correct and
(A) is incorrect. (C) is incorrect because partial acceptance renders the buyer liable only for the
part accepted, not the entire contract. (D) is incorrect because, while the goods were made for the
buyer, they were suitable for resale in the company’s business, because they were built to standard
industry specifications.

Answer to Question 8

(D) If the engine company loses, it will be because it could have sold the motors in the ordinary
course of its business. This question concerns the specially manufactured goods exception to the
U.C.C.’s Statute of Frauds. In order for the specially manufactured goods exception to apply, there

MPQ 100 preview diagnostic exam and divider P.indd 62 12/22/2015 4:13:32 PM
MBE PREVIEW DIAGNOSTIC EXAM 63.

must be all three of the following elements: (i) the goods must be specially manufactured for the
buyer, (ii) the seller must have started work on the goods or else entered into a commitment to
purchase them from someone else, and (iii) the goods must not be sellable in the seller’s ordinary
course of business. If the goods are sellable, as (D) states, this exception does not apply. (A) and
(B) are incorrect because the seller need only have started work on the goods, which the engine
company had done. (C) is incorrect because the company was entitled to stop work after the
manufacturer’s repudiation.
Answer to Question 9
(C) The lawyer will likely prevail. When a subdivision is created with similar covenants in all deeds,
there is a mutual right of endorsement (each lot owner can enforce against every other lot owner)
if two things are satisfied: (i) a common scheme for development existed at the time that sales of
parcels in the subdivision began; and (ii) there was notice of the existence of the covenant to the
party sued. Here, there was a common scheme evidenced by the recorded plan, and the fact that
the covenant was in the architect’s chain of title gave her constructive notice of the restriction.
Therefore, not only does the covenant apply to the architect’s land, but the lawyer (or any other lot
owner) can enforce it as a reciprocal negative servitude. (A) is incorrect. While it is true that the
architect’s deed had no restrictions, those restrictions are binding if they are in her chain of title
so as to give her notice of them. The restriction was in the deed from the developer to the investor,
so the fact that it was omitted in the deed from the investor to the architect is of no significance.
(B) is incorrect. While a covenant is normally only enforceable by the party receiving the promise
(here, the developer), this is a situation of mutual rights of enforcement within a geographically
defined area, a special situation that gives every lot owner in the area the right of enforcement,
even though they did not directly receive the benefit of the promise. (D) is incorrect. The fact that
gives the lawyer the right of enforcement is not just the fact that his deed contains the covenant,
but that the same covenant was in all of the deeds from the developer, including the one to the
architect’s predecessor in title.
Answer to Question 10
(A) The strongest ground for supporting this provision is the Necessary and Proper Clause. Congress
has the authority to legislate to protect federal parklands. The federal law safeguards wild animals
that wander off federal parklands. The Necessary and Proper Clause allows Congress to choose
any means to carry out its constitutional powers. Thus, it may regulate hunting to protect the
federal interest in safeguarding the wild animals. (B) is wrong because it rests on an unjustified
factual statement, that the “animals are moving in the stream of interstate commerce.” There are
no facts describing the movement patterns of the animals. (C) is wrong because there is no federal
police power to protect wild animals. Generally, the federal government lacks the police power;
the police power belongs to state and local governments. Although Congress possesses the police
power in legislating for federal lands, this choice is not limited to those circumstances. It speaks
broadly of a federal police power to protect wild animals, and none exists. (D) is wrong because
the Supreme Court has discarded the distinction between rights and privileges as a constitutional
principle. Moreover, it is a distinction used to determine when the government must provide
procedural due process before deprivations of property or liberty. This question does not involve
the Due Process Clause.
Answer to Question 11
(C) The most likely ground for dismissal is lack of standing. Except for very limited exceptions,
the traditional rule is that a person has standing only to raise constitutional issues which affect

MPQ 100 preview diagnostic exam and divider P.indd 63 12/22/2015 4:13:32 PM
64. MBE PREVIEW DIAGNOSTIC EXAM

her personally. She cannot claim that a third person’s constitutional rights were violated. The
husband is the injured party and must assert his own rights. (A) is incorrect. The issue in this
question is not the substance of America’s foreign policy, which would be a political question,
but the husband’s First Amendment right to speak out about it, which is not a political question.
Therefore, the court is not barred from litigating this issue under the political question doctrine.
(B) is incorrect. There are damages to be assessed if the husband was improperly dismissed.
Under Mt. Healthy Board of Education v. Doyle (1977), a public employee cannot be fired without
a hearing for exercising the right to free speech, even if there is no property right in the job. The
case is not moot because the husband has lost his job permanently. The case would be moot only
if the husband had been reinstated to his former job at his former pay. (D) is incorrect. The subject
matter of the suit—the dismissal of an employee for exercise of his First Amendment rights—
presents a federal question.

Answer to Question 12

(D) The defendant should be found not guilty. Larceny is a specific intent crime that requires that the
defendant intend to permanently deprive the owner of his property. If the defendant can show
that she really believed the property was her own, there is no intent to deprive the owner of his
property, and therefore the defendant would not be guilty of larceny. This is true whether or not
the mistake was reasonable or whether the mistake was attributable to intoxication. In the instant
case, the defendant did not realize that she had taken another’s coat, and thus did not have the
specific intent to commit larceny. (A) is incorrect. A mistake of fact that negates the state of mind
requirement for specific intent crimes need not be reasonable. Any mistake of fact, reasonable or
unreasonable, that negates specific intent would be a defense. For much the same reason, (B) is
incorrect. For specific intent crimes, intoxication may be a defense when it prevents the defendant
from formulating the requisite state of mind. (C) is incorrect because it implies that the asporta-
tion requirement for larceny must be substantial. It need not be; any movement of the property is
sufficient to satisfy the requirement. Here, the defendant walked out of the bar with the fur coat,
which is a sufficient asportation.

Exam Note: Reaching a place of temporary safety may be important in a question involving
killings committed during the course of a felony because it may cut off liability; it has nothing
to do with the asportation requirement for larceny.

Answer to Question 13

(B) The bakery will prevail regardless of when it sues. The farmer’s statement of August 10 that he
would be unable to deliver because he had sold all of his crop to others constitutes a repudia-
tion of the contract, which entitles the bakery to sue at once rather than wait for the September
1 performance date. [See U.C.C. §2-610] The bakery will prevail unless there have been some
unanticipated circumstances that would excuse performance, but a rise in price due to poor crop
performance is not sufficient to excuse performance under the doctrine of commercial imprac-
ticality. The farmer appears to have repudiated merely to make a greater profit. (A) is incor-
rect—because the farmer repudiated, the law does not require the bakery to wait until the due
date for performance to arrive. It may sue at once. [See U.C.C. §2-610] (C) is incorrect because
the facts do not indicate that the farmer has been harmed at all by the price increase, or that he
cannot perform the contract with the bakery. As a matter of fact, he is simply attempting to gain
a windfall by repudiating the contract with the bakery to sell at a higher price. (D) is incorrect
because the facts do not indicate that the circumstances that led to the poor crop were unforeseen.
Crop performance is risky and variable, and a poor crop can certainly result in a price increase

MPQ 100 preview diagnostic exam and divider P.indd 64 12/22/2015 4:13:32 PM
MBE PREVIEW DIAGNOSTIC EXAM 65.

by virtue of the operation of the law of supply and demand. The farmer must be assumed to have
anticipated such possibilities and taken such risks.

Answer to Question 14

(A) The second partner has title free and clear of the mortgage. When the partners bought the
property, they took title as joint tenants with right of survivorship. If the joint tenancy continued
until the first partner’s death, then the property would pass immediately on death to the second
partner. Because the second partner did not sign the mortgage, she would not be subject to it,
regardless of whether she knew about it. The key to answering this question is to know whether
execution of the mortgage by the first partner caused a severance of the joint tenancy. If it did
cause a severance, then the first partner’s one-half would not pass to the second under right of
survivorship but instead would pass to the first’s estate, and thus would go to the daughter by
will. Whether a mortgage creates a severance or not depends on whether the state follows the
lien theory or the title theory of mortgages. Lien theory means no severance; title theory means
severance. Because this is a lien theory state (majority rule on the MBE), there was no sever-
ance; thus, the joint tenancy remained intact. On the first partner’s death, the joint tenancy ended
and the first partner’s interest instantly passed to the second partner. The first partner’s estate got
nothing; hence, the daughter could get nothing. (B) would be the correct answer if the execution
of the mortgage had created a severance of the joint tenancy. The severance would have changed
the joint tenancy to a tenancy in common. The second partner would keep her one-half, free of
the mortgage she did not sign, and the daughter would inherit the first partner’s one-half, subject
to the mortgage the first partner signed. Because we are told this is a lien theory state, there was
no severance when the mortgage was executed so the joint tenancy remained intact until the first
partner’s death. (C) is incorrect. Not only is the joint tenancy unsevered so that it remains intact
to give title to the entire property to the second partner, but also under no circumstances could the
second partner be held liable for a mortgage she did not enter into. (D) is incorrect. When the first
partner died, the property passed free and clear to the surviving joint tenant. The mortgage signed
by the first partner did not sever the joint tenancy because this is a lien theory state. The mortgage
can be held only against the property the first partner has; when the first partner died, the right of
survivorship operated to end the first partner’s interest and automatically vested it in the survivor.

Answer to Question 15

(D) The drug dealer’s motion should be denied. The defense of double jeopardy does not apply if the
second crime requires an element which is not an element of the first crime and vice versa. In
this case, the crime of conspiracy has an element which is not in the possession crime, namely
the agreement to commit the crime with another person, and it is lacking an element, namely the
actual commission of the crime. (A) is incorrect because it is possible to commit two separate
crimes by the same criminal conduct, such as conspiracy and the substantive offense. (B) is incor-
rect because it is the Double Jeopardy Clause of the Bill of Rights that affords protection against
a subsequent prosecution, not the Due Process Clause. (C) is incorrect because even if a lesser
included offense to the first crime were contained in a separate statute, the defense of double
jeopardy would still apply.

Answer to Question 16

(C) The testimony is inadmissible on public policy grounds. This question involves the admissibility
of settlement offers (“I’ll settle for a refund of the purchase price plus $50,000”) and statements
made in the context of settlement negotiations (“Well, maybe it wasn’t fraud”). Both settlement

MPQ 100 preview diagnostic exam and divider P.indd 65 12/22/2015 4:13:32 PM
66. MBE PREVIEW DIAGNOSTIC EXAM

offers and statements made during settlement negotiations are inadmissible. [Fed. R. Evid. 408]
This is because public policy favors the voluntary settlement of disputes. If a settlement offer
has been made in a case that ultimately goes to trial, admission of the offer into evidence would
probably prejudice the party who made the offer. Thus, if such evidence were admissible, there
would be a disincentive to parties making settlement offers. Similarly, if statements made during
settlement negotiations could be admitted into evidence in cases that ultimately go to trial, then
that would be a disincentive to enter into settlement negotiations or to discuss settlement in a
freewheeling way. In either event, the public policy favoring settlements bars use of such state-
ments. (A), which asserts the offered testimony is admissible as a statement by a party-opponent
(to show plaintiff’s recognition of the weakness of his case), is plainly incorrect. Because of the
public policy valuing settlements, this type of testimony would not be admissible. (B) is incorrect
because it is only half true. Offers to compromise are excluded, as (B) says, but (B) is incorrect
in asserting that statements made in the course of settlement negotiations are admissible where
relevant. (D) is incorrect. The offered testimony involves a statement made by the plaintiff to the
defendant, not a statement made by a party to the party’s attorney. Thus, the attorney-client privi-
lege is not involved. Similarly, the attorney work product privilege is inapplicable. A settlement
offer made by a party or an opposing party is not attorney work product. (D)’s assertion—that any
statement made in the office of an attorney is privileged—is obviously incorrect.

Answer to Question 17

(C) The woman would not prevail in this lawsuit. The Supreme Court consistently has held that the
government may restrict aspects of citizenship such as voting and, here, jury service to citizens.
In fact, the Court has held that laws discriminating against aliens that are related to attributes
of citizenship need only meet a rational basis test. (A) is incorrect because the discrimination is
constitutional as long as it is rationally related to a legitimate government purpose. Restricting
jury service to citizens would be regarded as such a purpose. (B) is incorrect because, although
alienage is generally a suspect classification, only a rational basis test is used for discrimination
related to aspects of citizenship. (D) is incorrect because she could claim an injury based on the
fact that she was discriminated against. Those who claim a violation of equal protection of the law
in that they have been discriminated against have standing to sue.

Answer to Question 18

(B) The contractor may remove the action. A defendant may remove an action that could have origi-
nally been brought in the federal courts (because subject matter jurisdiction based on either a
federal question or on diversity of citizenship was present). Diversity of citizenship jurisdiction is
available when (i) there is complete diversity of citizenship, meaning that each plaintiff is a citizen
of a different state from every defendant; and (ii) the amount in controversy exceeds $75,000.
Generally speaking, a defendant has 30 days from the date he receives the initial summons or
complaint to remove a case. However, if a case becomes removable on the basis of diversity at
a later date, he has 30 days to remove the case from the date the defendant is served with the
document that first makes the case removable. That said, for cases based on diversity, removal
may not take place more than one year after the case was filed. In the instant case, the homebuyer
(from State A) initially sued the agent (also from State A) and the contractor (from State B) for
$150,000. Thus, the case was not initially removable. The case then became removable when the
homebuyer dismissed the agent from the case, leaving the State B corporation as the sole defen-
dant. At this point, the corporation has 30 days to remove the case, and the one-year restriction
does not come into play because the facts state that only six months have passed since the case
was filed. As a result, (B) is the correct answer. (A) is incorrect because the case was not initially

MPQ 100 preview diagnostic exam and divider P.indd 66 12/22/2015 4:13:32 PM
MBE PREVIEW DIAGNOSTIC EXAM 67.

removable. The claim was for a state law breach of contract, so no federal question was presented,
and complete diversity was initially lacking, so diversity of citizenship jurisdiction was not avail-
able. (C) is incorrect because the 30-day period has been met (since the corporation immediately
filed a notice of removal), and only six months have passed since the case was filed in state court,
making the case removable on the basis of diversity of citizenship jurisdiction. (D) is an incor-
rect statement of the law. The grounds for removal need not exist at the time the case is filed. If
grounds for removal come up later, the case may still be removed, subject to certain restrictions.

Answer to Question 19

(C) The hiker will not prevail because strict liability does not apply to a bull, which is a domestic
animal. The owner of a domestic animal, including a farm animal, is not strictly liable for injuries
it causes, as long as the owner has no knowledge that the animal has abnormally dangerous
propensities (i.e., propensities more dangerous than normal for that species). A bull is a domestic
animal, and nothing in the facts suggests that the bull was more dangerous than normal for
that type of animal. Hence, strict liability will not apply. (A) is incorrect because the rule for
trespassing animals does not apply. The owner of a trespassing animal is strictly liable for harm
done by the trespass as long as it was reasonably foreseeable. Here, the bolt of lightning caused
the fence to break and allowed the bull to escape. This unforeseeable intervening force was the
cause of the trespass; hence, the strict liability rule for trespassing animals does not apply here.
(B) is incorrect because, as discussed above, strict liability does not apply for domestic animals
with normal dangerous propensities. Only domestic animals with propensities more dangerous
than normal for the species may subject the owner to strict liability. (D) is incorrect because the
hiker’s status as a trespasser on the neighbor’s land is irrelevant as to the farmer’s liability. If the
hiker were a trespasser on the farmer’s land, strict liability would not apply even if the bull were
abnormally dangerous, but the farmer’s liability is not affected by the hiker’s status as to the
neighbor. Note that if strict liability applied for harm from an animal trespassing on a neighbor’s
property, the hiker’s status as a trespasser might be relevant because strict liability applies only
to injured persons who were rightfully on the property. However, as discussed above in (A), that
liability is inapplicable here because the bull’s trespass was unforeseeable.

Answer to Question 20

(C) The witness’s testimony would clearly qualify as hearsay. Because there is no applicable excep-
tion to the hearsay rule, his testimony is inadmissible. (A) is incorrect because the statement was
not made during and in furtherance of the conspiracy, a requirement for admitting statements by
a co-conspirator. (B) is incorrect because the woman has already been acquitted of the crime in
question; hence, she is not subject to further prosecution. (D) is an incorrect statement of law.

Answer to Question 21

(B) An assault is committed when the victim is put in reasonable apprehension of an offensive
touching. If the farmer believed that the melon grower might shoot him, and the belief was objec-
tively reasonable, then the melon grower’s action amounts to assault. The conditional threat, “stop
or I’ll shoot,” is sufficient for an assault. Because the melon grower was pointing a rifle at the
farmer at the time the threat was made, the requirement of immediacy was fulfilled. The melon
grower cannot claim defense of property because the farmer had a privilege based on private
necessity to enter the melon grower’s land. (B) is a better answer than (A), because the melon
grower would only have committed an assault if the farmer had a reasonable apprehension of a
harmful or offensive contact. (C) is a misstatement of the law. Assault is not a lesser included tort

MPQ 100 preview diagnostic exam and divider P.indd 67 12/22/2015 4:13:32 PM
68. MBE PREVIEW DIAGNOSTIC EXAM

within battery. (D) is incorrect because what is contained in the shotgun is irrelevant. The gun
created apprehension of a harmful or offensive contact.

Answer to Question 22

(D) The governor’s best argument is based on separation of powers principles. Under Article II,
Section 2 of the Constitution, the President has the power to “grant reprieves and pardon offenses
against the United States, except in cases of impeachments.” Thus, the President’s pardon power
is limited to violations of federal law. The President has no authority to pardon those convicted
of state crimes or held in state custody. Thus, the governor is correct. The President lacks the
authority to grant these pardons. Although the President’s ability to negotiate treaties only extends
to dealing with other nations, (A) is wrong because the President possesses other powers that
would allow him to negotiate such deals. For example, the President’s powers as Commander in
Chief and other inherent powers (such as the ability to negotiate executive agreements) would
justify this action. (B) is wrong because, under the doctrine of federal supremacy, federal law will
govern state law when there is an inconsistency between the federal and the state laws. Therefore,
the governor could be required to do an act that is a violation of state law, but a requirement of
federal law. (C) is wrong because the President has the power to grant pardons to those convicted
of violations of federal laws. The President’s exercise of this authority is part of his faithful execu-
tion of the laws of the United States.

Answer to Question 23

(B) The Court should hear the case on the merits. A state can sue another state to protect its natural
resources for the benefit of its own citizens. [Pennsylvania v. West Virginia (1923)] The ice
cutter’s state is suing the state operating the power plant under this doctrine. Under Article III,
the Supreme Court has original and exclusive jurisdiction over controversies between two states.
Because this is a controversy between two states, Supreme Court jurisdiction is proper. (A) is
incorrect. The ice cutter’s state is a real party in interest, and the Eleventh Amendment, which
bars suits by citizens of one state against another state in federal court, is inapplicable. (C) is
incorrect. Article III, Section 2(2) of the Constitution confers exclusive original jurisdiction to
the Supreme Court over all controversies between two or more states. (D) is incorrect. Article III,
Section 2 of the Constitution confers exclusive original jurisdiction on the Supreme Court in cases
between two or more states. In such cases, the Court must serve as a trial court and even Congress
may not limit that jurisdiction. [See Marbury v. Madison (1803)]

Answer to Question 24

(C) The doctor can sue the homeowner but can recover only $86,000. The homeowner gave a
warranty of title to the accountant when he gave her a general warranty deed. This warranty runs
with the land and can be enforced by any subsequent purchaser. Damages are limited, however, to
the purchase price received by the warrantor. Because the homeowner received only $86,000 from
the accountant, that is all that the doctor can recover. The doctor gets nothing from the accountant
because she gave only a quitclaim deed, which gives no warranty of title. (A) is incorrect because
the doctor cannot prevail over the accountant at all, and he can recover only $86,000 against
the homeowner. (B) is incorrect. While the doctor can win over the homeowner and not the
accountant, he is limited to the money the homeowner received from the accountant—$86,000.
(D) is incorrect. Because the homeowner gave a general warranty deed, that warranty for title
runs with the land and can be enforced by any subsequent purchaser. There are two (sometimes
three) future covenants contained in a general warranty deed—the covenants of quiet enjoyment,

MPQ 100 preview diagnostic exam and divider P.indd 68 12/22/2015 4:13:32 PM
MBE PREVIEW DIAGNOSTIC EXAM 69.

warranty, and (sometimes) further assurances. Quiet enjoyment is a covenant that the grantee will
not be disturbed in her possession or enjoyment of the property by a third party’s lawful claim
of title. Warranty is a covenant that the grantor agrees to defend on behalf of the grantee any
lawful or reasonable claims of title by a third party. Further assurances is a covenant to perform
whatever acts are reasonably necessary to perfect the title conveyed if it turns out to be imperfect.
These three covenants are future covenants that are breached, if at all, only on interference with
the possession of the grantee or her successors; thus, they “run” with the grantee’s estate. Here,
the future covenants contained in the accountant’s deed from the homeowner run with the land,
allowing the doctor to seek recourse against the homeowner, the original grantor, for the distur-
bance in possession.

Answer to Question 25

(C) The testimony should be found inadmissible. Extrinsic evidence of a prior inconsistent statement
may not be used to impeach a witness upon a collateral matter. The clerk testified that he could
recognize the defendant’s face, and so the color of the gun is not material to any issue in the case
under the facts given. (A) and (B) are incorrect because, even if the evidence does have some
bearing on the clerk’s credibility as a witness, it will be excluded because it will possibly confuse
the issues or because it is a waste of time. (D) is not correct because the defendant is not seeking
evidence to prove the truth of any material issue, but to impeach the clerk. The hearsay rule does
not apply.

Answer to Question 26

(C) If the lodge owner prevails, it will be because he made a reasonable inspection without discov-
ering the danger. In a jurisdiction applying traditional rules for landowners and possessors of
land, a landowner owes a duty to invitees to warn of or otherwise make safe dangerous natural
conditions of which he knows or could reasonably discover by inspection. (A) is wrong because
the fact that the tree limb was a natural condition does not relieve the lodge owner of this duty.
(B) is wrong because actual knowledge is not necessary; it is sufficient to impose liability if the
dangerous natural condition would have been discovered by a reasonable inspection. (D) is wrong
because the fact that other factors contributed to the danger does not negate the landowner’s
liability because those factors were foreseeable.

Answer to Question 27

(A) The best basis for recovery is a battery action. The facts in this case indicate that the actions of
the Guard were intentional. The facts also state that it was not necessary to use weapons. If it was
not necessary to use weapons, it would follow that the Guard had no privilege to use deadly force,
and the state would be liable to the strikers for battery. (C) is wrong because this is clearly an
intentional tort. Both (D), strict liability, and (B), abnormally dangerous activities, are torts based
on unintentional injuries, so even if the plaintiffs could apply these torts to this action, it would
not be the best basis for recovery since, as stated above, this is an intentional tort.

Answer to Question 28

(C) The hotelier will prevail. Ice buckets are movable goods; therefore, Article 2 of the U.C.C.
applies. The June 8 letter from the supply company is a firm offer under U.C.C. section 2-205.
No consideration is required, because the company is a “merchant” (i.e., one who ordinarily
deals in goods of the kind sold) of ice buckets. Where a time period for the offer is stated, the

MPQ 100 preview diagnostic exam and divider P.indd 69 12/22/2015 4:13:32 PM
70. MBE PREVIEW DIAGNOSTIC EXAM

period of irrevocability is that period, except that the period cannot exceed three months. Here,
the three-month period would end on September 8. The company’s fax stating that it had only 50
ice buckets left to sell constitutes an invalid attempt at revocation, because it is within the three-
month period of irrevocability. (A) is incorrect because section 2-205 does not require that the
offeree of a firm offer be a merchant; it requires that the offeror be a merchant, and the company
is (see above). (B) is incorrect because a firm offer that states a period longer than three months
is still firm for the first three months. (D) is incorrect because the hotelier’s knowledge, or lack
thereof, of the “revocation” of the company’s offer is irrelevant because it was invalid; the fact that
the company made a firm offer prevents it from revoking the offer within the stated time, not to
exceed three months.

Answer to Question 29

(D) The developer must accept the 24 sets and is not entitled to cancel the rest of the contract. The
ground for rejection of an installment contract is U.C.C. section 2-612(2), which provides that the
buyer may reject any installment that is nonconforming only if it substantially impairs the value
of that installment and cannot be cured. If the nonconformity can be cured and the seller gives
adequate assurances of its cure, the shipment must be accepted by the buyer, provided that the defect
is not such as to constitute a breach of the whole contract. (A) is incorrect. Under U.C.C. section
2-601, if defective goods arrive, the buyer normally has every right to reject them as unsatisfactory.
But this is not the case if the defective shipment was part of an installment contract, as is the case
here. (B) is incorrect for the reasons stated above. (C) is incorrect. Under U.C.C. section 2-612(3),
there is a breach of the entire contract only when the nonconformity or default with respect to a
single installment substantially impairs the value of the whole contract. Here, this is clearly not so,
and the developer may not repudiate the second shipment based upon the single incomplete set.

Answer to Question 30

(D) None of these people would be guilty of attempted murder because, on these facts, they did not
possess the specific intent to kill necessary for the crime of attempted murder. At common law,
all attempts were specific intent crimes. Thus, before a criminal conviction for attempt would lie,
it had to be established that the actor had the specific intent to engage in the behavior or to cause
the harm prohibited by the criminal statute that the actor was charged with attempting to violate.
Thus, before someone could be guilty of attempted murder, he had to possess the specific intent to
kill. In this question, it is clear that the mechanic and the doctor did not have the specific intent to
kill. Their intent was only to place a smoke bomb on the owner’s porch. The situation with respect
to the army veteran’s intent is not quite as clear, but a fair reading of the facts indicates that he
did not have a specific intent to kill. It is true that he made a bomb containing a lethal quantity
of explosives, but the question goes on to state that he only intended to scare the slovenly owner.
Had someone died in the blast, the army veteran could be found guilty of manslaughter due to
his recklessness, or perhaps he could be found guilty of murder under the depraved heart theory.
However, since no one died, he will not be guilty of attempted murder because there was no
specific intent to kill. Thus (D) is correct, and (A), (B), and (C) are wrong.

Answer to Question 31

(A) The judge should overrule the objection because the statement falls within the exception for state-
ments of present state of mind. Under that exception, declarations of existing state of mind are
admissible as circumstantial evidence to show subsequent acts of the declarant in conformity with
that intent. Here, an issue in this case is whether the cousin confronted the defendant at his office,

MPQ 100 preview diagnostic exam and divider P.indd 70 12/22/2015 4:13:32 PM
MBE PREVIEW DIAGNOSTIC EXAM 71.

as the prosecution alleges. His statement that he was intending to do so is circumstantial evidence
that he did so. It should therefore be admissible under the present state of mind exception. (B) is
incorrect. While the cousin’s confrontation with the defendant may have triggered the murder,
the prosecution is offering the statement in the letter for its truth (i.e., that the cousin planned to
confront the defendant and subsequently did so), in large part to rebut the defendant’s assertion
that he had not seen the cousin in several weeks. (C) is incorrect because, as discussed above, the
statement falls within the hearsay exception for statements of present state of mind. (D) is incor-
rect because, while the cousin’s state of mind is not directly in issue, it is relevant to show that he
acted in conformity with that state of mind.

Answer to Question 32
(A) The court should deny the motion. The question provides that service was made pursuant to the
State A long arm statute. The assertion of personal jurisdiction is statutorily authorized, in that it
is consistent with State A’s long arm statute, and the exercise of personal jurisdiction is constitu-
tional, given that the claim arises from the defendant’s purposeful activities (driving in and using
roads) in State A. Thus, a court in State A may properly exercise personal jurisdiction over the
defendant from State B, even if the defendant does not consent to personal jurisdiction and is not
domiciled there. (B) is incorrect because personal jurisdiction cannot be exercised against a defen-
dant based on the plaintiff’s domicile. (C) is incorrect, as the answer choice describes the exercise
of general jurisdiction (i.e., personal jurisdiction over the defendant for all causes of action). The
facts do not indicate that the defendant has such contacts with State A. (D) is incorrect because
consent is not necessary to assert personal jurisdiction over a defendant, as explained above.
Consent is one basis for exercising personal jurisdiction over a defendant, but not the only basis.

Answer to Question 33
(D) Their request for specific performance will be denied. With a future interests question that may
raise the Rule Against Perpetuities, you must first label the interests granted. The landowner
attempted to give a fee simple determinable to his sister (“to my sister and her heirs so long as
it is used for residential purposes”) followed by an executory interest in the community center
(“but if it is ever used for other than residential purposes, then to the local community center”).
If the grant to the community center is valid, the landowner would have retained nothing. But the
grant over to the community center is void under the Rule Against Perpetuities. It will not vest
until the property is used for nonresidential purposes, an event that could occur outside lives in
being plus 21 years. If the interest in the community center is void, that leaves only the sister’s fee
simple determinable. That means the landowner must have kept a possibility of reverter, which
always accompanies a fee simple determinable that is not followed by an executory interest. The
landowner’s possibility of reverter was devised to the friend under the landowner’s will. Thus,
both the sister and the friend have interests in the property, and the failure of the friend to join
in the contract means no specific performance of it. (A) is incorrect. The sister has a fee simple
determinable. When the Rule Against Perpetuities voided the attempted executory interest in the
community center, it struck all the language following the comma: “but if . . .”; that left “to my
sister and her heirs so long as it is used for residential purposes.” The fee simple determinable is
not a fee simple absolute; it is accompanied by the possibility of reverter in the grantor, and that
person (or his successor) must join in the conveyance for the grantee to get a fee simple absolute.
(B) is incorrect. The daughter owns nothing; the landowner devised his real property interests to
the friend, so his heir takes none of it. The sister has a fee simple determinable, not a fee simple
absolute. (C) is incorrect. The Rule Against Perpetuities voided the attempted executory interest
in the local community center; it has nothing.

MPQ 100 preview diagnostic exam and divider P.indd 71 12/22/2015 4:13:32 PM
72. MBE PREVIEW DIAGNOSTIC EXAM

Answer to Question 34
(C) The defendant does not need to produce the passenger’s statement. Work product prepared in
anticipation of litigation is discoverable only on a showing of substantial need and undue hardship
in obtaining the substantial equivalent of the work product. Here, there is no indication that the
passenger is unavailable or cannot recall the accident. Thus, it is unlikely that the plaintiff will
be able to show substantial need and undue hardship in obtaining a statement from the passenger
as to her recollection of the accident. As a result, the statement is protected from discovery.
(Note that this analysis is applicable to the statement only; the existence of the passenger as a
witness must be disclosed either as an initial disclosure—assuming the defendant is going to use
passenger to support her claim or defense—or in response to a properly submitted interrogatory.)
(A) is an overbroad statement of the requirements for discovery. Federal Rule 26(b)(3) specifically
exempts documents prepared in the anticipation of litigation from discovery, and the passenger’s
statement falls into this category. (B) is an incorrect statement of the law; the fact that the state-
ment was electronic does not prevent it from becoming protected under the work product doctrine.
(D) is incorrect because the statement is not “privileged” per se (such as the doctor-patient
evidentiary privilege), but rather is exempt from discovery under the work product doctrine under
Federal Rule 26(b)(3). As described above, there are exceptions to the work product doctrine.
Answer to Question 35
(B) The teen’s motion should be granted. A search is valid if the police reasonably believe that they
obtained valid consent for the search. Generally, a homeowner has authority to consent to a
search of her own home, as long as she has apparent access to the place searched. Here, nothing
indicates that the teen’s bedroom door was locked. Thus, the mother had apparent authority to
search the teen’s room. However, it appears unreasonable for the police to believe that the mother
had authority to consent to a search of the suitcase because it was locked, and the police had to
break the lock to open it (indicating that the mother did not have a key for the lock and so likely
did not have access to the contents of the suitcase). The parent-child relationship does nothing to
change this analysis. A parent of an adult teenager does not have automatic authority to search
the adult teen’s locked things (the rule probably is different for young children). (A) is incorrect.
To be effective, a consent to search must be voluntary and given under no threat or compulsion.
However, the police are under no obligation to inform the defendant or other person of the right
to refuse entry. Therefore, this search is not invalid because the mother was not informed of her
right to refuse entry. The issue is whether her consent was sufficient to allow the police to search
the son’s suitcase. (C) is incorrect because, as explained above, a homeowner does not necessarily
have authority to consent to a search of everything in her home. Where it is apparent that she does
not have access (e.g., when something is locked and she does not have a key), the police may not
reasonably believe she may consent to the search of the locked area. (D) is incorrect. The doctrine
of in loco parentis is a tort doctrine under which the state is charged with a parent’s responsibili-
ties over a child in its care. The doctrine does not apply here.
Answer to Question 36
(D) The passenger should be acquitted. Robbery is larceny from a person by violence or intimidation.
Larceny requires a specific intent to take personal property from the possession of another with
intent to permanently deprive the other of his possessory interest. Because the passenger thought
that the property belonged to him, his mistake of fact is a valid defense to the crimes of larceny and
robbery. This is because the mistake negates the existence of a mental state (specific intent to steal)
essential to the charged crime. (C) is incorrect. Robbery is larceny by either violence or intimidation.
Knocking the owner to the floor constitutes the necessary violence, whether or not the passenger

MPQ 100 preview diagnostic exam and divider P.indd 72 12/22/2015 4:13:32 PM
MBE PREVIEW DIAGNOSTIC EXAM 73.

used intimidation. Even though voluntary intoxication may be a defense to a specific intent crime
(in those circumstances where the intoxication negates the specific intent), the better answer is (D),
because the passenger’s mistake of fact is a defense to the specific intent crime charged since he did
not have the specific intent to steal. (A) is incorrect. As discussed above, although voluntary intoxi-
cation can be a defense to a specific intent crime, (D) is the better answer because the question does
not establish that the mistake of fact was a product of the intoxication. (B) is incorrect. The passen-
ger’s mistake of fact negates the specific intent to steal required to convict him of robbery.

Answer to Question 37
(B) The gardener is entitled to receive the fee. The common law rule is that a promise to modify a
contract must be supported by consideration. Here, the gardener agreed to work an extra year in
return for the landowner’s promise to pay extra money. In the absence of facts indicating duress
or unconscionability (facts not present here), courts will not inquire into the adequacy of the
consideration. There is no indication that the landowner had no “meaningful choice” but to accept
the price increase, and because the landowner received a year’s extension in exchange, there was
adequate consideration to support a modification. (A) is incorrect because, as discussed above,
the modification is valid and the price increase can take effect as agreed. (C) is incorrect for two
reasons: First, economic duress generally is not a good defense to contract unless caused by the
party seeking to enforce the contract. Second, as mentioned above, there are no facts indicating
that the landowner had no “meaningful choice” but to accept the proposed modification; nothing
indicates that the landowner could not have found another gardener or could not have sued the
gardener for breach if she had not performed the original contract in the absence of modification.
(D) is incorrect because the gardener did not merely promise to perform a preexisting duty in
exchange for the promise to pay $200 a month more. She promised a year’s extension of services,
and this is additional and valuable consideration.

Answer to Question 38
(A) The federal court does not have jurisdiction because neither diversity of citizenship jurisdiction
nor federal question jurisdiction exists. Diversity of citizenship jurisdiction is available when (i)
there is complete diversity of citizenship, meaning that each plaintiff is a citizen of a different
state from every defendant; and (ii) the amount in controversy exceeds $75,000. A natural
person’s citizenship is the state that is the person’s domicile. A partnership is a citizen of each
state of which its partners, both limited and general, are citizens. Here, the writer is a citizen of
State A, and the advertiser’s partners are citizens of State A, State B, and State C. Given the State
A-State A connection, complete diversity does not exist. Hence, (B) is an incorrect answer choice
as to diversity jurisdiction. Federal question jurisdiction is available when the plaintiff, in his well-
pleaded complaint, alleges a claim that arises under federal law. Anticipation of a federal defense
or the fact that federal law is implicated by the plaintiff’s claim do not give rise to federal question
jurisdiction; the plaintiff’s claim must arise under federal law. Here, although federal copyright
law is peripherally involved, the writer’s cause of action is actually based on state contract law. As
a result, no federal question has been presented by the writer’s complaint, making (C) incorrect.
Note too that federal question jurisdiction does not have a complete diversity requirement, making
(B) incorrect as to federal question jurisdiction. (D) is incorrect because federal question jurisdic-
tion does not arise merely because interstate commerce is affected.

Answer to Question 39
(A) The law should be held constitutional. Congress has very broad power over interstate commerce,
and it may enact laws that regulate commerce. (B) is a misstatement of the law, because Congress

MPQ 100 preview diagnostic exam and divider P.indd 73 12/22/2015 4:13:32 PM
74. MBE PREVIEW DIAGNOSTIC EXAM

is not required to find that there exists some burden on interstate commerce before it enacts
laws regulating commerce. (C) is wrong because it is Congress that is empowered to regulate
commerce between the states. Current state laws have no effect on whether Congress can, or
cannot, regulate. (D) is wrong because the federal statute did not reverse the decision of the
Supreme Court. It merely enacted a requirement as a matter of federal law, while the Supreme
Court had ruled that a state law on the subject was invalid.

Answer to Question 40
(A) If the son can prove that “most doctors” would not have used this drug to treat a pregnant woman,
then he has probably shown that the doctor has failed to comply with the required standard of
care. The mother’s doctor owes a duty to both mother and child not to act in a negligent manner.
If a family practitioner practicing according to the standard of care required in this jurisdiction
would not have treated an obviously pregnant woman with the drug, then the doctor’s treatment of
the pregnant woman was negligent. (B) is incorrect. The doctor can only prevail if he can prove
that treating the pregnant woman with the drug was in accord with prevailing medical procedure
as practiced by a physician of ordinary care and skill. The fact that the doctor was not personally
aware that the treatment he prescribed would cause injury will not allow him to avoid liability if
his conduct was not in accord with the standard of care required of doctors in this jurisdiction. If
he needed to read certain journals to acquire the knowledge and skill of a competent physician,
then failure to do so is failure to exercise reasonable care. (C) is incorrect because the “learned
intermediary” rule would protect the pharmaceutical company from being sued for failure to warn
the mother about the risks (because it provided a warning to the prescribing physician); it does not
affect the doctor’s liability. (D) is incorrect because a fetus who is born alive has a cause of action
against those who negligently cause prenatal injuries.

Answer to Question 41
(A) The best defense is that First Amendment rights do not apply to the organization. First
Amendment rights are made enforceable against a state through the Fourteenth Amendment.
However, the amendments do not prevent private individuals or organizations from encroaching
upon individual civil liberties. Because there is nothing in the facts to indicate significant state
involvement (state action) with the organization (granting corporate status does not suffice), the
amendments do not apply to their actions in expelling one of their members. (B) is incorrect
because such membership may be protected by the First and Fourteenth Amendments. (C) and
(D) would have no bearing on the alleged violation of the member’s rights.
Answer to Question 42
(D) The written agreement is enforceable in its entirety. The initial contract had two components: the
sale of the smaller company’s refining facilities to the oil producer, and the oil producer’s option
to purchase the smaller company’s output. Normally, the option provision would be illusory,
because the oil producer’s promise to purchase, at its option, 100% of the smaller company’s
output is not a promise to do anything and therefore is not consideration. However, there is
other consideration here—the oil producer’s purchase of the smaller company’s refining facili-
ties. The contract must be considered as a whole, and here the oil producer promised to purchase
the smaller company’s refining facilities. Moreover, the specification of the quantity of 100% of
output is sufficiently certain and can be enforced because the quantity of output can be estimated
from prior output. The specification of the price of “current market prices” is also sufficient. Thus,
the written agreement is enforceable in its entirety, and (A), (B), and (C) are incorrect.

MPQ 100 preview diagnostic exam and divider P.indd 74 12/22/2015 4:13:32 PM
MBE PREVIEW DIAGNOSTIC EXAM 75.

Answer to Question 43
(C) The stockbroker and developer would prevail. The rights of the newlyweds, as intended benefi-
ciaries, would have vested if they had learned of the contract made for their benefit and either
accepted it at the request of the contracting parties or changed their position in reliance on it. In
this case, they did neither before the donors rescinded the contract, and therefore they have no
right to enforce it. (A) is incorrect. This choice correctly states that the newlyweds are intended
third-party beneficiaries of the contract made by their fathers. However, their rights do not vest as
of the formation of the contract. Rather, they must at least assent to the contract at the request of
the contracting parties or must change their position in reliance on the contract. Thus, rescission
of the contract by the stockbroker and the developer was sufficient to terminate the rights of the
newlyweds. (B) is incorrect. However, their rights do not vest merely because they have received
notice of the contract. They must either accept the contract at the request of the contracting parties
or must substantially rely on the contract to their detriment before they would be able to prevent
the original contracting parties from rescinding the contract. (D) is incorrect. Because the parties
to the contract contemplated that the newlyweds would receive a benefit under the terms of the
agreement, the newlyweds were intended, and not incidental, beneficiaries.
Answer to Question 44
(D) The testimony is admissible. The best evidence rule requires the production of the original
document when attempting to prove the contents of the document. [Fed. R. Evid. 1002] The
director of the tool producer’s order department seeks to testify that the price of the hardware
store’s order was $5,500. It appears that this testimony is based solely on the director having seen
the standard tool producer’s form on which the hardware store placed their order. Thus, the testi-
mony is about the contents of a document—the tool producer’s form that the store owner completed
to order hammers. At first glance, therefore, it would appear that (B) is the correct answer, that the
director’s testimony would be inadmissible because of the best evidence rule. However, that rule
does not require production of the original if the original has been lost or destroyed in good faith.
[Fed. R. Evid. 104(1)] It appears that the order form was destroyed in good faith in a fire at the tool
producer’s headquarters. Thus, (B) is incorrect. (C) is also incorrect. If the original of a document
cannot be produced to prove its contents, any form of secondary evidence can be substituted
for the original. There is no requirement, for example, that a copy of the original be produced.
Testimony as to the contents of the document is equally permissible. From the standpoint of the
best evidence rule, then, (D) is the correct answer. The proposed testimony of the director of the
tool producer’s order department is admissible. (A) is incorrect because it confuses the question of
admissibility of evidence with that of the weight that a fact finder should give to the evidence. That
the proposed testimony is self-serving may render it unpersuasive, but not inadmissible. Also, if
the self-serving nature of testimony rendered it inadmissible, then virtually any time a party testi-
fied in support of his proposition, the testimony would have to be excluded. The law of evidence
has for centuries rejected such a broad rule requiring disqualification of witnesses. In addition, (A)
states that the proposed testimony of the director of the tool producer’s order department is untrust-
worthy. There is no factual basis in the question for reaching that conclusion.

Answer to Question 45
(D) The defendant may plead both defenses regardless of how they are labeled. The Federal Rules
of Civil Procedure expressly permit inconsistent pleadings in the alternative or hypothetically,
reasoning that the discovery process will operate to sort out the viable pleadings. For this reason,
(A) and (B) are incorrect. (C) is incorrect because the Rules do not limit inconsistent claims or
defenses to “affirmative” defenses.

MPQ 100 preview diagnostic exam and divider P.indd 75 12/22/2015 4:13:32 PM
76. MBE PREVIEW DIAGNOSTIC EXAM

Answer to Question 46

(D) The imperfect self-defense doctrine applies to these facts. In states that apply the doctrine, murder
is reduced to voluntary manslaughter when the defendant unreasonably, but honestly, believed
in the necessity of responding with deadly force. In the instant case, the performer unreason-
ably, but apparently honestly, believed that the pedestrian would kill him. Thus, the performer
would be guilty of voluntary manslaughter in such a jurisdiction. (A) is wrong. In order to have
murder reduced to voluntary manslaughter based on a “heat of passion” theory, it must be shown
that: (i) the provocation must have been one that would arouse sudden and intense passion in the
mind of an ordinary person such as to cause him to lose self-control; (ii) the defendant was in
fact provoked; (iii) there must not have been sufficient time to cool off; and (iv) the defendant
did not in fact cool off. Adequate provocation generally consists of being subjected to a serious
battery or threat of deadly force, or of discovering one’s spouse in bed with another person. In the
instant case, not only is it unlikely that a slap on the face would be considered a serious battery
such that would provoke one to murder, the facts do not indicate that the performer acted in a fit
of rage. (B) is wrong because there is no evidence of any diminution of the defendant’s ability to
reason. (C) is wrong. Although some states would call a killing committed during a misdemeanor
“manslaughter,” in the instant case there are no facts to indicate that the performer was in the
process of committing a misdemeanor when he killed the pedestrian.

Answer to Question 47

(B) The strongest argument is based on merger. When the cousin bought both parcels from the owner
and the buyer, the dominant (buyer) and servient (owner) estates came together in the same owner
and the easement was terminated by merger. Once terminated by merger, the easement does
not revive when the ownership is later split. (A) is incorrect. An easement may be terminated
by abandonment, but mere nonuse is not enough. The holder of the dominant estate must physi-
cally manifest an intent to abandon. The facts do not show that any holder of the western parcel
did anything to indicate this intent. (C) is incorrect. The easement granted in the deed from the
owner to the buyer was a valid express easement. No implied easement arose because there was
no necessity for one. The later construction of the public road can have no effect if there had been
no valid implied easement in the first place. (D) is incorrect. The fact that the easement was not
included in the subsequent deeds is irrelevant because the easement was in the owner-buyer deed,
which was recorded properly in the chain of title. That puts all subsequent purchasers on notice of
the easement, and it need not be put in subsequent deeds to be given effect.

Exam Tip: Multiple transfers can be confusing on the exam. A technique to aid analysis is
to make a quick chart in the margin of the exam booklet. This would have clearly shown the
merger and unity of ownership:

West East
Owner Owner
Buyer Owner

Cousin Cousin

Lawyer Doctor
Dev. Co. Doctor

MPQ 100 preview diagnostic exam and divider P.indd 76 12/22/2015 4:13:32 PM
MBE PREVIEW DIAGNOSTIC EXAM 77.

Answer to Question 48

(D) The neighbor will not prevail. The owner has made an adverse use of the neighbor’s land since
the sewer line was first constructed across it. The fact that the owner tore down the house on his
property does not stop the prescriptive period from running, because the mere existence of the
sewer line was an adverse use and there was no intent to abandon it. Therefore, the owner’s use
ripened into a prescriptive easement 20 years after that use first began. Moreover, the use of the
line to service the new house is within the scope of the easement by prescription because the
same pipe in the same location will be used for the new house. (A) is incorrect. By the time the
owner constructed the new house, he had maintained the sewer line across the neighbor’s property
for the statutory period and therefore had acquired an easement by prescription to continue to
use that line. Once the owner obtained an easement by prescription over the neighbor’s property,
that property right persists despite any actions by the municipal authorities with regard to the
neighbor’s property, or any actions by the owner with regard to the authorities. (B) is incorrect.
An easement is not terminated merely because it is not used for a long period of time. It can only
be extinguished when the owner clearly demonstrates by physical action an intent to permanently
abandon it. Here, abandonment could have arisen if the owner had been able to connect to a new
sewer line, but that did not happen. (C) is incorrect. The owner never had a license. A license
is a permissive use of property. The owner placed the sewer line across the neighbor’s property
without her permission. Moreover, the use never ripened into an implied easement. Such an
easement can only occur when there is a division of a large parcel of property into smaller parcels
and there is a reasonable necessity for the owner of one of the parcels to continue to use rights
over another part of the larger parcel. There is no such division of property here. Instead, the
adverse use by the owner ripened into an easement by prescription.

Answer to Question 49

(B) The best argument is based on the Privileges and Immunities Clause. State laws that discrimi-
nate against out-of-state residents in terms of the right to earn a living trigger the Privileges and
Immunities Clause of Article IV and are almost always unconstitutional. (A) is incorrect. A Bill
of Attainder requires, inter alia, a legislative act which is intended to “punish” an identifiable
group. Punishment is an unlikely motive of the statute. (C) is incorrect. Out-of-state residents are
not considered a protected class under the Fourteenth Amendment Equal Protection Clause. This
would more than likely be a Fourteenth Amendment Due Process Clause violation because the
government may not take a recognized “liberty” or “property” interest of a person without some
procedural due process. (D) is incorrect. Only retroactive criminal laws can violate the ex post
facto clause.

Answer to Question 50

(C) Lack of a proper foundation is the most likely reason for this evidence to be held inadmissible.
No single facet of the law of evidence provides a ready answer to this question. Thus, it is useful
to analyze each choice and arrive at the correct answer by a process of elimination. (A) is incor-
rect because the videotape showing the defendant offering a television set to one of the police
officers is not hearsay evidence. Even if the defendant’s conduct on the videotape is considered
assertive conduct and is thus a “statement” within the meaning of the hearsay rule (much like
nodding one’s head to indicate “yes” is a statement), under the Federal Rules a statement of a
party-opponent is admissible nonhearsay. [Fed. R. Evid. 801(d)(2)] Because the defendant is a
party to this criminal prosecution, his conduct, if considered a statement, would be admissible.
It is not clear, however, that the defendant’s conduct on the videotape is assertive conduct. If it

MPQ 100 preview diagnostic exam and divider P.indd 77 12/22/2015 4:13:32 PM
78. MBE PREVIEW DIAGNOSTIC EXAM

is considered nonassertive conduct—conduct that is not intended to communicate a thought—it


would not be a statement within the meaning of the hearsay rule. [Fed. R. Evid. 80l(a)] Thus, this
is an alternative reason, albeit a less clear one, why (A) is incorrect. (B) is incorrect because the
privilege against self-incrimination applies to compulsory self-incrimination. The defendant was
not forced to incriminate himself. He voluntarily took an action (offering to sell a television set to
one of the police officers) that helps establish that he is guilty of receiving stolen property. Such
an act is not covered by the self-incrimination privilege. (D) is also incorrect because specific
instances of misconduct pertaining to an event (here, the receipt of stolen property) are frequently
used to prove criminality. If (D) said that specific instances of past misconduct pertaining to
a defendant’s character (i.e., propensity evidence) were inadmissible, that would be true as an
abstract statement of law, but (D) does not say that, nor do the facts in the question remotely
relate to propensity evidence. Thus, (A), (B), and (D) are clearly incorrect. (C), on the other
hand, is a plausible answer. If the videotape is held to be inadmissible, one reason could be that
a proper foundation had not been established for its introduction into evidence. A videotape, like
other forms of tangible evidence, must be authenticated before it can be admitted. The failure to
properly authenticate the videotape is one reason why it could have been held inadmissible.

Answer to Question 51

(B) The homeowner is likely to recover from the construction company. The general rule of proximate
cause is that the defendant is liable for all harmful results that are the normal incidents of and
within the increased risk caused by his acts. In indirect cause cases, an independent intervening
force may be foreseeable where the defendant’s negligence increased the risk that these forces
would cause harm to the plaintiff. Even a criminal act by a third party will not cut off the defen-
dant’s liability if the defendant’s negligence created a foreseeable risk that a third person would
commit the crime. Here, the construction company negligently cut the power in a high crime
neighborhood, increasing the risk of criminal conduct occurring. But for the power being cut,
the homeowner’s valuables would not have been stolen. A jury is likely to find that the burglar’s
intervening act was sufficiently foreseeable that the construction company will be held liable for
at least some of the damages suffered by the homeowner. (A) is incorrect because, under pure
comparative negligence rules, the homeowner could recover some of her damages even if her
negligence was deemed to be greater than that of the defendant. (C) is incorrect because super-
seding cause analysis does not apply to the plaintiff’s negligence. The homeowner’s failure to have
the security system reset is an issue of contributory negligence, which under the jurisdiction’s pure
comparative negligence rules is not a complete defense. (D) is incorrect because, as discussed
above, the burglar’s conduct probably would be deemed foreseeable and therefore not a super-
seding force that cuts off the construction company’s liability.

Answer to Question 52

(A) The son will probably prevail. Under the husband’s will, the sister-in-law’s interest will terminate
upon the wife’s death. Regardless of whether the court determines it to be a life estate per autre
vie (i.e., a life estate with another’s life as the measuring life) or a fee simple determinable (the
wife’s death being the terminating event), the sister-in-law’s interest will end when the wife dies
and the property will go automatically to the son. Because the sister-in-law’s estate will end on
the wife’s death, she cannot affect any rights of any holder of a future interest that will become
possessory on that event. Any lease purporting to continue after the wife’s death is simply invalid
as of that event. When the wife dies, the son has a fee simple and the tenant has nothing. Thus (A)
is correct. (B) is incorrect. The son received a fee simple on the wife’s death, and he is not bound
by any actions of the holder of the life estate purporting to lease the property for any period that

MPQ 100 preview diagnostic exam and divider P.indd 78 12/22/2015 4:13:33 PM
MBE PREVIEW DIAGNOSTIC EXAM 79.

would extend beyond the terminating event. (C) is incorrect because the son’s right to possession
begins on the instant of the wife’s death. The life tenant can do nothing to extend that date. (D) is
incorrect. The tenant’s right to possession ends on the wife’s death, regardless of what the tenant’s
lease from the sister-in-law may say. She could give no more than she had, and her interest ended
on the wife’s death.

Answer to Question 53

(A) The classmate is likely to prevail. The motorcyclist intended to assault the classmate, because he
drove at the classmate with the intent to scare him. Under the doctrine of transferred intent, the
intent to assault is sufficient to establish a battery if a touching results. A battery requires: (i) a
harmful or offensive touching to the plaintiff’s person; (ii) intent; and (iii) causation. The causa-
tion element is satisfied here because the motorcyclist set in motion the force which brought about
the assault and subsequent battery. (C) is incorrect because the motorcyclist’s intent to commit an
assault is sufficient. (B) is incorrect because the motorcyclist’s potential negligence in hitting the
accelerator would not be sufficient to cut off liability for his battery. (D) is incorrect because the
fact that the tire may have been defective would not qualify as an intervening force sufficient to
cut off the motorcyclist’s liability. The classmate may have a separate claim against the motorcycle
manufacturer on a products liability theory, but the motorcyclist remains liable for the battery.

Answer to Question 54

(A) The speaker’s conviction will be reversed. A park is a public forum. The government can limit
rights of speech in such a forum only when there is a serious and imminent threat to the public
order. It can restrict the speech of a speaker because of an unruly audience only in the rare case
when the police are absolutely unable to control the crowd. [See Feiner v. New York (1951)] In this
question, the conditions under which the police can prevent a speaker from continuing because of
an unruly crowd have not been met. There were 50 police officers who would have been able to
restrain or subdue anyone who appeared to be intent on committing violence. Hence, (C) is incor-
rect. (B) is incorrect. No one in the audience has raised any constitutional argument. The speaker
probably cannot raise the audience members’ First Amendment rights in this situation. (D) is
incorrect. The state may not limit access to a public forum on the sole basis that there are other
times and places where the right of free speech can be exercised. The state must show a more
substantial reason.

Answer to Question 55

(B) The mechanic is bound to pay the neighbor the additional $50. A contract for the sale of goods
may be modified without consideration to support the modification if the modification was sought
in good faith. No writing is required under the Statute of Frauds unless the contract, as modified,
is within the Statute. Here, the parties formed a contract for the sale of goods (a car) when the
mechanic agreed to buy the car for $400. (B) is correct because the contract as modified is under
$500, so it is enforceable, even though it is not evidenced by a writing. (A) is incorrect because
the fact that the original contract was not in writing is irrelevant. The original contract was for the
sale of goods under $500 in value, and the Statute of Frauds does not apply. Thus, the mechanic
could have enforced the original oral contract if he had not agreed to the modification. (C) is
incorrect because under U.C.C. section 2-209, no consideration is needed for the modification of a
contract for the sale of goods. (D) is incorrect because all Article 2 contracts can be modified, if
done in good faith.

MPQ 100 preview diagnostic exam and divider P.indd 79 12/22/2015 4:13:33 PM
80. MBE PREVIEW DIAGNOSTIC EXAM

Answer to Question 56

(D) The contractor will prevail because the buyer received the substantial benefit of her bargain. The
failure to perform on time is a breach of contract, but in this case, it was a minor breach. Unless
the nature of the contract is such as to make performance on the exact day agreed upon of vital
importance, or the contract provides that time is of the essence, failure to perform at the stated
time is not a material breach. Here, the home was nearly complete, and the delay was relatively
short. The contract did not specify that time was of the essence; thus, the breach was minor. The
remedy for a minor breach is damages; the aggrieved party is not relieved of her duty to perform.
(A) is incorrect because merely stating a date for performance does not indicate that time is of
the essence. There must be some explicit statement indicating that time is of the essence. (B) is
incorrect. Although the delivery on March 1 is a condition precedent to the buyer’s duty to pay,
the condition is excused by substantial performance. The test for whether a party has substan-
tially performed is the same as the one for assessing whether a breach is minor or material. Here
the breach is minor, the contractor substantially performed, and the condition is excused. (C) is
incorrect because an unforeseeable event does not discharge a party’s duty to perform. A strike at
the contractor’s supplier does not rise to the level of impossibility or impracticability, which would
discharge his duty to perform. The contractor could have procured the supplies elsewhere.

Answer to Question 57

(B) The hunter can be convicted of murder. Conviction for common law murder requires malice. At
common law, malice is: (i) the intent to kill (express malice); (ii) intent to inflict great bodily
injury; (iii) reckless indifference to an unjustifiably high risk to human life (“depraved heart”); or
(iv) intent to commit a felony (“felony murder”). The hunter did not intend to kill a child when he
fired at the target, so he did not meet the specific intent test. Thus, (A) is incorrect. Malice is also
shown when a person acts with a wanton and reckless disregard for human life. Based on these
facts, the hunter clearly acted with malice by firing a high-powered gun in front of an occupied
playground; thus, (B) is the correct answer. (C) is incorrect, given that specific intent is not the
only way to show malice. (D) is incorrect because the hunter’s actions go far beyond simple negli-
gence.

Answer to Question 58

(A) Because the shopper was falsely imprisoned, she can recover damages for the humiliation she
suffered. False imprisonment requires an act or omission that confines or restrains the plaintiff
to a bounded area, intent, and causation. The shopkeeper’s privilege does not apply because the
length of detention must be for a reasonable period of time only, for the purpose of making an
investigation, and here the detention of an hour to check her name on a list was clearly unreason-
able. Hence, the store is vicariously liable for the actions of its security guard. (B) is not quite an
accurate statement. The fact that the shopper was falsely imprisoned may be attributable to the
fact that the grocery store was negligent in identifying her, but her cause of action arises out of
the fact that she was falsely imprisoned, regardless of how it came about. (C) is incorrect; once
the tort of false imprisonment is established, humiliation is a type of actual damages that can be
recovered. (D) is an incorrect statement because, while the grocery store’s outrageous conduct
might be important with regard to the shopper’s right to recover for intentional infliction of
emotional distress, she can recover humiliation damages because of the false imprisonment even
without that showing.

MPQ 100 preview diagnostic exam and divider P.indd 80 12/22/2015 4:13:33 PM
MBE PREVIEW DIAGNOSTIC EXAM 81.

Answer to Question 59

(D) The court should deny the consumer’s motion. The plaintiff must prove the elements of the prima
facie case for her claim. Absent proof of an element of the prima facie case, summary judgment
for the plaintiff is not appropriate. Thus, (A) is incorrect and (D) is correct. Furthermore, although
a party is generally required to respond to a motion for summary judgment with affidavits, the
facts here indicate that the consumer has not come forward with any evidence pertaining to the
manufacturer’s fault for the damage and injury. Thus, because the consumer has not properly
supported his motion for summary judgment with relevant, admissible information, there is no
need for manufacturer to produce any evidence to survive a motion for summary judgment. This
makes (B) incorrect. (C) is an incorrect statement of the law. A motion for summary judgment
may be granted even though the motion addresses an ultimate issue in the case.

Answer to Question 60

(B) The best argument is that the burglar made the statements spontaneously. Prior to a suspect’s
being charged with a crime, the Fifth Amendment privilege against compelled self-incrimination
is the usual basis for ruling on the admissibility of a confession. [Miranda v. Arizona (1966)]
Under Miranda, statements made during custodial interrogations are inadmissible unless the
defendant is first warned of his right to remain silent and his right to an attorney. Thus, Miranda
applies only when the defendant is in custody and only when the defendant’s statements are the
result of interrogation. Although almost any words or actions on the part of the police that they
should know are reasonably likely to elicit an incriminating response qualify as interrogation,
Miranda does not apply to spontaneous statements not made in response to interrogation. Here,
the police did nothing to solicit the statement from the burglar; it was spontaneous. Thus, (B) is
correct. (A) is incorrect because the defendant need not yet be charged for Miranda rights to apply
as long as he is in custody (i.e., not free to leave). Being in jail on another charge (as the burglar
was) satisfies the custody requirement. (C) is incorrect because the fact that the officer who took
the burglar’s admission had nothing to do with the investigation of the burglary does not alter the
rules of Miranda—questioning that is totally unrelated to the matter for which the accused is in
custody may still violate the accused’s Miranda rights. (D) is incorrect. Due process requires that
a confession be voluntary (i.e., not the product of police coercion). The Miranda rule, however,
goes beyond voluntariness. It makes inadmissible all statements obtained without Miranda
warnings or without a valid waiver of Miranda rights, not just statements actually coerced by the
police.

Answer to Question 61

(B) The proctologist will likely win. When the proctologist bought the property from the homeowner,
he was a bona fide purchaser who gave value and who had no notice of the earlier sale to the
buyer. Not only did the proctologist not have actual notice of the earlier sale, he did not have
constructive notice either because the buyer did not record before the proctologist bought. The
recording statute in the jurisdiction is a notice statute. In a jurisdiction with a notice recording
statute, a subsequent purchaser who gives value and takes without notice wins over the earlier
grantee. If the facts had shown a jurisdiction with a race-notice recording act, the proctologist
would have been in trouble. With race-notice, the proctologist would not only have to take without
notice, he would have to be the first to record. Because the facts do not show that the proctologist
recorded at all, he would lose. But because this is a notice act jurisdiction, the fact that the buyer
finally recorded before the proctologist is irrelevant. (A) is incorrect. Simply because a tenant pays
rent to someone who the tenant thinks owns the property does not create an estoppel requiring the

MPQ 100 preview diagnostic exam and divider P.indd 81 12/22/2015 4:13:33 PM
82. MBE PREVIEW DIAGNOSTIC EXAM

tenant to continue paying rent to that person. The proctologist will win, but not for this reason. (C)
is incorrect because it is not necessary for a grantee to record a deed to give the grantee legal title
to the property. Recordation is necessary only to protect against subsequent purchasers from the
same grantor. Unrecorded deeds are perfectly valid as long as the recording act does not dictate
otherwise. (D) is incorrect. The proctologist did in fact have good title to the property because, as
a bona fide purchaser, he was protected by the notice recording act.

Answer to Question 62

(B) The court should grant the motion. This question involves the interrelationship between the
exercise of testimonial privilege (the privilege against self-incrimination) and the right to cross-
examine a witness claiming such a privilege. Should the witness’s direct testimony be stricken
because he refused to answer a question on cross-examination? The answer is yes, at least in
the context of this case. Here, the question on cross-examination (“Isn’t it true that on the day
of the accident you were 300 miles away from the scene attempting to extract protection money
from the owner of a seaside restaurant?”) is centrally relevant to the witness’s direct testimony
(pertaining to his supposed observation of the accident). Every party has a right to adequate cross-
examination of a witness who has testified for an opposing party. If adequate cross-examination
cannot be obtained, the remedy is to strike the direct testimony of the witness. For this reason,
(B) is correct, and (D) is an incorrect answer. (C) is incorrect, although it correctly states that
the witness has a right to refuse to answer. The allegation of criminal activity contained in the
question he declined to answer is very clear. There is thus no doubt that it is proper for the witness
to “take the Fifth Amendment” in response to the question. However, (C) is incorrect in its
conclusion, that the request to strike the witness’s direct testimony should be denied. It does not
logically follow that, because a witness has a right to refrain from answering a question, he has a
corresponding right to have his direct testimony admitted into evidence. No such “right” exists,
particularly where the invocation of a testimonial privilege has the effect of depriving a party of
the right to adequately cross-examine the witness claiming the privilege. (A) is incorrect because
the privilege against self-incrimination is applicable in any setting (civil case, legislative hearing,
custodial interrogation of a suspect, etc.). As long as the information requested of a person could
then be used against him in a subsequent criminal proceeding, that person has a right to invoke
the privilege against self-incrimination.

Answer to Question 63

(A) The motion to suppress will be granted unless the officer had probable cause to believe that the
car contained seizeable items. The automobile exception to the warrant requirement permits a
stopped vehicle to be searched in the absence of a warrant if the police have probable cause to
believe that the vehicle contains seizeable evidence. (B) is incorrect. A search incident to arrest
may not include the passenger compartment of a vehicle from which the person was arrested
unless (i) the person has not been secured or (ii) the arresting officer has reason to believe that the
vehicle contains evidence of the crime for which the arrest was made. [Arizona v. Gant (2009)]
Here, the officer was able to secure the driver and the arrest was for speeding, so neither justi-
fication applies. However, even though the evidence is not admissible under the exception for a
search incident to arrest, it is admissible under the automobile exception (as discussed above) if
the officer had probable cause to believe that the car contained seizeable items, as stated by choice
(A). (C) is incorrect because it is too broad. The passenger compartment of a vehicle cannot be
searched incident to arrest unless one of the two criteria stated above are met. (D) is incorrect
because “reasonable fear” is not a ground for searching a vehicle. A vehicle may be searched
incident to arrest under the circumstances detailed above. It may also be searched under the

MPQ 100 preview diagnostic exam and divider P.indd 82 12/22/2015 4:13:33 PM
MBE PREVIEW DIAGNOSTIC EXAM 83.

automobile exception if the police have probable cause to believe that the vehicle contains seize-
able evidence. Finally, a vehicle can be searched without a warrant pursuant to an investigative
stop (i.e., a Terry stop) if the police have reason to believe that the person stopped is armed and
dangerous. But simple “fear” is not enough.

Answer to Question 64

(B) Because of a lessened expectation of privacy in a car (as compared to a home) and because of the
inherent mobility of a car which can prevent the police from easily obtaining a search warrant
and searching it, the police are permitted to make a complete search of an automobile if there is
probable cause to believe that the car contains the fruits, evidence, or instrumentalities of a crime.
[Carroll v. United States (1925)] When the police have probable cause to search an entire vehicle,
they may conduct a warrantless search of every part of the vehicle and its contents. [United States
v. Ross (1982)] While it is doubtful that the marijuana joints provided sufficient probable cause
to search the entire vehicle, this is the best reason to deny the motion, because all of the other
choices are clearly incorrect. (A) is incorrect because a lawful search of a vehicle incident to an
arrest includes a search of the defendant and, in limited circumstances, the area of the vehicle
within the defendant’s control. [Arizona v. Gant (2009)] In this case, as the trunk is outside the
defendant’s area of control, the search went beyond the scope of a proper search incident to an
arrest. Therefore, this was an invalid search incident to arrest and the driver’s motion would be
granted on this basis. (C) is incorrect. Technically, there is no such thing as a custody search.
“Custody” here may refer to custody of the person, in which case it is a search incident to arrest
discussed above, or custody of the vehicle, which is an inventory search. An inventory search
occurs when the officers properly impound a car and search it while it is in their control to make
sure that the assets of the owner of the car are fully accounted for. [South Dakota v. Opperman
(1976)] This choice is incorrect because it does not articulate a legal basis to conduct the search,
as the car was not impounded. (D) is incorrect. In this case, the police did not take possession of
the car, so this is not the best reason to deny the motion.

Answer to Question 65

(B) The court should rule that the expenditure is unconstitutional. The federal power to tax and
spend includes the power to attach conditions to the expenditures, and to the extent the univer-
sity research program conflicts with the federal legislation, the state action is superseded by the
federal statute. Here, because the federal program expressly limited use of the distributed funds
to development of synthetic materials to replace petroleum, the university projects involving
solar, geothermal, biomass, and fusion energy would be in conflict with the statutory prescrip-
tion. “Substantial” conformity is not enough, so (C) is incorrect. (D) is an incorrect statement of
the law. The supremacy doctrine permits Congress to effectively control the actions of a state in
spending money provided by the federal government. (A) is incorrect. There is nothing unconsti-
tutional about the means by which the funds are disbursed.

Answer to Question 66

(C) The sister will likely prevail. While an obligor (the stepson here) may raise defenses on the
obligation he owes to the assignor (the brother), he may not raise defenses that the assignor might
have had against the assignee (the sister-creditor) on a different obligation as a means of avoiding
his own obligation. Here, there are no apparent defenses to the stepson’s liability to the brother for
the $2,000 debt, and the stepson is not entitled to assert the brother’s statute of limitations defense
against the sister because the debt that the brother owed to his sister was a different obligation.

MPQ 100 preview diagnostic exam and divider P.indd 83 12/22/2015 4:13:33 PM
84. MBE PREVIEW DIAGNOSTIC EXAM

(B) is incorrect for the reason discussed above—she is suing the stepson on a different obliga-
tion. (A) is incorrect because the assignment of the stepson’s debt to the brother is of the stepson’s
entire debt to him, even though it will only partially repay the money that the brother had owed
to his sister. In any event, partial assignments are not invalid. Assignors may transfer some rights
under the contract and retain others. (D) is incorrect because the assignment was not a novation.
A novation substitutes a new party for an original party to the contract and requires the assent of
all parties. If valid, it completely releases the original party. Here, the stepson did not assent to
anything and no one was released, so there was no novation.

Answer to Question 67

(D) The bank will be successful in obtaining a judgment against both the landowner and the attorney,
although it may only collect once. When a grantee assumes the mortgage, the grantee expressly
promises the grantor-mortgagor that he will pay the mortgage obligation as it becomes due. The
mortgagee then becomes a third-party beneficiary of the grantee’s promise to pay and can sue the
grantee directly if the grantee fails to pay. After the assumption, the grantor-mortgagor becomes
a surety who is secondarily liable to the mortgagee on the note if the grantee fails to pay. The
landowner and the attorney are jointly liable, even though the attorney is primarily liable and
the landowner is secondarily liable as a surety. Therefore, (A) and (B) are incorrect. (C) is incor-
rect. The bank is not required to choose between the landowner and the attorney and can obtain
a judgment against both, although it may only collect once. Because the attorney assumed the
mortgage obligation, the bank can sue the attorney, but it can also sue the landowner in the same
action as a surety. The landowner and the attorney are jointly liable, even though the attorney is
primarily liable and the landowner is secondarily liable as a surety.

Answer to Question 68

(A) The creditor will be successful against the developer only. The developer initiated an agreement
with the creditor to convey the property to the creditor in satisfaction of the developer’s obligation
to the creditor. The deed was meant to convey absolute title to the creditor, and was not intended
to be another form of the mortgage that already existed. Therefore, the developer has no further
interest in the property. (B) is incorrect. The creditor will not prevail over the bank. The convey-
ance from the developer to the creditor discharged the first mortgage on the property because
the conveyance was in satisfaction of the mortgage obligation. The first mortgage, however, was
never foreclosed and therefore did not wipe out the second mortgage. When the first mortgage was
discharged by the conveyance, the second mortgage became a first mortgage and is still a valid
encumbrance on the property. (C) and (D) are incorrect because the creditor will prevail against
the developer but not against the bank.

Answer to Question 69

(A) The farmer should recover all of his damages against the chemical plant. To establish the element
of actual cause in a negligence action, the “but for” test is ordinarily used. However, when several
causes commingle and bring about an injury, and any one alone would have been sufficient to
cause the injury, the “but for” test is inadequate to determine actual cause. In these cases, it is
sufficient if the defendant’s conduct was a “substantial factor” in causing the injury. Here, the
farmer cannot establish that, but for the chemical plant’s negligence, he would not have suffered
damage, because either discharge was sufficient to cause his injury. However, he can establish
actual cause because the chemical plant’s negligence was a substantial factor in causing his injury;
hence, he can recover against the chemical plant. (B) is incorrect because the farmer does not

MPQ 100 preview diagnostic exam and divider P.indd 84 12/22/2015 4:13:33 PM
MBE PREVIEW DIAGNOSTIC EXAM 85.

need to present evidence to apportion damages. Under joint and several liability, when two or
more tortious acts combine to proximately cause an indivisible injury to a plaintiff, each tortfeasor
is liable to the plaintiff for the entire damage incurred. Thus, he can collect all of his damages
from the chemical plant; it will be up to the chemical plant to seek apportionment of damages in
a contribution action against the steel mill. (C) is similarly incorrect. The farmer does not need to
join the steel mill in the litigation to recover. (D) is incorrect because that choice applies the “but
for” test. As discussed above, the appropriate test in this case is the “substantial factor” test.

Answer to Question 70

(D) The court should admit the evidence if it was used to refresh the witness’s recollection. Because
the witness read the notes, and then had an independent recollection of events, this qualifies as
a present recollection refreshed. Normally a writing used to refresh is not placed into evidence.
However, under Federal Rule of Evidence 612, if a writing is used to refresh the recollection of
a witness, the opposing party has a right to introduce the document into evidence. Thus, (A)
and (B) are incorrect. (C) is incorrect because, if the witness’s memory was not refreshed, the
memorandum would have to be introduced as a past recollection recorded. The proper foundation
has not been laid for admission of a past recollection recorded.

Answer to Question 71

(B) The summary of the accident may be read into evidence. If the witness on the stand has insuf-
ficient recollection to testify to a relevant event fully and accurately, Federal Rule 803(5) permits
the introduction of an out-of-court written record of the event made by the witness at a time when
the witness’s memory of the event was fresh. The fact that the witness’s memory of the actual
event is insufficient even after reviewing the summary would satisfy that standard, assuming a
proper foundation has been laid for reading the statement into evidence. If the past recollection
recorded is admissible, the record itself may not be admitted; unless it is offered into evidence by
the adverse party, the offering party may only read the record to the jury, as here. (A) is incor-
rect. There is no question that the document which is read must satisfy the best evidence rule
and be either the original or a duplicate, or there must be a satisfactory excuse for nonproduction
of an original. There is no indication in this case that the summary in the witness’s possession
would fail to satisfy the best evidence rule. It appears to be her original notes. The real question
here is whether the document in question is admissible despite the hearsay rule. (C) is incorrect.
If a written record is used to refresh the memory of a witness who then proceeds to testify from
her own memory as to the matter, then the memorandum is not evidence coming within the past
recollection recorded exception under Federal Rule 803(5). A document which is used to refresh a
witness’s memory may not be read to the jury. Instead, the witness will testify from her refreshed
memory. (D) is incorrect. The hearsay rule applies in general to any out-of-court statement,
whether or not the declarant is available for cross-examination. However, this statement comes
within the hearsay exception for a past recollection recorded under Federal Rule 803(5). Under
that Rule, the contents of the memorandum may be read to the jury.

Exam Tip: The difference between present recollection refreshed and past recollection
recorded is a recurring exam favorite.

Answer to Question 72

(D) The enactment appears to be an unconstitutional infringement on the President’s authority as


Commander in Chief. The President’s role as Commander in Chief of the armed forces includes

MPQ 100 preview diagnostic exam and divider P.indd 85 12/22/2015 4:13:33 PM
86. MBE PREVIEW DIAGNOSTIC EXAM

extensive power to deploy the military against any enemy, foreign or domestic. Congress lacks such
power. Therefore, this enactment directly infringes on the President’s authority as Commander in
Chief to make such orders as he deems proper with respect to the armed forces, and thus violates
the doctrine of separation of powers. (A) is incorrect. Congress does have the power to raise and
support an army, but the enactment here does not result in the appropriation of money to support
the armed forces. Rather, it seeks to control their activities, and Congress has no such power.
(B) is incorrect. Congress does have the power to regulate commerce, and when that power is
combined with the Necessary and Proper Clause, Congress would have the power to enact legisla-
tion protecting the toy shipments at issue here. Nevertheless, the Commerce Clause, even when
combined with the Necessary and Proper Clause, does not give Congress the power to violate other
aspects of the Constitution. Ordering the President to send military troops violates the separation
of powers doctrine because the Constitution gives the President the power as Commander in Chief
of the armed forces. (C) states the correct result but is based on an incorrect rationale—the duty to
execute the laws of the United States is an obligation, not a grant of authority.

Answer to Question 73

(B) The federal court may not hear these claims together because there is no supplemental jurisdic-
tion over the negligence claim. Generally, every claim in federal court must have a basis for
federal subject matter jurisdiction. There are two main bases for federal subject matter jurisdic-
tion—diversity of citizenship jurisdiction and federal question jurisdiction. Once a claim is in
federal court, supplemental jurisdiction sometimes may be used to have a claim heard. Diversity
of citizenship jurisdiction is available when (i) there is complete diversity of citizenship, meaning
that each plaintiff is a citizen of a different state from every defendant; and (ii) the amount
in controversy exceeds $75,000. A natural person’s citizenship is the state that is the person’s
domicile. In the instant case, the facts state that the broker and customer are from the same state.
As a result, complete diversity does not exist. Furthermore, although the customer may aggre-
gate all the claims he has against the broker, the aggregate amount ($74,000) does not meet the
minimum amount in controversy requirement. For these reasons, subject matter based on diversity
is not available. Federal question jurisdiction is available when the plaintiff, in his well-pleaded
complaint, alleges a claim that arises under federal law. Anticipation of a federal defense or the
fact that federal law is implicated by the plaintiff’s claim do not give rise to federal question
jurisdiction; the plaintiff’s claim must arise under federal law. Here, the customer alleges that the
broker violated federal securities law. That is sufficient to invoke federal question jurisdiction over
the securities claim (which does not have an amount in controversy requirement or a complete
diversity requirement). However, the claim for damages to the customer’s car is a state law claim,
and federal question jurisdiction is not available. Thus, to be heard, the negligence claim must
invoke supplemental jurisdiction. When the federal court has subject matter jurisdiction over one
claim, it has discretion to exercise supplemental jurisdiction over related claims that derive from
the same common nucleus of fact and are such that a plaintiff would ordinarily be expected to try
them in a single judicial proceeding. Here, though, the customer’s negligence claim is not related
in any way to the customer’s claim for violating federal securities law. As a result, supplemental
jurisdiction is not available. Thus, (B) is the correct choice and (D) is incorrect. (C) is incorrect
even though it contains a true statement of law because there is no supplemental jurisdiction and
thus no federal subject matter jurisdiction. (A) is too broad of a statement. When dealing with a
single plaintiff against a single defendant, the plaintiff is allowed to join any number and type of
claims against the defendant. Thus, if subject matter jurisdiction requirements could have been
satisfied, the customer here could have joined all the claims he has against the broker. (When
multiple plaintiffs or multiple defendants are involved, it is essential only that at least one of the
claims arise out of a transaction in which all were involved.)

MPQ 100 preview diagnostic exam and divider P.indd 86 12/22/2015 4:13:33 PM
MBE PREVIEW DIAGNOSTIC EXAM 87.

Answer to Question 74

(C) The nurse is only an assignee of the tenant. The nurse is not directly contractually obligated to
the landlord. Once an assignee of the nurse goes into possession of the premises (especially with
the consent of the landlord), the privity of estate between the landlord and the nurse, the prior
assignee, is terminated. An assignee has no contractual obligation under the original lease and is
only liable for rent during the period of his estate, i.e., his possession. Thus, in this case, there is
no longer any privity of estate between the nurse and the landlord, and the landlord cannot sue the
nurse on the rent covenant. (A) is incorrect. The covenant of quiet enjoyment was not breached
here because no one with superior title interfered with possession of the apartment. Moreover,
the duty to provide quiet enjoyment is owed by the landlord only to the tenant and his succes-
sors in interest (i.e., those in privity of estate). Here, there is no privity of estate between the
landlord and the nurse because the nurse is an assignee who is no longer in possession at the time
of his brother’s abandonment, and that is the nurse’s best defense. (B) is incorrect. The defense
of estoppel may be invoked where a party, acting in good faith, has been induced by the conduct
of the adverse party to do something that it otherwise would not have done, that resulted in his
harm, and that the adverse party had cause to know would so result in harm. These facts do not
support the use of the theory of estoppel to avoid the application of landlord-tenant law, because
the landlord has made no representations which were relied upon by the nurse to his detriment.
(D) is incorrect, if only because there is a better defense available to the nurse. The defense of
constructive eviction might be available to the nurse’s brother, who was in privity of estate with
the landlord during the month in question. However, it is less clear that it would be derivatively
available to the nurse. Moreover, it is a difficult factual defense. The landlord must be shown to
have the ability to control the noise and to have failed to exercise that control. The noise must be
of sufficient magnitude to deprive the nurse’s brother of the reasonable beneficial use of the apart-
ment. The defense of lack of privity of estate is clear-cut and much more beneficial to the nurse.

Answer to Question 75

(A) Venue is proper only in the Northern District of State A. Federal venue in diversity actions is
proper in (i) the district in which any defendant resides if all defendants reside in the same state;
and (ii) the district in which a substantial part of the events or omissions giving rise to the claim
occurred. Here, the accident occurred in the Northern District of State A, making that district a
proper venue under prong (ii). However, given that the defendants here reside in different states,
venue cannot be based on the residence of the defendants. Thus, choices (B), (C), and (D) are
incorrect. Choice (D) is also incorrect because venue is not based on the residence of the plaintiff.

Answer to Question 76

(C) The judge should rule for the state. This question is not about propensity evidence. Rather, it is
about evidence of prior bad acts to help prove something other than a person’s propensity to act
in a manner consistent with a character trait. Under Federal Rule 404(b), it is permissible to use
evidence of a criminal defendant’s prior bad acts to establish a motive for committing the crime,
to help to identify him as the perpetrator of a crime, or, as here, to show opportunity to commit
the crime. The defendant claims that she could not have committed the murder because she
was on a business trip the day it occurred. The witness’s testimony, however, pertaining to his
encounter with the defendant in a crosstown bar on the day of the murder, is clearly relevant to
show that the defendant had the opportunity to commit the murder. For this reason, (C) is correct.
All the other answers are wrong because they mistakenly assume that the witness’s testimony
involves propensity evidence rather than prior bad acts evidence. (A) correctly states that the

MPQ 100 preview diagnostic exam and divider P.indd 87 12/22/2015 4:13:33 PM
88. MBE PREVIEW DIAGNOSTIC EXAM

defendant did not place her character at issue by testifying that she was across the country when
the murder occurred. That would be a reason for excluding the witness’s testimony if his testi-
mony amounted to an attack on the defendant’s character (i.e., propensity evidence). However, his
testimony is to show opportunity; thus, it may be admitted. (B) correctly implies that propensity
evidence can be used to impeach the testimony of a witness only if it addresses the witness’s
character for truth and veracity. The defendant was a witness in the trial. If the purpose of the
witness’s testimony had been to impeach her testimony by attacking her character for something
other than truth or veracity, the testimony would be inadmissible. Because this evidence was
admitted not to impeach but to show opportunity, (B) is incorrect. (D) correctly implies that
the prosecution cannot introduce propensity evidence establishing a defendant’s bad character
until the defendant has placed her character in issue. The defendant’s testimony did not place
her character in issue. Thus, if the witness’s testimony amounted to an attack on the defendant’s
character, the testimony would be inadmissible. Because the testimony was not an attack on the
defendant’s character, however, (D) is incorrect.

Answer to Question 77

(D) The electrician will not succeed because there was an express condition precedent that the repairs
had to be performed on a day when the homeowner would be home, and the homeowner was
unable to honor that commitment due to an unforeseeable supervening event. (C) is not the best
answer. Although the electrician did nothing between April 4 and April 26, he did confirm the
repair date of April 29 with the homeowner. The electrician would have honored the contract but
for the supervening impossibility of the homeowner being unable to stay home. (A) is a false state-
ment. The promise to make the house available was an express condition, not an implied-in-fact
contract. (B) is wrong because impossibility discharges duties on both sides.

Answer to Question 78

(D) The trustee will probably recover nothing. Destruction of a building being built under a construc-
tion contract does not render the contract impossible or impracticable to perform and therefore
does not discharge the contractor from his duties. The original construction company could not
continue to perform because the loss of its equipment rendered it bankrupt. Therefore, it was in
breach and not entitled to any contractual remedy. Neither could it recover in restitution or quasi-
contract. Unlike a contract to repair or remodel, no restitutionary or quasi-contract remedy is
available when the subject matter of a construction contract is destroyed.

Answer to Question 79

(B) The court should rule that the statute is constitutional. A state has the right, under its police
power, to enact legislation for the health, safety, and welfare of its citizens, provided that such
legislation is not in areas reserved to the federal government or preempted by federal legislation,
and provided that a state does not unduly restrain interstate commerce. The legislation in question
affects only that state’s banks and does not discriminate against other banks. The political process
is considered the adequate method for bringing such statutes as the anti-usury statute in line with
contemporary conditions. (A) is incorrect because even if a state statute does not actually conflict
with a federal statute, the state statute may still fail under the Supremacy Clause if it prevents
achievement of federal objectives or Congress has indicated an intent to preempt an entire field.
Thus, (B) is a better answer than (A). (C) is incorrect because income level is not a “suspect”
category, and the fact that the super-rich can purchase homes while everyone else cannot is an
incidental result of economic conditions. Regardless of the interest rate, there will always be some

MPQ 100 preview diagnostic exam and divider P.indd 88 12/22/2015 4:13:33 PM
MBE PREVIEW DIAGNOSTIC EXAM 89.

citizens who cannot obtain mortgages. (D) is incorrect because the legislation affects only that
state’s banks. There may be a slight incidental effect on interstate commerce, but the legislation in
no way discriminates in favor of that state’s banks.

Answer to Question 80

(D) The driver must disclose the identity of the witness in response to an appropriate interrogatory.
Federal Rule 26(a) requires, as an initial disclosure, a party to reveal the name and contact infor-
mation of individuals who are likely to have discoverable information and who the disclosing
party may use to support his claims or defenses (unless the use would be solely for impeach-
ment). After initial disclosures are made, discovery proceeds, and the parties may continue with
discovery of nonprivileged information that is relevant to any party’s claim or defense, including
the names and contact information of any person who knows of any discoverable matter. Here,
the eyewitness would not need to be disclosed as an initial disclosure because the driver obviously
will not use the eyewitness to support the driver’s claim or defense. However, the identity of the
eyewitness would need to be disclosed eventually, assuming the pedestrian submits a proper
discovery request. This makes (D) correct and (B) incorrect. (A) is incorrect because the work
product doctrine does not prevent the disclosure of the existence of the eyewitness. Any materials
generated by the attorney would probably be protected under the work product doctrine (unless
a showing of substantial need and undue hardship can be made); however, the eyewitness’s name
and contact information would not be protected. (C) is an overbroad description of the initial
disclosure requirements and is, thus, incorrect.

Answer to Question 81

(D) The strongest argument is that the shopkeeper was reacting to a sufficient provocation. This is a
very commonly tested MBE issue. (A) is incorrect. It is clear from the facts that the shopkeeper
manifestly intended to kill or seriously injure the gangster, and this intent is transferred to the
delivery person, who was killed by mistake. (B) is incorrect because any imminent threat of death
or serious injury to the shopkeeper had passed at the time he shot at the gangster. (C) is incorrect
because there is nothing in the facts that indicates duress. Therefore, (D) presents the strongest
argument—that the gangster’s intentional near miss with the knife was sufficient provocation to
mitigate the intended homicide from murder to voluntary manslaughter.

Answer to Question 82

(D) Either the pilot’s or the son’s conduct may reduce the pilot’s recovery in a jurisdiction applying
pure comparative negligence. Recovery in a wrongful death claim is allowed only to the extent
that the deceased could have recovered in the action had he lived. Because the pilot’s wrongful
death claim derives from her son’s death, either the son’s or the pilot’s contributory negligence
will diminish the claim. Thus, although (A) and (B) are correct statements, they are not as good
as (D), which is more complete. (D) states that the negligence of either the pilot or the son will
reduce recovery, and this is the correct rule. (C) is wrong because only one person need be negli-
gent to reduce recovery.

Answer to Question 83

(C) The neighbor should be found not guilty of larceny because he did not have the requisite intent.
Larceny consists of a taking and carrying away of tangible personal property of another by
trespass, with intent to permanently deprive the person of his interest in the property. If the

MPQ 100 preview diagnostic exam and divider P.indd 89 12/22/2015 4:13:33 PM
90. MBE PREVIEW DIAGNOSTIC EXAM

defendant intends to return the property within a reasonable time and at the time of the taking
has a substantial ability to do so, the unauthorized borrowing does not constitute larceny. Hence,
the neighbor is not guilty of larceny because he planned to return the mulcher. (A) is incorrect
because the fact that the neighbor had not yet used the mulcher would be irrelevant if he had
formed the intent to steal it. (B) is incorrect because the fact that the garage door was open is
irrelevant. The act of breaking into a dwelling is an element of burglary, not larceny. (D) is incor-
rect because the neighbor had taken possession of the mulcher and started to carry it away, which
satisfies the asportation requirement. It is irrelevant that he had not yet exited the garage.

Answer to Question 84

(A) The court should deny the motion because the nursing home could be found liable under the
doctrine of res ipsa loquitur. This doctrine would apply because the injury ordinarily would not
have occurred in the absence of negligence on the part of the nursing home, such as by accepting
violent patients without providing adequate security for other patients. The nursing home has
assumed the duty of ordinary care for the safety and security of its patients. Because the doors
were locked, the injury occurred as a result of either a nursing home employee or a patient,
and the nursing home would be liable in either case. Admitting patients known for their violent
behavior created a foreseeable risk of injury to all the patients, and the exercise of ordinary
care would require that the nursing home provide adequate security to protect the patients. The
patient’s beating establishes enough evidence of the breach of duty by the nursing home here to
withstand the motion and send the case to the jury. (B) is incorrect. There is no rule of law that
imposes a high degree of care in this situation—just ordinary care under the circumstances.
Thus, the nursing home is only liable because it has breached its duty of ordinary care for the
safety and security of the patients. (C) is incorrect. The patient need not prove that the beating
was caused by an employee in order to impose liability on the nursing home, because violence
by another patient was foreseeable. The nursing home has assumed the duty of ordinary care
for the safety and security of its patients. (D) is incorrect. An intervening criminal act of a third
party will not cut off liability if it was foreseeable and the defendant owed a duty of care to the
plaintiff. The nursing home has assumed the duty of ordinary care for the safety and security of
its patients, and here criminal acts were foreseeable because the nursing home accepted violent
patients.

Answer to Question 85

(A) The court should order specific performance by the buyer. Because land is considered unique,
specific performance is always appropriate for the enforcement of a valid land sale contract. This
option is available to either the buyer or the seller. The contract gave the seller the option of using
the liquidated damages provision if the seller wished, but the seller has sued for specific perfor-
mance. Because that is appropriate, it will be granted. It is not of significance that the house
burned to the ground. When a contract for the sale of land is signed, equitable conversion takes
place and it is, for all practical purposes, the buyer’s land and the buyer’s risk. Here, the risk of
loss shifted to the buyer upon the signing of the contract. (B) is incorrect because frustration of
purpose is not applicable. The doctrine of equitable conversion shifted the risk of loss to the buyer
when the contract of sale was signed. The fact that an improvement on the property (the house) is
no longer present is not relevant to the grant of specific performance. (C) is incorrect. Based upon
these facts, the seller could request as a remedy either specific performance or liquidated damages
of $6,000. Because the seller elected the specific performance remedy, the court will not award
liquidated damages. (D) is also incorrect because there are no facts which would indicate mutual
mistake. The burning down of the house would not qualify.

MPQ 100 preview diagnostic exam and divider P.indd 90 12/22/2015 4:13:33 PM
MBE PREVIEW DIAGNOSTIC EXAM 91.

Answer to Question 86

(C) The court may not transfer the action. Federal venue in diversity actions is proper in (i) the district
in which any defendant resides if all defendants reside in the same state; and (ii) the district
in which a substantial part of the events or omissions giving rise to the claim occurred. The
Southern District of State A is not a proper venue because it is neither the defendant’s residence
nor a place where substantial events occurred giving rise to the action. As a result, the court must
either dismiss, or in the interests of justice, transfer the case to a district in which the action could
have been brought on the defendant’s timely objection to venue. Here, however, the court may
not transfer to the Northern District of State A because that district also is improper, as it is also
neither the defendant’s residence nor a place where substantial events occurred giving rise to the
action, and thus the action could not have been brought there. Thus, (B) is incorrect. (A) is incor-
rect because, in federal court, venue may not be based on the plaintiff’s residence. (D) is incorrect
because the venue statute does not allow the judge to make such a discretionary call; rather, the
district to which venue is transferred ordinarily must be a place of proper venue.

Answer to Question 87

(D) The State A citizen must assert her claim. Because the State A citizen’s claim arises from the
same transaction or occurrence as the claim asserted against her in the pending action, the State A
citizen’s claim is a compulsory counterclaim which must be asserted in the pending action, or it is
lost. As a result, (D) is correct and (A) is incorrect. (B) is incorrect in that it is not a discretionary
call with the judge whether the claim may be filed as a counterclaim. (C) is incorrect because the
claim may not be asserted as an independent action. FRCP 13 states that the pleader need not state
a compulsory counterclaim if the subject matter of the compulsory counterclaim was already
pending in another court. However, if the pleader has not filed suit on the subject matter of the
compulsory counterclaim (which she has evidently not done here), she must file the compulsory
counterclaim.

Answer to Question 88

(C) The testimony is inadmissible. For purposes of the hearsay rule, a statement is defined by Federal
Rule 801(a) as an oral or written assertion, or nonverbal conduct which is intended as an asser-
tion. In this case, the witness’s pointing at the defendant is a statement because it was intended
by the witness to assert that the defendant was the criminal. The sergeant’s testimony concerning
the witness’s identification is testimony concerning an out-of-court statement and is therefore
hearsay. Because the statement comes within no exception to the hearsay rule, it is inadmissible.
The evidence is not evidence of a prior identification because the witness is not present on the
witness stand. (A) is incorrect. Evidence of a prior identification is nonhearsay under Federal
Rule 801(d)(1)(c) in only a very narrow range of cases. For a prior identification to be nonhearsay,
a witness on the stand must have made a prior statement identifying the defendant as someone
he perceived earlier. The offered evidence does not come within that definition of nonhearsay
because the witness is not present on the witness stand. The sergeant’s testimony of the witness’s
assertive conduct is inadmissible hearsay. (B) is incorrect. The evidence is inadmissible because it
is hearsay and comes within no hearsay exception. The evidence is not evidence of a prior identi-
fication because the witness is not present on the witness stand to testify to the prior identification.
(D) is incorrect. The choice correctly states that the evidence is inadmissible. But it is inadmis-
sible because it is hearsay, not because the defendant’s counsel was not present at the lineup.
Under constitutional principles, the defendant’s counsel would not need to be present at a prein-
dictment lineup such as this in order to have the prior identification be admitted at trial.

MPQ 100 preview diagnostic exam and divider P.indd 91 12/22/2015 4:13:33 PM
92. MBE PREVIEW DIAGNOSTIC EXAM

Answer to Question 89

(C) The lawyer has violated Rule 11. By “presenting” a document to the court, Rule 11 provides that a
lawyer certifies that he believes the denials of factual contentions in the document are warranted
on the evidence and that his belief is formed after a reasonable inquiry. One “presents” a
document not only by signing or filing it, but also by “later advocating” it. Thus, Rule 11 imposes
a continuing certification requirement, applicable any time a matter is presented to the court. At
the time he signed the complaint, the lawyer believed there was evidence that the individual did
not sign the contract based on the individual’s own statement. When the lawyer referred to the
answer in the motion hearing, the lawyer “presented” the answer to the court anew and renewed
his certification, despite the fact that he never discussed the signature. At that time, the lawyer
no longer believed that all the facts in the answer had evidentiary support. He knew that he
lacked evidence to support the answer’s denial that the signature on the contract was that of the
individual. Thus, (C) is correct and (B) is incorrect. (A) is incorrect because it does not take into
consideration the “later advocated” basis for presenting a document under Rule 11. (D) is incorrect
because an attorney need not have his client sign a sworn statement for every fact that the client
tells the attorney.

Answer to Question 90

(D) The President’s order is unconstitutional because it is too broad a limit on the freedom of speech
and association of government employees. No government employees were allowed to have even
a conversation with the press without the permission of a supervisor. Thus, it is unconstitutional.
(A) is incorrect because the President’s plenary power to control executive employees is limited
by the Constitution. For example, the President cannot violate the First Amendment (or other
constitutional guarantees) in regulating federal employees. (B) is incorrect because the Court no
longer draws a distinction between rights and privileges. Furthermore, government employees
would retain their First Amendment rights even if government employment were termed a “privi-
lege.” (C) is incorrect because the President has authority to enact some regulations for federal
employees. The problem with this regulation is that it unduly restricts freedom of speech.

Answer to Question 91

(B) The animal shelter will lose on both farms. As to the deed to the son conveying the 590-acre
farm, delivery is presumed because the deed was validly recorded, even though the son did not
physically receive it and does not seem to have known of its existence. As to the deed to the
daughter conveying the 640-acre farm, the fact that she did not physically keep the deed when it
was presented to her does not show lack of delivery under the circumstances. The key to delivery
is the grantor’s intent to pass all legal controls. Because the daughter simply asked her mother to
safeguard the deed, we can presume delivery under these circumstances. But because the deed
to the daughter was never recorded, we cannot rely on the rule that provided valid delivery in the
son’s case. (A) is incorrect. Both deliveries were valid. The son’s deed was valid by reliance on the
presumption that recordation shows a valid delivery, and the daughter’s deed on the circumstances
of the gift that show proper intent. (C) is incorrect. The animal shelter does lose on the 590-acre
farm, but does not win on the 640-acre farm because the daughter’s having her mother keep the
deed does not show lack of delivery. (D) is incorrect. The animal shelter does lose on the 640-acre
farm, but does not win on the 590-acre farm because the recordation of the son’s deed gives a
presumption of valid delivery.

MPQ 100 preview diagnostic exam and divider P.indd 92 12/22/2015 4:13:33 PM
MBE PREVIEW DIAGNOSTIC EXAM 93.

Answer to Question 92

(B) The court should rule for the telephone company. This question turns simply on whether or not
there was a valid easement. The easement granted by the owner to the telephone company 30
years ago was properly recorded. That alone would put subsequent purchasers of the property on
constructive notice of the easement. The fact that the easement was not in the deed from the owner
to the buyer is irrelevant. A valid easement passes with title whether or not it is mentioned in the
conveyance. For the subsequent purchaser to avoid the easement, that person must have taken
without notice of it. Although there was no actual notice of the easement, a proper title search
would have disclosed the easement, and the buyer is bound as if he knew. (A) is incorrect. If an
easement is valid, there is no need to balance the hardships in order to determine whether the
holder of the easement may make proper use of it. An easement to lay cables underground reason-
ably carries with it the right to inspect those cables or make replacements as is necessary. Hardship
to the holder of the servient estate is not relevant if the easement is proper and the excavation is
considered reasonable use of that easement. (C) is incorrect because the telephone company was
under no obligation to mention its underground cable easement, and its failure to do so is meaning-
less. (D) is incorrect. Valid easements are conveyed along with the servient estate, regardless of
whether they are mentioned in the conveyance or not. Because the easement was recorded, a title
search would have disclosed it, and thus the buyer will be held to have constructive knowledge of it.
Answer to Question 93
(C) The judge should rule the witness’s testimony inadmissible. This question involves the admis-
sibility of propensity evidence. The driver’s witness is prepared to testify that the driver has a
reputation for being a safe and prudent driver, which the driver plans to use to prove that, because
he has a propensity for driving safely, he was in fact driving safely at the time of the accident.
This is a classic case of the impermissible use of propensity evidence. The general rule is that
evidence of character traits (here, safety and prudence) is inadmissible in a civil case to prove
that a party acted in conformity with those traits on a particular occasion. [Fed. R. Evid. 404(a)]
This case fits squarely within that general rule. Thus, (C) is correct. All the other answers are
wrong because they each assume that propensity evidence is admissible under some circum-
stances in a wrongful death action. (A) indicates that such evidence is admissible if there are no
unbiased eyewitnesses to the accident. (B) indicates that such evidence is admissible because
the witness has personal knowledge of the character traits (the driver’s propensity for safety and
prudence) that are the subject of his testimony. (D) indicates that such evidence is admissible
if the plaintiff has attacked the character of the defendant. All of these answers miss the basic
point—that propensity evidence is inadmissible in civil cases. Furthermore, (A) is wrong because
it presumes that there are no unbiased witnesses to the accident, but the question makes clear
that there was an unbiased witness: the bystander. (B) is also wrong because it indicates that
the witness must first testify that he has personal knowledge of the driver’s driving habits before
testifying about the driver’s reputation for safety and prudence. This answer confuses reputation
testimony with opinion testimony. The witness’s testimony pertained to the driver’s reputation. To
be a competent witness, the witness would have to have personal knowledge of the driver’s reputa-
tion. He would not have to have personal knowledge of the driver’s actual driving habits. Although
perhaps not as nonsensical as (A) and (B), (D) is just as inaccurate, as it bears no similarity to
anything in the rules of evidence pertaining to propensity evidence.
Answer to Question 94
(A) The court should rule for the driver. This question is best approached by the process of elimina-
tion, and a preliminary examination of (A) suggests that it is a correct answer. The homeowner’s

MPQ 100 preview diagnostic exam and divider P.indd 93 12/22/2015 4:13:33 PM
94. MBE PREVIEW DIAGNOSTIC EXAM

negligence in burning leaves during a period of relatively high winds resulted in a fire, and it is
foreseeable that third persons will attempt to rescue the victims of the tortfeasor’s negligent acts.
To the extent that the emergency vehicle was an intervening force, it was a dependent one (also
responding to the fire) and was not abnormal or unforeseeable. Thus, the homeowner’s negligence
was a proximate cause of the driver’s injuries. (B) is incorrect. The issue upon which the homeown-
er’s liability to the driver turns is proximate cause; it is immaterial to that analysis whether the
homeowner’s negligence is established by a showing that he failed to exercise due care (indicated
by the facts) or that his conduct violated a statute and constituted negligence per se (also estab-
lished by the facts), which still leaves causation and damages to be established. Even if no statute
were applicable to the circumstances, the homeowner would still be liable to the driver, thus (B) is
not as good an answer as (A). (C) is incorrect. The homeowner’s negligence was a cause in fact of
the driver’s injuries, because it was the homeowner’s negligence which both prompted the driver to
attempt a rescue of the occupants of the burning residence, and brought the emergency vehicle onto
the scene. It is possible for the separate actions of two independently operating actors to combine to
injure the plaintiff, and whether either of them is liable for those injuries depends on the elements
of the negligence analysis, including proximate cause. Here, the homeowner breached the duty of
due care, and that breach was an actual and a proximate cause of the injury to the driver. (C) is
therefore inaccurate. (D) is incorrect because there is no evidence that the driver assumed the risk
of being struck by the emergency vehicle. Note that the “firefighter’s rule” may bar a firefighter
on assumption of risk or public policy grounds from recovering for injuries while responding to a
negligently caused fire; however, the rule does not apply to the driver here.

Answer to Question 95

(B) The defendant may amend with leave of the court. A party may amend a responsive pleading of
right within 21 days after serving it. Thereafter, according to Federal Rule 15, the party may amend
only with consent of all parties or with leave of the court, but the “court should freely grant leave
when justice so requires.” With so much time left for discovery and before trial, a court would
almost certainly grant leave to amend. (A) is incorrect because, as stated, the time for amendment
as of right is 21 days after serving it, not any time before trial as (A) implies. (C) is incorrect in that
due diligence in discovering the defense need not be shown. (D) is incorrect. The 21-day period
applies to amendment as of right, but a court may grant leave to amend after that period.

Answer to Question 96

(A) The Constitution is inapplicable because the bus company is a private company. The Supreme
Court has ruled that the grant of a franchise is not sufficient to create state action. [Jackson v.
Metropolitan Edison (1974)] Thus, there is no basis for a First Amendment claim against the bus
company. (B) is wrong; the Constitution is inapplicable because of the lack of state action. Also,
(B) is wrong based on the facts of the question. The speech restricted was political, not commer-
cial. Thus, even though commercial speech receives less protection than political speech, this
speech cannot properly be characterized as speech that seeks to induce business transactions. (C)
is wrong because the audience’s opposition to a message does not justify otherwise impermissible
censorship. Also, (C) wrongly assumes that the Constitution applies. (D) is wrong because the
availability of alternative avenues of expression does not justify otherwise impermissible censor-
ship. Moreover, it also wrongly assumes that the Constitution applies.

Answer to Question 97

(A) The programmer is likely to prevail if the statement is true. A defamatory statement is action-
able only if it is false. The programmer has a complete defense if her statement about the designer

MPQ 100 preview diagnostic exam and divider P.indd 94 12/22/2015 4:13:33 PM
MBE PREVIEW DIAGNOSTIC EXAM 95.

is true. (B) is incorrect. There likely was publication of the statement because it was made at a
crowded party and overheard by a third party; negligence is sufficient to establish this element.
(C) is incorrect for the same reason. The programmer could also be liable if she was negligent
in allowing third persons to overhear her statement. (D) is incorrect because the programmer’s
statement disparaging the designer’s competence in his profession is slander per se, and therefore
actionable, even without a showing of pecuniary loss.

Answer to Question 98

(A) The entire debt is discharged. The initial contract was for $30,000. By cashing the check in light
of the letter, the wholesaler has accepted the new payment terms. An accord and satisfaction
occurs when there is a contract dispute or some slight alteration of the debtor’s consideration, and
here the debtor (the retailer) paid the entire contract price immediately on receipt of the coats.
The acceptance of the new terms constitutes an accord; the cashing of the check represents the
satisfaction. (B) is incorrect because discharge of a debt on the basis of account stated occurs
when one payment is made to pay for a number of earlier invoices; that has not occurred here.
(C) is incorrect because the accord is a new contract. The letter is the offer inviting acceptance
by cashing the check. By cashing the check, the wholesaler accepted the discount provision. (D)
is incorrect. Although an accord and satisfaction may occur through the use of such wording on a
check, those words are not required when the offer of accord is communicated in another way.

Answer to Question 99

(A) The court should rule the writing to be admissible. The handwriting sample is relevant to the
issue of the identity of the bank robber and is admissible because it was properly obtained and
violated no rule of privilege. A handwriting sample is not testimonial in nature and, therefore,
does not require Fifth or Sixth Amendment protections. (B) is incorrect. The handwriting sample
is evidence of physical characteristics and not testimonial in nature and, therefore, not subject
to Fifth Amendment protections. Therefore, there is no requirement that Miranda warnings
be given, advising the defendant that the sample could be used against him. (C) is incorrect.
Because the privilege against self-incrimination does not apply, there is no right to refuse to give
the handwriting sample, provided that the content of the writing is not used against the defen-
dant. (D) is incorrect. The presence of counsel is not required at a scientific identification made
by the police for the purposes of investigation, such as taking a handwriting sample. [Gilbert v.
California (1967)]

Answer to Question 100

(C) The court should find for the corporation. Under the U.C.C., a seller may obtain specific perfor-
mance (i.e., force the goods on the buyer) where the buyer has not yet accepted the goods only
if the seller is unable to resell the goods at a reasonable price or the goods have been lost or
damaged after risk of loss has passed to the buyer. Because the first farm has plenty of time to
find another buyer and its apples are of a high quality, it will be able to resell the apples and
obtain damages, but it cannot obtain specific performance. (A) is incorrect because mere breach
of contract is not a sufficient basis for specific performance. (B) is evidence of a breach, namely
that the corporation had an existing contract with the first farm at the time it signed the contract
with the rival farm, but it does not address the issue of specific performance. (D) is an incorrect
statement of the law. Specific performance is an equitable remedy that is available when the legal
remedy is inadequate. However, because the first farm can easily resell its products, monetary
damages are sufficient.

MPQ 100 preview diagnostic exam and divider P.indd 95 12/22/2015 4:13:33 PM
MPQ 100 preview diagnostic exam and divider P.indd 96 12/22/2015 4:13:33 PM

You might also like